PDA

View Full Version : Một số kiến thức về hình trong các cuộc thi Olympic Toán !


ma 29
28-07-2008, 10:03 AM
MỘT SỐ KIẾN THỨC VỀ HÌNH TRONG CÁC CUỘC THI OLYMPIC TOÁN
(Mathscope.org)
Những kiến thức sau đây gồm một số kiến thức cơ sở để khám phá hình học olympiad hoặc là những kết quả đẹp nổi tiếng :hornytoro:.Bài viết này được soạn ra nhằm đáp ứng nhu cầu tra cứu ,học hỏi của nhiều bạn đọc.
Nó sẽ cần sự chung tay của nhiều thành viên !.
Đầu tiên mình sẽ giới thiệu mục lục và nếu ai biết phần kiến thức ấy thì có thể post lên , nhưng để đảm bảo cho tính hệ thống , chặt chẽ và dễ theo dõi của bài viết ,mình xin nêu một số quy ước như sau:
1) Mỗi bài viết đều phải vẽ hình minh họa.
2)Mỗi bài viết chỉ đề cập đến 1 đề mục kiến thức.
3) Phải đảm bảo thứ tự nêu trong mục lục.
4)Chúng tôi chỉ giữ lại những trao đổi có ích kể từ sau khi hoàn thành mục lục, điều đó có nghĩa là những trao đổi chen giữa không bị xóa lúc này nhưng sẽ bị xóa khi mục lục được hoàn tất..
Bây giờ sẽ là nội dung chính:)
A/ MỤC LỤC


I/ Một số định nghĩa ,định lí , điểm và đường đặc biệt không duy nhất :


I.1)Định lí Menelaus
I.2)Mở rộng định lí Menelaus theo diện tích
I.3)Định lí Menelaus cho tứ giác
I.4)Định lí Ceva
I.5)Định lí Ceva dạng sin
I.6)Định lí Desargues
I.7)Định lí Pappus
I.8)Một trường hợp đặc biệt của định lí Pappus qua góc nhìn hình xạ ảnh.
I.9)Đẳng thức Ptolemy
I.10)Bất đẳng thức Ptolemy
I.11)Định lí Pascal
I.12)Định lí Brianchon
I.13)Định lí Miquel
I.14)Công thức Carnot
I.15)Định lí Carnot
I.16)Định lí Brokard
I.17)Định lí Euler về khoảng cách giữa tâm 2 đường tròn nội, ngoại tiếp của tam giác
I.18)Định lí Euler về khoảng cách giữa tâm 2 đường tròn nội, ngoại tiếp của tứ giác (Định lí Fuss)
I.19)Định lí Casey
I.20)Định lí Stewart
I.21)Định lí Lyness
I.22)Định lí Lyness mở rộng (Bổ đề Sawayama)
I.23)Định lí Thébault
I.24)Công thức Jacobi liên quan đến tâm tỉ cự,định lí Lebnitz
I.25)Định lí Newton cho tứ giác ngoại tiếp
I.26)Định lí Breichneider
I.27)Định lí con nhím
I.28)Định lí Gergonne -Euler
I.29)Định lí Peletier
I.30)Định lí Miobiut
I.31)Định lí Viviani
I.32)Công thức Lagrange mở rộng
I.33) Đường thẳng Simson
I.34)Đường thẳng Steiner
I.35) Điểm Anti-Steiner (Định lí Collings)
I.36)Định lí Napoleon
I.37)Định lí Morley
I.38)Định lí con bướm với đường tròn
I.39)Định lí con bướm với cặp đường thẳng
I.40)Điểm Blaikie
I.41)Định lí chùm đường thẳng đồng quy
I.42)Đường tròn Apollonius
I.43)Định lí Blanchet
I.44)Định lí Blanchet mở rộng
I.45) Định lí Jacobi
I.46) Định lí Kiepert
I.47)Định lí Kariya
I.48)Cực trực giao
I.49)Khái niệm tam giác hình chiếu ,công thức Euler về diện tích tam giác hình chiếu
I.50)Khái niệm hai điểm đẳng giác
I.51)Khái niệm tứ giác toàn phần.
I.52)Đường thẳng Droz-Farny
I.53) Đường tròn Droz-Farny
I.54)Định lí Van Aubel về tứ giác và các hình vuông dựng trên cạnh
I.55)Hệ thức Van Aubel
I.56)Định lí Pithot
I.57)Định lí Johnson
I.58) Định lí Eyeball
I.59) Bổ đề Haruki
I.60)Bài toán Langley
I.61)Định lí Paul Yiu về đường tròn bàng tiếp.
I.62)Định lí Maxwell
I.63)Định lí Brahmagupta về tứ giác nội tiếp có hai đường chéo vuông góc.
I.64)Định lí Schooten
I.65)Định lí Bottema
I.66)Định lí Pompeiu
I.67)Định lí Zaslavsky
I.68)Định lí Archimedes
I.69) Định lí Urquhart
I.70)Định lí Mairon Walters
I.71)Định lí Poncelet về bán kính đường tròn nội tiếp,bàng tiếp trong tam giác vuông.
I.72)Định lí Hansen
I.73)Định lí Steinbart suy rộng
I.74)Định lí Monge & d'Alembert I
I.75)Định lí Monge & d'Alembert II
I.76)Định lí Steiner về bán kính các đường tròn.
I.77)Định lí Bellavitis
I.78)Định lí Feuer bach-Luchterhand:








II/Một số điểm và đường đặc biệt được xác định duy nhất với tam giác và tứ giác,tứ điểm:

Ở đây nếu không giải thích gì thêm thì yếu tố được hiểu là trong tam giác.



II.1) Đường thẳng Euler của tam giác
II.2)Đường tròn và tâm Euler
II.3)Đường đối trung, điểm Lemoine
II.4)Điểm Gergone,điểm Nobb, đường thẳng Gergone
II.5)Điểm Nagel
II.6)Điểm Brocard
II.7)Điểm Schiffler
II.8)Điểm Feuerbach
II.9)Điểm Kosnita
II.10)Điểm Musselman,định lí Paul Yiu về điểm Musselman
II.11)Khái niệm vòng cực của tam giác.
II.12)Điểm Gibert
II.13)Trục Lemoine
II.14)Tâm Morley
II.15) Tâm Spieker và đường thẳng Nagel
II.16)Hai điểm Fermat
II.17)Điểm Parry reflection.
II.18)Đường tròn Taylor ,tâm Taylor
II.19)Điểm Bevan
II.20)Điểm Vecten
II.21)Điểm Mittenpunkt
II.22)Điểm Napoleon
II.23)Đường tròn Adam
II.24)Tam giác Fuhrmann ,đường tròn Fuhrmann
II.25)Hình luc giác và đường tròn Lemoine thứ nhất
II.26)Hình lục giác và đường tròn Lemoine thứ hai
II.27)Điểm Euler của Tứ giác nội tiếp
II.28)Đường thẳng Steiner của tứ giác toàn phần
II.29)Đường thẳng Gauss của tứ giác toàn phần.
II.30) Điểm Miquel của tứ giác toàn phần
II.31)Đường tròn Miquel của tứ giác toàn phần
II.32)Hình bình hành Varignon của tứ giác .
II.33)Điểm Poncelet của tứ giác.



Ghi chú:Bản pdf đã có tại đây :[Only registered and activated users can see links]

ma 29
01-08-2008, 02:25 PM
B/MỘT SỐ KHÁI NIỆM,ĐỊNH LÍ.
Mình sẽ viết cái đầu tiên nha::)

I.1)Định lí Menelaus





Định lí: Cho tam giác ABC và 3 điểm M,N,P lần lượt thuộc BC,CA,AB.
Khi đó M,N,P thẳng hàng khi và chỉ khi:

\frac{\bar{MB}}{\bar{MC}} .\frac{\bar{NC}}{\bar{NA}}.\frac{\bar{PA}}{\bar{PB }} =1(1)
Chứng minh:

[Only registered and activated users can see links]
a)Khi M,N,P thẳng hàng.
Trên MN lấy 1 điểm Q sao cho AQ//BC
Theo Thales ;
\frac{\bar{NC}}{\bar{NA}}=\frac{\bar{MC}}{\bar{QA} }

\frac{\bar{PA}}{\bar{PB}}= \frac{\bar{QA}}{\bar{MB}}

Từ đó dễ có đẳng thức (1)trên.
b)Ngược lại ,khi có (1):
Giả sử PN cắt BC tại M'.
Theo phần trước ta có:

\frac{\bar{M'B}}{\bar{M'C}} .\frac{\bar{NC}}{\bar{NA}}.\frac{\bar{PA}}{\bar{PB }} =1

Kết hợp với (1) suy ra
\frac{\bar{MB}}{\bar{MC}}=\frac{\bar{M'B}}{\bar{M' C}}
Do đó M trùng M' tức là M,N,P thẳng hàng.

Vậy ta có điều phải chứng minh.

Mình đưa lên vài file ,còn dùng dc cho cả mấy phần sau nữa:)

ma 29
02-08-2008, 10:47 AM
Mình sẽ làm nốt phần về Menelaus :))

I.2)Mở rộng định lí Menelaus theo diện tích


Định lí:Cho tam giác ABC và 3 điểm M,N,P lần lượt nằm trên BC,CA,AB.Khi đó ta có:

\frac{S_{MNP}}{S_{ABC}}= \frac{ \bar{BM}.\bar{CN}.\bar{AP}-\bar{CM}.\bar{AN}.\bar{BP} }{\bar{AB} .\bar{BC}.\bar{CA}}

Chứng minh :(thamtuhoctro post)
[Only registered and activated users can see links]

Gọi e_1 ,e_2 ,e_3 là vector chỉ phương của BC, CA, AB.
Ta có:
\begin{array}{l}S\left[ {ABC} \right] = S\left[ {MAB} \right] + S\left[ {MCA} \right] \\ \Rightarrow S\left[ {ABC} \right] = S\left[ {PMA} \right] + S\left[ {PBM} \right] + S\left[ {NMC} \right] + S\left[ {NAM} \right] \\\Rightarrow S\left[ {ABC} \right] = S\left[ {MNP} \right] + S\left[ {BMP} \right] + S\left[ {CNM} \right] + S\left[ {APN} \right] \\ \end{array}
mặt khác :
\frac{{S\left[ {BMP} \right]}}{{S\left[ {ABC} \right]}} = \frac{{\overline {BM} .\overline {BP} .\sin \left( {e_1 ;e_2 } \right)}}{{\overline {BC} .\overline {BA} .\sin \left( {e_1 ;e_2 } \right)}} = \frac{{\overline {BM} .\overline {BP} }}{{\overline {BC} .\overline {BA} }}
tương tự:

\frac{{S\left[ {CNM} \right]}}{{S\left[ {ABC} \right]}} = \frac{{\overline {CN} .\overline {CM} }}{{\overline {CA} .\overline {CB} }}

\frac{{S\left[ {APN} \right]}}{{S\left[ {ABC} \right]}} = \frac{{\overline {AP} .\overline {AN} }}{{\overline {AB} .\overline {AC} }}
Ta suy ra:

\begin{array}{l}\frac{{S\left[ {MNP} \right]}}{{S\left[ {ABC} \right]}} = 1 - \frac{{S\left[ {BMP} \right]}}{{S\left[ {ABC} \right]}} - \frac{{S\left[ {CNM} \right]}}{{S\left[ {ABC} \right]}} - \frac{{S\left[ {APN} \right]}}{{S\left[ {ABC} \right]}} \\\Rightarrow \frac{{S\left[ {MNP} \right]}}{{S\left[{ABC}\right]}} = 1 - \frac{{\overline {BM} .\overline {BP} }}{{\overline {BC} .\overline {BA} }} - \frac{{\overline {CN} .\overline {CM} }}{{\overline {CA} .\overline {CB} }} - \frac{{\overline {AP} .\overline {AN}}}{{\overline {AB} .\overline {AC} }} \\\Rightarrow \frac{{S\left[ {MNP} \right]}}{{S\left[ {ABC} \right]}} = \frac{{\overline {BM} .\overline {CN} .\overline {AP} - \overline {CM} .\overline {AN} .\overline {BP} }}{{\overline {AB} .\overline {BC} .\overline {CA} }} \\ \end{array}
:D

ma 29
02-08-2008, 10:50 AM
I.3)Định lí Menelaus cho tứ giác:


Định lí:Cho tứ giác ABCD và một đường thẳng d cắt AB,BC,CD,DA lần lượt ở M,N,P,Q. Khi đó ta có:
\frac{\bar{MA}}{\bar{MB}} .\frac{\bar{NB}}{\bar{NC}}.\frac{\bar{PC}}{\bar{PD }} .\frac{\bar{QD}}{\bar{QA}}=1
[Only registered and activated users can see links]
Chứng minh:
Ta sẽ làm giống cách chứng minh ở tam giác:)
Trên d lấy hai điểm I,J sao cho AI//BJ//CD
Theo Thales ta có:
\frac{\bar{MA}}{\bar{MB}}=\frac{\bar{IA}}{\bar{JB} }
\frac{\bar{NB}}{\bar{NC}}=\frac{\bar{JB}}{\bar{PC} }
\frac{\bar{QD}}{\bar{QA}}=\frac{\bar{PD}}{\bar{IA} }
Từ đó dễ có điều cần chứng minh.

*Chú ý
1)Khi áp dụng cho tứ giác ,định lí Menelaus chỉ phát biểu dạng thuận bởi dạng đảo nói chung không đúng!
2) Các bạn thử suy nghĩ xem với dạng thuận như thế này thì có thể mở rộng cho đa giác được không? -Một vấn đề khá thú vị:)

bali
04-08-2008, 09:11 AM
I.4) Định lý Ceva



Định lý:
Cho tam giác ABC.Gọi E, F, G là ba điểm tương ứng nằm trên BC, CA, AB. Ba đường thẳng AE, BF, CG cắt nhau tại một điểm O khi và chỉ khi:
\frac {\overline{AG}}{\overline{GB}}.\frac {\overline{BE}}{\overline{EC}}.\frac {\overline{CF}}{\overline{FA}}=1
[Only registered and activated users can see links]
Chứng minh:
Phần thuận:
Giả sử ba đường thẳng AE, BF, CG cắt nhau tại một điểm O. TỪ A và C, kẻ các đường song song với BF, chúng lần lượt cắt CG và AE tại K, I tương ứng.
Ta có: \frac {\overline{CF}}{\overline{FA}}=\frac {\overline{CO}}{\overline{OK}} và \frac {\overline{CI}}{\overline{AK}}=\frac {\overline{CO}}{\overline{OK}} (Sử dụng định lý Thales)
\Rightarrow \frac {\overline{CF}}{\overline{FA}}=\frac {\overline{IC}}{\overline{AK}}. Các cặp tam giác đồng dạng IEC và OEB, AKG và BOG : \frac {\overline{BE}}{\overline{CE}}=\frac {\overline{BO}}{\overline{CI}} và \frac {\overline{AG}}{\overline{BG}}=\frac {\overline{AK}}{\overline{BO}}
Do đó: \frac {\overline{AG}}{\overline{GB}}.\frac {\overline{BE}}{\overline{EC}}.\frac {\overline{CF}}{\overline{FA}}=\frac {\overline{AK}}{\overline{OB}}.\frac {\overline{BO}}{\overline{IC}}.\frac {\overline{CI}}{\overline{AK}}=1

Phần đảo:
Giả sử ta có: \frac {\overline{AG}}{\overline{GB}}.\frac {\overline{BE}}{\overline{EC}}.\frac {\overline{CF}}{\overline{FA}}=1
Qua giao điểm của các đường thẳng AE và BF, kẻ đường thẳng CC_1 với C_1 nằm trên cạnh AB. Khi đó, theo chứng minh phần thuận:
\frac {\overline{AC_1}}{\overline{C_1 B}}.\frac {\overline{BE}}{\overline{EC}}.\frac {\overline{CF}}{\overline{FA}}=1=\frac {\overline{AG}}{\overline{GB}}.\frac {\overline{BE}}{\overline{EC}}.\frac {\overline{CF}}{\overline{FA}}=1
Suy ra \frac {\overline{AC_1}}{\overline{C_1 B}}=\frac {\overline{AG}}{\overline{GB}}, hay C_1 \equiv G, ta có điều phải chứng minh

bali
04-08-2008, 01:17 PM
I.5) Định lý Ceva sin



Định lý:
Gọi E, F, G là ba điểm tương ứng nằm trên các đường thẳng BC, CA, AB của tam giác ABC. Ba đường thẳng AE, BF, CG cắt nhau tại một điểm O khi và chỉ khi: \frac {\sin ABF}{\sin CBF}.\frac {\sin BCG}{\sin ACG}.\frac {\sin CAE}{\sin BAE}=1

[Only registered and activated users can see links]

Chứng minh:
Phần thuận: Giả sử AE, BF, CG đồng quy tại O. Khi đó hai tam giác ABE và ACE có cùng chiều cao hạ từ đỉnh A.
\Rightarrow \frac {BE}{EC}=\frac {S_{ABE}}{S_{ACE}}=\frac {AB.AE.\sin BAE}{AC.AE.\sin CAE}=\frac {AB.\sin BAE}{AC.\sin CAE}
Tương tự \frac {CF}{FA}=\frac {BC.\sin CBF}{BA.\sin ABF}
Và \frac {AG}{GB}=\frac {CA.\sin ACG}{CB.\sin BCG}
Nhân từng vế ba đẳng thức trên được: \frac {\sin ABF}{\sin CBF}.\frac {\sin BCG}{\sin ACG}.\frac {\sin CAE}{\sin BAE}=\frac {BE}{EC}.\frac {CF}{FA}.\frac {AG}{GB}=1 (Theo định lý Ceva)
Từ đó suy ra đpcm.

Phần đảo: CM tương tự phần đảo ở mục 4.

trung anh
04-08-2008, 01:50 PM
I.6) Định lý Desargues


Định lý:
Cho tam giác ABC và tam giác A'B'C'. Khi đó AA', BB', CC' đồng quy khi và chỉ khi các giao điểm của BC và B'C', CA và C'A', AB và A'B' thẳng hàng.

[Only registered and activated users can see links] ([Only registered and activated users can see links])


Chứng minh:

Gọi X, Y, Z là lần lượt là các giao điểm của các cặp cạnh BC và B’C’, CA và C’A’, AB và A’B’ .

Phần thuận:
Giả sử các đường thẳng AA’, BB’, CC’ đồng quy tại S. Ta chứng minh X, Y, Z thẳng hàng.
Áp dụng định lí Menelaus cho tam giác SBC với cát tuyến XB'C' ta có:
\frac{\bar{XB}}{\bar{XC}}.\frac{\bar{C'C}}{\bar{C' S}}.\frac{\bar{B'S}}{\bar{B'B}}=1 hay \frac{\bar{XB}}{\bar{XC}} = \frac{\bar{SC'}}{\bar{SB'}}.\frac{\bar{BB'}}{\bar{ CC'}}

Tương tự, ta có:
\frac{\bar{YC}}{\bar{YA}} = \frac{\bar{SA'}}{\bar{SC'}}.\frac{\bar{CC'}}{\bar{ AA'}} và \frac{\bar{ZA}}{\bar{ZB}} = \frac{\bar{SB'}}{\bar{SA'}}.\frac{\bar{AA'}}{\bar{ BB'}}

Nhân từng vế các đẳng thức trên lại với nhau, và theo định lí Menelaus suy ra X, Y, Z thẳng hàng.

Phần đảo:
Giả sử các điểm X, Y, Z thẳng hàng. Ta chứng minh các đường thẳng AA’, BB’, CC’ đồng quy.
Gọi S là giao điểm của AA’ và BB’. SC cắt đường thẳng AC’ tại C”.
Xét 2 tam giác ABC và A’B’C” có các đường nối các đỉnh tương ứng đồng quy, do đó theo phần thuận giao điểm của các cạnh tương ứng cũng đồng quy.
Ta thấy AB cắt A’B’ tại Z, AC cắt A’C” tại Y (do A’, C’, C” thẳng hàng), suy ra giao điểm X’ của BC và B’C” phải thuộc YZ. Tức là X’ là giao của YZ và BC nên X’ trùng với X.
Suy ra C” trùng với C’, hay AA’, BB’, CC’ đồng quy.

ma 29
07-08-2008, 03:33 PM
I.7)Định lí Pappus

Định lí: Cho ba điểm A,B,C nằm trên đường thẳng a, X,Y,Z nằm trên đường thẳng b.Gọi M,N,P lần lượt là giao điểm của các cặp đường thẳng (AY,BX) ,(AZ,CX),CY,BZ).
Khi đó M,N,P thẳng hàng.

Chứng minh:

[Only registered and activated users can see links]
Định lí này có một cách chứng minh dùng Menelaus ,nếu có điều kiện mình sẽ post lên,còn sau đây là một cách dựa trên kiến thức cơ sở về tỉ số kép và phép chiếu xuyên tâm.
Ta có bổ đề sau được chứng minh dễ dàng nhờ những hiểu biết ban đầu về tỉ số kép và phép chiếu xuyên tâm:

Bổ đề: Cho góc xOy và các điểm A,B,C thuộc Ox; D,E,F thuộc Oy.
Khi đó AD,BE,CF đồng quy khi và chỉ khi: (OABC) =(ODEF) .

Bổ đề trên bạn đọc tự chứng minh, bây giờ ta sẽ trở lại bài toán.
Kí hiệu F_E là phép chiếu xuyên tâm E.
Gọi T,Q lần lượt là giao điểm của BX và AZ; CX và BZ.
Sử dụng bổ đề trên thì ta sẽ cần chứng minh: (BTMX) =(BZPQ)

+)Trường hợp a//b bạn đọc hãy chứng minh nhờ Thales

+)Khi a không song song với b.Gọi S là giao của a và b.
Ta thấy:
Với :F_A: (BTMX) =(SZYX)
Với F_C : (SZYX)=(BZPQ)
Từ đó suy ra điều cần chứng minh.

ma 29
07-08-2008, 03:34 PM
I.8)Một trường hợp đặc biệt của định lí Pappus qua góc nhìn hình xạ ảnh.


Ở phần này chúng tôi chỉ dùng hình xạ ảnh để dẫn dắt đến kết quả còn nội dung định lí và cách chứng minh thì hoàn toàn phù hợp với kiến thức hình THCS!
Ta có kết quả sau liên quan đến hình xạ ảnh: Các đường thẳng song song với nhau thì gặp nhau tại một điểm ở vô cực và ngược lại .
Vận dụng vào định lí Pappus ở trên , cho các điểm A,B,C ra vô cực thì theo kết quả về hình xạ ảnh ta có
YM//ZN ( Vì YM,ZN cùng đi qua một điểm (A) ở vô cực )Tương tự thì :XN//YP,XM//ZP.
Và khi ấy M,N,P vẫn thẳng hàng. Ta phát biểu lại được một định lí đơn giản và hữu dụng sau đây:


Định lí:Trên mặt phẳng cho ba điểm X,Y,Z thẳng hàng và ba điểm M,N,P thỏa mãn XN//YP,YM//ZN,XM//ZP.
Khi đó ta cũng có M,N,P thẳng hàng.

Chứng minh:
[Only registered and activated users can see links]
Trường hợp MP//XYZ thì đơn giản,bạn đọc tự chứng minh.
Ta sẽ xét khi MP không song song với XYZ.
Gọi S là giao điểm của MP với XYZ.
Đường thẳng qua X song song với YP cắt MP ở N'. Bài toán sẽ được gải quyết nếu ta chứng minh được rằng ZN' // YM (Vì khi ấy N' trùng N).

Thật vậy,chú ý YP//XN', ZP//XM nên theo Thales ta có:
\frac{SY}{SZ} =\frac{SY}{SX} .\frac{SX}{SZ} = \frac{SP}{SN'} .\frac{SM}{SP} = \frac{SM}{SN'}
Đến đây theo Thales đảo ta suy ra ZN' //YM. Chứng minh được hoàn tất.!

long14893
07-08-2008, 03:41 PM
I.9)Đẳng thức Ptolemy

Định lí Với tứ giác nội tiếp ABCD thì:
AB.CD+AD.BC=AC.BD

Chứng minh:
[Only registered and activated users can see links]
Lấy điểm E thuộc AC sao cho \hat {DEC} =\hat {ADB}

\to \Delta ADB đồng dạng \Delta DEC \to \frac{AD}{DE} = \frac{DB}{DC}=\frac {AB}{EC} \to AB.DC = EC.DB

Tương tự \Delta ADE đồng dạng \Delta BDC \to AE.BD = AD.BC \to AD.BC+AB.CD = BD(EA+EC)=BD.AC (DPCM)

trung anh
07-08-2008, 05:36 PM
I.10) Bất đẳng thức Ptolemy


Định lý:
Cho tứ giác ABCD. Khi đó có AC.BD \leq AB.CD + AD.BC
[Only registered and activated users can see links]
Chứng minh:
Lấy E nằm trong tứ giác ABCD sao cho
\hat{EDC}=\hat{ADB} và \hat{ECD}=\hat{ABD}
Khi đó \Delta ABD ~ \Delta ECD \Rightarrow \frac{AB}{BD}=\frac{EC}{DC} hay AB.DC=EC.BD.
Hơn nữa \Delta ADE ~ \Delta BDC (c.g.c) \Rightarrow \frac{AD}{AE}=\frac{BD}{CB} hay AD.CB=BD.AE.
Vậy ta có AB.CD + BC.AD = BD(EA+EC) \geq BD.AC(đpcm).
Xem thêm:[Only registered and activated users can see links]

trung anh
07-08-2008, 08:12 PM
I.11) Định lý Pascal


Định lý:
Cho 6 điểm A,B,C,D,E,F cùng thuộc một đường tròn. Khi đó các giao điểm của các cặp cạnh AB và DE, BC và EF, CD và FA thẳng hàng.

[Only registered and activated users can see links]
Chứng minh:
Gọi P,M,N lần lượt là giao điểm của AF và CD, AB và DE, BC và EF. Gọi P', M', N' lần lượt là giao điểm của BC và DE, BC và AF, DE và AF.
Áp dụng định lí Menelaus cho \Delta P'M'N' với cát tuyến PCD:
\frac{\bar{CP'}}{\bar{CM'}}.\frac{\bar{DN'}}{\bar{ DP'}}.\frac{\bar{PM'}}{\bar{PN'}}=1
\Leftrightarrow \frac{\bar{PM'}}{\bar{PN'}}=\frac{\bar{CM'}}{\bar{ CP'}}.\frac{\bar{DP'}}{\bar{DN'}}
Tương tự ta có:
\frac{\bar{NP'}}{\bar{NM'}}=\frac{\bar{FN'}}{\bar{ FM'}}.\frac{\bar{EP'}}{\bar{EN'}} và \frac{\bar{MN'}}{\bar{MP'}}=\frac{\bar{AN'}}{\bar{ AM'}}.\frac{\bar{BM'}}{\bar{BP'}}
Nhân các biểu thức trên lại kết hợp với các biểu thức phương tích sau:
\bar{BM'}.\bar{CM'}=\bar{AM'}.\bar{FM'}
\bar{EN'}.\bar{DN'}=\bar{FN'}.\bar{AN'}
\bar{CP'}.\bar{BP'}=\bar{DP'}.\bar{EP'}
Ta có :
\frac{\bar{NP'}}{\bar{NM'}}.\frac{\bar{MN'}}{\bar{ MP'}}.\frac{\bar{PM'}}{\bar{PN'}}=1.
Áp dụng định lí Menelaus đảo ta có đpcm.

Các bạn có thể vào đây xem thêm:[Only registered and activated users can see links]

trung anh
07-08-2008, 09:22 PM
I.12) Định lý Brianchon


Định lý:
Cho lục giác ABCDEF ngoại tiếp (O). Chứng minh rằng ba đường chéo lớn AD, BE, CF đồng quy.

[Only registered and activated users can see links]
Chứng minh:
Ta kí hiệu các tiếp điểm của (O) trên AB,BC,CD,DE,EF,FA lần lượt là M,N,P,Q,R,S. Xét cực và đối cực đối với (O). Gọi K,I,J lần lượt là giao điểm của các cặp đường thẳng (SM,PQ) ,(MN,QR),(NP,RS). Vì SM và PQ là đường đối cực của A và D nên AD là đường đối cực của K. Tương tự BE và FC lần lượt là đường đối cực của I và J.
Dùng định lí Pascal cho lục giác nội tiếp MNPQRS ta có I,J,K thẳng hàng. Nên ta có các đường đối cực của I,J,K (lần lượt là BE,CF,AD) cùng đi qua cực của đường thẳng này (đường thẳng đi qua I,J,K) nên AD,BE,CF đồng quy (đpcm).
Tương tự ngược lại có thế chứng minh định lí pascal thông qua Brianchon và cực đối cực(xem thêm cực đối cực ở mục III.5 hoặc xem [Only registered and activated users can see links]).

ma 29
08-08-2008, 05:56 PM
I.13)Định lí Miquel

Định lí: Cho tam giác ABC và ba điểm M,N,P lần lượt nằm trên BC,CA,AB. Khi đó các đường tròn ngoại tiếp các tam giác APN,BPM và CMN đồng quy.

Chứng minh:
[Only registered and activated users can see links]
Gọi S là giao điểm của (BPM) và (CMN).Ta sẽ chứng minh S nằm trên (APN).
Thật vậy:

(SN,SP) \equiv (SN,SM) + (SM,SP) \equiv (CN,CM) +(BM,BP) \equiv( CA,CB) + (BC,BA) \equiv (CA,BA) \equiv (AN,AP) (mod \pi)

Suy ra điều cần chứng minh.!

trung anh
08-08-2008, 06:57 PM
I.14) Công thức Carnot


Định lý:
Cho \Delta ABC nội tiếp (O,R). Gọi x,y,z lần lượt là khoảng cách từ O đến BC,AC,AB. Gọi r là bán kính đường tròn nội tiếp tam giác ABC. Ta có:
a)Nếu \Delta ABC nhọn thì công thức Carnot là x+y+z=R+r.
b)Nếu \hat{A}>9O^0 thì công thức carno là y+z-x=R+r
Chứng minh:

a)Nếu \Delta ABC nhọn
[Only registered and activated users can see links]
Gọi F, E, D lần lượt là trung điểm của BC,CA,AB. Như vậy ta có
OF=x, OE=y, OD=z. Đặt BC=a, CA=b, AB=c.
Áp đụng bất đẳng thức Ptolemy cho tứ giác nội tiếp OFBD ta có:
OB.DF=OF.DB+FB.OD hay R.\frac{b}{2}=z.\frac{a}{2}+x.\frac{c}{2}
Tương tự ta có R.\frac{c}{2}=y.\frac{a}{2}+x.\frac{b}{2} và R.\frac{a}{2}=y.\frac{c}{2}+z.\frac{b}{2}
ta lại có r(\frac{b}{2}+\frac{a}{2}+\frac{c}{2})= S_{ABC} = S_{OBC}+S_{AOC}+S_{ABO}=x.\frac{a}{2}+y.\frac{b}{2 }+z.\frac{c}{2}
Cộng bốn biểu thức trên lại ta có
(r+R)(\frac{a+b+c}{2})=(x+y+z)(\frac{a+b+c}{2}) \Rightarrow R+r=x+y+z

b)Nếu \hat{A}>9O^0 chứng minh tương tự.
[Only registered and activated users can see links]

Viết dưới dạng lượng giác, công thức Carnot chính là hệ thức cosA + cosB + cosC = 1 + \frac{r}{R}. Chú ý hệ thức này đúng với mọi tam giác.

trung anh
08-08-2008, 08:42 PM
I.15) Định lí Carnot


Định lý:
Cho \Delta ABC. Gọi M, N, P lần lượt là các điểm thuộc các cạnh BC,AC,AB. d_M, d_N, d_P lần lượt là các đường thẳng đi qua M, N, P và vuông góc với BC, AC, AB. d_M, d_N, d_P đồng quy khi và chỉ khi
MB^2+NC^2+PA^2=MC^2+NA^2+PB^2

[Only registered and activated users can see links]

Chứng minh:
a)Phần thuận:
Gọi d_M, d_N, d_P đồng quy tại O.
ĐPCM \Leftrightarrow MB^2+OM^2+NC^2+ON^2+PA^2+OP^2=MC^2+OM^2+NA^2+ON^2+ PB^2+OP^2
\Leftrightarrow OB^2+OC^2+OA^2=OB^2+OC^2+OA^2
Đẳng thức này đúng nên ta có điều phải chứng minh.
b) Phần đảo
Gọi giao điểm của d_M, d_N tại O. Qua O hạ đường vuông góc xuống AB tại P'. Áp dụng định lí thuận ta có MB^2+NC^2+P'A^2=MC^2+NA^2+P'B^2 \Rightarrow P trùng với P' \Rightarrow d_M, d_N, d_P đồng quy.
Các bạn có thể vào đây xem vài điều liên quan:[Only registered and activated users can see links]

nbkschool
11-08-2008, 07:40 PM
Hix vậy để em post cái khác đúng quy trình.:hugging:

I.16/Định lý Brokard

Định lý:
Cho tứ giác lồi ABCD nội tiếp đường tròn tâm O.AD giao BC tại M,AB giao CD tại N,AC giao BD tại I.Chứng minh rằng O là trực tâm của tam giác MIN.

[Only registered and activated users can see links]

Chứng minh:

Gọi H là giao thứ 2 của hai đường tròn ngoại tiếp các tam giác AID,BIC.
Xét tứ giác DOHC,ta có:
\hat{DHC}=\360^o -\hat{DHI}-\hat{CHI}=\hat{DAC}+\hat{DBC}=\hat{DOC}
Từ đó suy ra tứ giác DOHC nội tiếp.Tương tự ta cũng suy ra tứ giác AOHB nội tiếp.
Dễ thấy \overline{NA}. \overline{NB}=\overline{NC}. \overline{ND} suy ra N nằm trên trục đẳng phương của hai đường tròn (AIHD),(BIHC)-->O,H,N thẳng hàng.
Ta có:
\hat{IHO}=\hat{IHD}-\hat{OHD}=\hat{ADC}+\hat{ACD}-\hat{OCD}=\hat{OCA}+\hat{ODA}+\hat{ODC}
=\90^o
Từ đó suy ra IM \perp ON
Tương tự ta có:IN \perp OM
Suy ra O là trực tâm tam giác MIN (đpcm)

******T.Anh:Định lý này sử dụng cách chứng minh bằng cực đối cực sẽ nhanh hơn rất nhiều: Xem bài toán số 2 phần I mục C trong bài viết [Only registered and activated users can see links]
==============

trung anh
11-08-2008, 10:08 PM
I.17) Định lí Euler về khoảng cách giữa tâm 2 đường tròn nội, ngoại tiếp của tam giác



Định lý:
Cho tam giác ABC nội tiếp (O;R) và ngoại tiếp (I;r). Chứng minh rằng OI^2=R^2-2Rr.
[Only registered and activated users can see links]
Chứng minh:
Kéo dài AI cắt (O) tại M. Vẽ đường kính MN của đường tròn (O).
Hạ ID \perp AB. Kéo dài OI cắt (O) tại E và F. Ta có \Delta AID ~ \Delta NMC (g.g) \Rightarrow \frac{ID}{MC}=\frac{AI}{MN} \Rightarrow 2Rr=ID.MN=AI.MC.
Mặt khác dễ dàng chứng minh MC=MI \Rightarrow 2Rr=IA.IM
Lại có IA.IM=IE.IF=R^2-OI^2 nên ta có điều phải chứng minh.

ma 29
16-08-2008, 02:47 PM
I.18)Định lí Euler về khoảng cách giữa tâm hai đường tròn nội ngoại tiếp tứ giác!(Định lí Fuss)

Định lí :Cho tứ giác ABCD vừa nội tiếp (O,R) vừa ngoại tiếp (I,r). Đặt d=OI. Khi đó ta có:

\frac {1}{(R-d)^2} +\frac{1}{(R+d)^2} =\frac{1}{r^2}{


Chứng minh
[Only registered and activated users can see links]

Gọi tiếp điểm của (I) trên AB,BC,CD,DA lần lượt là M,N,P,Q.
BI,CI cắt (O) lần lượt ở E,F .
Ta thấy:(DE,DF) \equiv (DE,DC) +(DC,DF) \equiv (BE,BC) +(DC,DF) \equiv \frac{(BA,BC)+(DC,DA)}{2} \equiv \frac{\pi}{2} (mod \pi)

Do đó E,O,F thẳng hàng ,nên O là trung điểm của EF.
Theo công thức đường trung tuyến trong tam giác IEF ta có:
d^2 = IO^2 = \frac{IE^2}{2} + \frac{IF^2}{2} - \frac{EF^2}{4}=\frac{IE^2}{2} + \frac{IF^2}{2} -R^2
Từ đó suy ra:
\frac {1}{(R-d)^2} +\frac{1}{(R+d)^2} = \frac{2(R^2+d^2)}{(R^2-d^2)^2} = \frac{IE^2 + IF^2} {(P_{I/(O)}) ^2} = \frac{IE^2}{(P_{I/(O)}) ^2} +\frac{IF^2}{ (P_{I/(O)}) ^2}= \frac{IE^2}{ (IE.IB)^2} +\frac{IF^2}{(IF.ID)^2} = \frac{1}{ IB^2} + \frac{1}{ ID^2} = \frac{1}{ ({\frac{IM}{ \sin{\frac{B}{2}}}) ^2 }}+ \frac{1}{(\frac{IP}{ (\sin {\frac{D}{2}}})^2} =\frac{1}{r^2} (vì \frac{B}{2}+ \frac{D}{2}=90)

trung anh
16-08-2008, 03:27 PM
I.19)Định lí Casey(Định lí Ptolemy mở rộng)

Định lí :Cho tứ giác ABCD nội tiếp (O,R). Đặt các đường tròn \alpha, \beta, \gamma, \delta là các đường tròn tiếp xúc với (O) tại các đỉnh A,B,C,D. Đăt t_{\alpha\beta} là độ dài đoạn tiếp tuyến chung của hai đường tròn \alpha, \beta. Trong đó t_{\alpha\beta} là độ dài tiếp tuyến chung ngoài nếu hai đường tròn \alpha, \beta cùng tiếp xúc trong hoặc cùng tiếp xúc ngoài với (O), và là độ dài đoạn tiếp xúc trong nếu trong trường hợp còn lại. Các đoạn t_{\beta\gamma}, t_{\gamma\delta}, ... được xác định tương tự. Khi đó ta có:
t_{\alpha\beta}.t_{\gamma\delta}+t_{\beta\gamma}.t _{\alpha\delta}=t_{\alpha\gamma}.t_{\beta\delta}

[Only registered and activated users can see links]

Chứng minh
Ta chứng minh trường hợp \alpha, \beta, \gamma, \delta cùng tiếp xúc ngoài với (O). Các trường hợp còn lại chứng minh tương tự. Lần lượt đặt tâm các đường tròn trên là A',B',C',D' và bán kính lần lượt là x,y,z,t.
Đặt AB=a, BC=b, CD=c, DA=d, AC=m, BD=n.
Áp dụng định lý Pythagore:
(t_{\alpha\beta})^2=A'B' ^2- (x-y)^2
Mặt khác lại có: (theo định lí hàm số cos)
A'B' ^2=(R+x)^2+(R+y)^2-2(R+x)(R+y)cos(\hat{A'OB'})
A'B' ^2=(R+x)^2+(R+y)^2-2(R+x)(R+y)(1-\frac{a^2}{2R^2})
A'B' ^2=(R+x)^2-2(R+x)(R+y)+(R+y)^2+(R+x)(R+y).\frac{a^2}{R^2}
A'B' ^2=(x-y)^2+\frac{a^2}{R^2}.(R+x)(R+y)
\Rightarrow t_{\alpha\beta}=\frac{a}{R}.\sqrt{(R+x)(R+y)}
Tương tự với t_{\beta\gamma}, t_{\gamma\delta}, ...
Ta có
t_{\alpha\beta}.t_{\gamma\delta}+t_{\beta\gamma}.t _{\alpha\delta}=t_{\alpha\gamma}.t_{\beta\delta} \Leftrightarrow a.c+b.d=m.n(định lý Ptolemy)
Ngược lại ta thấy định lý Ptolemy là một trường hợp đặc biệt của định lí Casey khi x=y=z=t=0.
Xem [Only registered and activated users can see links]

trung anh
16-08-2008, 05:55 PM
I.20)Hệ thức Stewart

Định lí:Cho ba điểm A,B,C thẳng hàng. Và một điểm M bất kì. Ta luôn có hệ thức
\bar{MA}^2.\bar{BC}+\bar{MB}^2.\bar{CA}+\bar{MC}^2 .\bar{AB}+\bar{BC}.\bar{CA}.\bar{AB}=0

[Only registered and activated users can see links]

Chứng minh
Qua M hạ MH \perp AC.
Ta có:
\bar{MA}^2.\bar{BC}+\bar{MB}^2.\bar{CA}+\bar{MC}^2 .\bar{AB}+\bar{BC}.\bar{CA}.\bar{AB}
=(MH^2+HA^2). \bar{BC}+(MH^2+HB^2).\bar{CA}+(MH^2+HC^2).\bar{AB} +\bar{BC}.\bar{CA}.\bar{AB}
=\bar{MH}^2.(\bar{BC}+\bar{CA}+\bar{AB})+(\bar{HA} ^2.\bar{BC}+\bar{HB}^2.\bar{CA}+\bar{HC}^2.\bar{AB }+\bar{BC}.\bar{CA}.\bar{AB})
=0+\bar{HA}^2.\bar{BC}+\bar{HB}^2.\bar{CA}+\bar{HC }^2.\bar{AB}+\bar{BC}.\bar{CA}.\bar{AB}
(Đưa về trường hợp hệ thức Stewart cho 4 điểm thẳng hàng (khi M nằm trên đường thẳng chứa A,B,C))
\bar{HA}^2.\bar{BC}+\bar{HB}^2.\bar{CA}+\bar{HC}^2 .\bar{AB}+\bar{BC}.\bar{CA}.\bar{AB}
=\bar{HA}^2.( \bar{HC}-\bar{HB})+\bar{HB}^2.(\bar{HA}-\bar{HC})+\bar{HC}^2.(\bar{HB}-\bar{HA})+(\bar{HC}-\bar{HB}).(\bar{HA}-\bar{HC}).(\bar{HB}-\bar{HA})=0
Ta có đpcm.

chu t tung
17-08-2008, 12:31 PM
I.21)Định lí Lyness
Định lí:Nếu đường tròn tâm O tiếp xúc trong với đường tròn ngoại tiếp tam giác ABC tại T và tiếp xúc với các cạnh AB,AC của tam giác lần lượt tại E và F thì tâm đường tròn nội tiếp của tam giác nằm trên EF.
[Only registered and activated users can see links]
Chứng minh:
Để chứng minh định lí này ta cần chứng minh 2 bổ đề sau:
Bổ đề 1:AB là dây của một đường tròn tâm (O). Đường tròn (l) tiếp xúc với dây AB tại K và tiếp xúc trong với (O) tại T. Chứng minh L là trung điểm của cung AB ko chứa T và LA^2=LK.LT
Bổ đề 2: Điểm M là trung điểm cung BC ko chứa A của đường tròn ngoại tiếp tam giác ABC. Điểm I thuộc đoạn MA sao cho MI=MB. Chứng minh rằng I là tâm đường tròn nội tiếp tam giác ABC.
Việc chứng minh 2 bổ đề này là khá đơn giản.
Ta tiếp tục quay trở lại với việc chứng minh định lí Lyness.
kẻ TF giao (O) tại P; BP cắt EF tại H.
Theo bổ đề 1 ta có BP là phân giác của góc B.
Ta có: \hat{HET}=\hat{HBT} (=\hat{FTx}) \Rightarrow HEBT nt \Rightarrow \hat{THB}=\hat{TEB}
Mà \hat{TEB}=\hat{TFE} \Rightarrow \hat{TFE}=\hat{THB}
\Rightarrow \Delta PHF \sim \Delta PTH \Rightarrow PH^2 = PF.PT
Theo bổ đề 1 ta lại có PC^2 = PF.PT \Rightarrow PC=PH
Theo bổ đề 2 ta được H là tâm đường tròn nội tiếp tam giác ABC (ĐPCM)
Lần đầu vẽ hình xấu quá nhỉ!!!!!:))

trung anh
17-08-2008, 05:24 PM
I.22)Định lý Lyness mở rộng(Bổ đề Sawayama)

Định lí:Cho tam giác ABC nội tiếp đường tròn (O).M thuộc BC (Có cách phát biểu khác là: cho tứ giác ABDC và M là giao của BC và AD; nhưng hai cách phát biểu này là tương đương). Một đường tròn (O') tiếp xúc với hai cạnh MA và MC tại E và F đồng thời tiếp xúc với cả đường tròn (O) tại K. Khi đó ta có tâm đường tròn nội tiếp của tam giác ABC nằm trên đường thẳng EF.

[Only registered and activated users can see links]

Chứng minh
KF cát đường tròn (O) tại G. Áp dụng bổ đề 1 tại bài viết của chu t tung về định lý Lyness ở trên, ta có G là điểm chính giữa cung BC. Gọi I là giao của AG với EF. Ta có
\hat{IEK}=\hat{IAK}=\hat{FKD} \Rightarrow AEIK nội tiếp \Rightarrow \hat{AIK} = \hat{EFK}=\hat{AEK}
\Rightarrow \Delta EFK ~ \Delta IAK (g.g) \Rightarrow \hat{EKA}=\hat{GKI}=\hat{GIF}
\Rightarrow \Delta GIF ~ \Delta GKI (g.g) \Rightarrow GI^2=GF.GK
Lại cũng theo bổ đề 1 ta có GC^2=GF.GK \Rightarrow GC=GI \Rightarrow I là tâm nội tiếp của \Delta ABC(theo bổ đề 2)
Xem thêm các hệ quả của định lý Lyness tại báo toán tuổi thơ 2 số 42 và 43

chu t tung
17-08-2008, 05:42 PM
I.23) Định lí Thébault
Định lí: Cho tam giác ABC nội tiếp đường tròn (O). D là một điểm nằm trên cạnh BC. Đường tròn tâm P tiếp xúc với 2 đoạn AD,DC và tiếp xúc trong với (O). Đường tròn tâm Q tiếp xúc với 2 đoạn AD,DB và tiếp xúc trong với (O). Gọi I là tâm nội tiếp tam giác ABC. Ta có: P,I,Q thẳng hàng.
Chứng minh
[Only registered and activated users can see links]
Gọi G,H lần lượt là tiếp điểm của (Q) với DB,AD. Gọi I là giao điểm của EF và GH. Theo định lí lyness mở rộng(đã có trong bài của trung anh), I là tâm nội tiếp tam giác ABC. Vậy ta chỉ cần chứng minh P,I,Q thẳng hàng. Thật vậy, gọi X,Y lần lượt là giao điểm của GH và DQ; EF và DP. Áp dụng định lí Thales ta có: \frac{IX}{PD}=\frac{YD}{PD}=\frac{QX}{QD}. Vậy , P,I,Q thẳng hàng(dpcm)

ma 29
18-08-2008, 11:30 AM
Cứ tiến độ này chắc sang năm mới cái đống kiến thức hình này vẫn ko hoàn thành nổi mất thôi !!!!!:tire: Mọi người cố gắng đóng góp đi chứ:hornytoro:

Chú yên tâm ,sẽ xong trong năm nay! :hornytoro:
Nếu anh còn sống đến cuối năm thì anh cam đoan điều đó.!:nemoflow:
Mọi người cứ cố gắng nhá!!!:hornytoro::D

nbkschool
18-08-2008, 10:33 PM
I.24)Công thức Jacobi liên quan đến tâm tỉ cự,định lí Lebnitz

1)Công thức Jacobi:

Nếu I là tâm tỉ cự của hệ điểm {A_1,A_2,...,A_n} ứng với các hệ số a_1,a_2,...,a_n thì với mọi điểm M trên mặt phẳng ta đều có:
\sum^n_{i=1} a_iMA_i^2=\sum^n_{i=1}a_iIA_i^2+(a_1+...+a_n)MI^2

Chứng minh:

\sum^n_{i=1} a_iMA_i^2=\sum^n_{i=1} a_i(\vec{MI}+\vec{IA_i})^2=(a_1+...+a_n)MI^2+2\vec {MI}( \sum^n_{i=1} a_i.\vec{IA_i})+\sum^n_{i=1} a_i.IA_i^2
=(a_1+...+a_n)MI^2+\sum^n_{i=1}a_iIA_i^2
(do I là tâm tỉ cự của hệ điểm nên \sum^n_{i=1} a_i.\vec{IA_i}=0)
->đpcm.

2)Định lí Lebnitz Đây là trường hợp đặc biệt của công thức trên khi n=3

3)Hệ quả khác:
Giá trị nhỏ nhất của biểu thức \sum^n_{i=1} a_iMA_i^2 (với các kí hiệu như phần trên) đạt được khi M \equiv I.
(phần này thuần về tính toán nên chắc không cần vẽ hình phải không anh ma29?)

nbkschool
21-08-2008, 11:53 AM
I.25)Định lí Newton cho tứ giác ngoại tiếp

Định lý
Cho tứ giác ABCD ngoại tiếp đường tròn O.Khi đó trung điểm hai đường chéo AC,BD và tâm O thẳng hàng.

Hình: (vẽ bằng Cabri hơi xấu):rokeyrulez:

[Only registered and activated users can see links]

Chứng minh
Gọi P,Q,R,S lần lượt là các tiếp điểm của các đoạn thẳng AB,BC,CD,DA đối với đường tròn (O).
Đặt SA=AP=a,BP=BQ=b,CQ=CR=c,DR=DS=d.Áp dụng định lý con nhím cho tứ giác ABCD ta có:
(a+b)\vec{OP}+(b+c)\vec{OQ}+(c+d)\vec{OR}+(d+a) \vec{OS}=0
<->\sum (a+b)[\frac{b}{a+b}\vec{OA}+\frac{a}{a+b}\vec{OB}]=0
<->(b+d)(\vec{OA}+\vec{OC}) +(a+c)(\vec{OB}+\vec{OD})=0
<->(b+d)\vec{OM}+(a+c)\vec{ON}=0
Từ đó suy ra hai vecto \vec{OM},\vec{ON} cùng phương->O,M,N thẳng hàng (đpcm)

trung anh
21-08-2008, 10:45 PM
I.26)Định lí Breichneider (định lý hàm số cos cho tứ giác)

Định lý
Cho tứ giác ABCD có độ dài các cạnh AB, BC, CD, DA lần lượt là a, b, c, d và độ dài hai đường chéo AC, BD là m, n. Khi đó ta có: m^2n^2=a^2c^2+b^2d^2-2abcd.cos(A+C)

[Only registered and activated users can see links]

Chứng minh

Trên cạnh AB ra phía ngoài dựng tam giác ABN đồng dạng với tam giác CAD, và dựng ra phía ngoài cạnh AD tam giác ADM đồng dạng với tam giác CAB. Khi đó dễ thấy: AN = \frac{ac}{m}, AM = \frac{bd}{m}, NB = DM = \frac{ad}{m} và BDMN là hình bình hành.
Đồng thời có \hat{NAM}=\hat{A}+\hat{C}
Áp dụng đính lí hàm số cos cho tam giác NAM, ta có

n^2= (\frac{ac}{m})^2 + (\frac{bd}{m})^2 - 2(\frac{ac}{m}) . (\frac{bd}{m}) . cos(A+C)
\Leftrightarrow m^2n^2=a^2c^2+b^2d^2-2abcd.cos(A+C)

ma 29
23-08-2008, 08:12 AM
I.27)Định lí con nhím:


Định lí:Cho đa giác lồi A_1A_2A_3...A_n và các vecto \vec{e_1},\vec{e_2},...,\vec{e_n} là các vecto có độ dài bằng các cạnh A_1A_2,A_2A_3,...,A_nA_1 ,tương ứng vuông góc với các cạnh ấy và hướng ra phía ngoài đa giác.Thế thì :
\vec{e_1}+\vec{e_2}+...+\vec{e_n} =\vec{0}

[Only registered and activated users can see links]


Chứng minh:
Không giảm tính tổng quát ,giả sử đa giác A_1A_2A_3...A_n có hướng dương.
Xét phép quay vecto:
Q_{\frac{\pi}{2}} (\vec{e_1}+\vec{e_2}+...+\vec{e_n}) = Q_{\frac{\pi}{2}} \vec{e_1} + Q_{\frac{\pi}{2}} \vec{e_2}...+ Q_{\frac{\pi}{2}}\vec {e_n}= \vec{A_1A_2} + \vec{A_2A_3}+....+\vec{A_nA_1} =\vec{0}
Từ đó suy ra điều cần chứng minh.

ma 29
23-08-2008, 08:53 AM
I.28)Định lí Gergone-Euler.

Định lí:Xét tam giác ABC và một điểm S trong mặt phẳng.AS,BS,CS lần lượt cắt BC,CA,AB ở D,E,F.Khi đó ta có:

\frac{\bar{SD}}{\bar{AD}} + \frac{\bar{SE}}{\bar{BE}} +\frac{\bar{SF}}{\bar{CF}} =1
Chứng minh:

[Only registered and activated users can see links]
Kí hiệu S_{[..]} chỉ diện tích đại số của đa giác.
Ta thấy :
\frac{\bar{SD}}{\bar{AD}} = \frac{S_{[SBD]} }{S_{[ABD]}} = \frac {S_{[SDC]}}{S_{[ADC]}}=\frac{S_{[SBD]} +S_{[SDC]}}{S_{[ABD]}+S_{[ADC]}}= \frac{S_{[SBC]} }{ S_{[ABC]}} (1)
Tương tự ta có:
\frac{\bar{SE}}{\bar{BE}}=\frac{S_{[SCA]}}{S_{[BCA]}} (2)
\frac{\bar{SF}}{\bar{CF}}=\frac{S_{[SAB]}}{S_{[CAB]}} (3)

Cộng theo vế (1),(2)và (3) ta có điều cần chứng minh.

ma 29
23-08-2008, 09:30 AM
I.29)Định lí Peletier

ma 29
23-08-2008, 09:52 AM
I.30)Định lí Miobiut
[Only registered and activated users can see links]

Keke đánh y nguyên báo cho đỡ mệt:))

Định lí:Cho ngũ giác lồi

Chứng minh:

ma 29
23-08-2008, 09:58 AM
I.31)Định lí Viviani

Định lí: Trong tam giác đều ABC ta lấy 1 điểm S .Ta sẽ có tổng các khoảng cách từ điểm S tới ba cạnh sẽ có độ dài bằng 1 đường cao của tam giác.

Chứng minh:
[Only registered and activated users can see links]
Kẻ SD,SE,SF lần lượt vuông góc với BC,CA,AB.
Đặt độ dài cạnh tam giác ABC là a,độ dài đường cao AH là b.
Ta có :
BC.AH=a.b=2S_{ABC}= 2(S_{SBC} +S_{SCA}+S_{SAB}) = SD.BC+ SE.CA+SF.AB= a(SD+SE+SF)
Giản ước hai vế cho a ta có điều cần chứng minh.

nbkschool
25-08-2008, 06:16 PM
Xin lỗi về sự chậm trễ của em,kể từ hôm nay là mỗi ngày đều phải học trọn sáng chiều ở trường rùi,hixhix :((
I.32)Công thức Lagrange mở rộng.

Định lý:
Gọi I là tâm tỉ cự của hệ điểm \{A_1,...,A_n \} ứng với các hệ số a_1,...a_n thì với mọi điểm M:
\sum^n_{i=1}a_iMA_i^2=\frac{\sum_{1\leq i <j \leq n} a_ia_jA_iA_j^2}{\sum^n_{i=1} a_i}+(\sum^n_{i=1} a_i)MI^2

Chứng minh:

Từ hệ thức Jacobi (có thể xem ở mục I.24) thì ta chỉ cần chứng minh rằng:
\frac{\sum_{1\leq i <j \leq n} a_ia_jA_iA_j^2}{\sum^n_{i=1} a_i}=\sum^n_{i=1}a_iIA_i^2
Do I là tâm tỉ cự của hệ điểm nên:
(\sum^n_{i=1}a_i\vec{IA_i})^2=0
<->\sum^n_{i=1} a_i^2IA_i^2 + 2.(\sum_{1\leq i <j \leq n}a_ia_j\vec{IA_i}.\vec{IA_j}) =0
<->\sum^n_{i=1} a_i^2IA_i^2 +[\sum_{1\leq i <j \leq n}a_ia_j(IA_i^2+IA_j^2-A_iA_j^2)]=0
<->(\sum^n_{i=1} a_i)(\sum^n_{i=1} a_iIA_i^2)-\sum_{1\leq i <j \leq n} a_ia_jA_iA_j^2=0
<->\frac{\sum_{1\leq i <j \leq n} a_ia_jA_iA_j^2}{\sum^n_{i=1} a_i}=\sum^n_{i=1}a_iIA_i^2(đpcm)

chu t tung
25-08-2008, 07:14 PM
I.33) Đường thẳng Simson
Định lí:Cho \Delta ABC và điểm M nằm trên đường tròn ngoại tiếp tâm O của tam giác. Gọi N,P,Qlần lượt là hình chiếu vuông góc của M trên các đường thẳng BC,CA,AB thì chúng cùng thuộc một đường thẳng (đây gọi là đường thẳng Simson).
Chứng minh:
[Only registered and activated users can see links]
Dùng góc định hướng
Ta có:
(PN,PQ) \equiv(PN,PM) + (PM,PQ) (mod \pi)
\equiv(CN,CM) +(AM,AQ) (mod \pi)
\equiv (BC,MC) +(MA,BA) (mod\pi)
\equiv 0 (mod\pi)
Vậy N,P,Q thẳng hàng.

Các bạn có thể tham khảo thêm ở đây:[Only registered and activated users can see links]

trung anh
25-08-2008, 08:24 PM
I.34) Đường thẳng Steiner


Định lí:Cho \Delta ABC và điểm D nằm trên đường tròn ngoại tiếp tâm O của tam giác. Gọi A_2,B_2,C_2 lần lượt là điểm đối xứng với của D qua các đường thẳng BC,CA,AB thì chúng cùng thuộc một đường thẳng và đường thẳng này đi qua trực tâm H của tam giác ABC. Đường thẳng đó được gọi là đường thẳng steiner ứng với điểm D của tam giác ABC. Còn điểm D được gọi là điểm anti steiner.
[Only registered and activated users can see links]

Chứng minh:

Dễ thấy nếu gọi A_1, B_1,C_1 lần lượt là hình chiếu của D xuống ba cạnh của tam giác ABC thì C_1 là trung điểm của đoạn DC_1 và tương tự ta có A_2,B_2,C_2 thẳng hàng.
Ta có
(HC_2,HB_2) \equiv (HC_2,HB) + (HB,HC) + (HC,HB_2)
\equiv (H_CD,H_CB) +(HF,HE) + (H_BC,H_BD)
\equiv (AD,AB) +(AB,AC) + (AC,AD)
\equiv 0(mod \pi)
Vậy đường thẳng steiner đi qua H.
Từ đó ta có được tính chất rằng đường thẳng simson ứng với điểm D đi qua trung điểm của đoạn DH.
==============
CM = pp sơ cấp đi bạn.........
Đây là ý kiến của anh ma 29, bọn em phải làm theo thôi

trung anh
25-08-2008, 10:15 PM
I.35) Điểm Anti Steiner(Định lí Collings)


Định lí:Cho \Delta ABC và đường thẳng d đi qua H trực tâm của tam giác ABC . Gọi d_a,d_b,d_c lần lượt là đường thẳng đối xứng của d qua BC,AC,AB. Các đường thẳng đó đồng quy tại một điểm nằm trên đường tròn ngoại tiếp tam giác ABC(điểm anti steiner của d). Và d được gọi là đường thẳng steiner của điểm đó (gọi là G).
[Only registered and activated users can see links]

Chứng minh:

Gọi H_a, H_b, H_c lần lượt là hình chiếu của H qua ba cạnh \Rightarrow ba điểm này thuộc (O) ngoại tiếp tam giác ABC và H_A, H_B, H_C lần lượt thuộc d_a,d_b,d_c
(d_a,d_b)\equiv(d_a,BC)+(BC,CA)+(CA,d_b)\equiv(BC, d)+(BC,CA)+(d,CA)\equiv2(BC,CA)\equiv(CH_A,CH_B) (mod \pi)
Vậy nếu gọi giao điểm của d_a,d_b là G thì G thuộc đường tròn ngoại tiếp tam giác ABC. Tương tự ta có đpcm

Theo hình của bài đường thẳng steiner ta dễ thấy H_CD đối xứng với HC_2, H_BD đối xứng với HB_2
Vậy ta có d đúng là đường thẳng steiner của G.

Ta có một tính chất khác của điểm Anti Steiner như sau:
Định lí 2:
Gọi P là một điểm thuộc đường thẳng d. P_A,P_B,P_C lần lượt là điểm đối xứng với P qua các cạnh của tam giác ABC. Ta có các đường tròn (A,P_C,P_B),(B,P_C,P_A),(C,P_A,P_B) cùng đi qua điểm G
[Only registered and activated users can see links]
Chứng minh :
Dễ thấy
(AP_C;AP_B)\equiv2(AB,AC)(mod \pi)
Lại có theo chứng minh trên có:
(d_c,d_b)\equiv(GP_C,GP_B)\equiv2(AB,AC)(mod \pi)
Suy ra G thuộc (A,P_C,P_B). Tương tự có đpcm
Tham khảo : [Only registered and activated users can see links]
Mọi người thông cảm d_a,d_b,d_c em quên không kí hiệu vào hình nên mọi người cứ nhìn theo tên điểm thuộc đường đó để phân biệt vậy !!

ma 29
26-08-2008, 07:02 AM
Mình nghĩ nên song song cả 2 cách giải...........
Ko phải ai cũng hiểu góc định hướng là gì........(mình cũng lơ mơ lắm)
Mục đích của pic là phổ biến các định lý hình học cho tất cả mọi người lên mục tiêu đầu tiên phải là mọi người đọc đều hiểu chứ


Đúng là mục đích để phổ biến nhưng nếu phổ biến sai hoặc không chặt chẽ thì phổ biến làm gì.
Ở đây mấy bài đó mà dùng góc thường thì muốn chặt chẽ thì phải xét nhiều trường hợp lắm -------> ko ổn.
Còn đáng lí ra thì cần có mục riêng cho gdh nhưng trên MS có bài của anh mailancuctruc rồi đó.

Quyết định là dùng góc định hướng rồi và cám ơn Trung Anh
và Tùng đã chấp thuận:hornytoro::dreamer:



**Chú ý: Khi dùng góc định hướng,trong biến đổi cần thay dấu "=" bằng dấu "\equiv".
Có thể nhiều bạn sẽ thắc mắc về điều này vì trong những tài liệu hiện hành có rất nhiều tài liệu dùng dấu bằng.
Tuy nhiên có một điều đáng chú ý là : Khi dùng dấu đồng dư để biến đổi thì không ai phản đối còn dùng dấu bằng thì có người phản đối ( TS.Nguyễn Minh Hà ) (Mà theo mình dùng đồng dư là tốt nhất với hiểu biết THPT:hornytoro:) .Thế nên tốt nhất là cứ dùng dấu đồng dư.:hornytoro:


Một số bài dùng dấu bằng thì mình sẽ sửa dần:))

ma 29
26-08-2008, 10:07 AM
I.36)Định lí Napoleon

Định lí:Dựng ra phía ngoài tam giác ABC các tam giác đều BMC,CNA,APB và gọi D,E,F lần lượt là tâm của ba tam giác ấy. Khi đó ta có tam giác DEF đều.

Chứng minh:
[Only registered and activated users can see links]
Bài này có nhiều cách giải,nếu thuận lợi mình sẽ giới thiệu ,tuy nhiên ở đây mình sẽ trình bày một chứng minh ngắn gọn dựa trên phép quay vecto như sau:
Q_{\frac{\pi}{3}} (\vec{DE})= \frac{1}{3}Q_{\frac{\pi}{3}} (\vec{MN} +\vec{BA})=\frac{1}{3}Q_{\frac{\pi}{3}} (\vec{MC}+\vec{CN}+\vec{BA})= \frac{1}{3} (Q_{\frac{\pi}{3}} (\vec{MC}) +Q_{\frac{\pi}{3}} (\vec{CN}) + Q_{\frac{\pi}{3}} (\vec{BA}))= \frac{1}{3}(\vec{MB} + \vec{CA} +\vec{BP})=\vec{DF}
Từ đó có điều cần chứng minh.



****trunganh:Em xin gửi lên một số cách c/m khác và các tài liệu có liên quan
[Only registered and activated users can see links]
[Only registered and activated users can see links]

trung anh
26-08-2008, 08:23 PM
I.37)Định lí Morley

Định lí:
Trong tam giác ABC. D,E,F lần lượt là giao điểm của các đường chia ba góc trong và cùng kề các cạnh tam giác ABC. Khi đó ta có tam giác DEF đều và được gọi là tam giác Morley.

[Only registered and activated users can see links]

Chứng minh:

Để ngắn gọn ta đặt \hat{A}=3a và tương tự với các góc kia \Rightarrow a+b+c=60^0 Như hình vẽ kẻ các đường chia trong ở B và C và lần lượt cắt tại D,I. Dễ thấy ID là phân giác của góc \hat{BIC}. Tại D dựng góc \hat{EDF}=60^0 sao cho Di là phân giác của góc DEF và E thuộc CI và F thuộc BI \Rightarrow \Delta DEF đều.
Lấy D_1 và D_2 lần lượt là điểm dối xứng với D qua CI và BI \Rightarrow D_1 \in AC, D_2 \in AB và dễ dàng c/m được D_1EFD_2 là hình thang cân với D_1E=EF=D_2F
Vì định lí Morley chỉ có một trường hợp nên em xin phép chỉ sử dụng góc thường cho nó đơn giản:
\hat{DEC}=180^0-60^0-\hat{FEI}=120^0-(b+c)=60^0+a
\hat{ED_1D_2}=180^0-\hat{D_1EF}=180^0-(360^0-60^0-2\hat{DEC})=2a
Ta lại có D_1EFD_2 là hình thang cân và D_1E=D_2F=EF \Rightarrow trong đường tròn ngoại tiếp D_1EFD_2 thì sđ \stackrel{\frown}{D_1D_2}= 3a \Rightarrow A thuộc đường tròn (D_1EFD_2). Từ đó ta có đpcm

Định lý Morley có thể mở rộng các đường chia trong thành các đường chia ngoài, và có thể là giao của đường chia trong với đường chia ngoài(mỗi trường hợp này lại cho ta một tam giác Morley khác nhau và theo thống kê có 36 tam giác Morley như vậy). Sau đó bài toán còn được phát triển và tương ứng được đặt thêm nhiều định nghĩa mới như "góc lửng", "tam giác ngoại lai", "tập hợp đẳng cấu", ...
Sau đây là bài toán mở rộng nhất định lý Morley:
Nếu chia n (n nguyên dương, n \geq 3) tất cả các góc của một đa giác m cạnh, thì tất cả các giao của các đường thẳng là các đỉnh phân biệt của một hệ \frac{1}{2}m(m-1)n(n-1)^2 đa giác n cạnh đều, có thể phân chia làm \frac{1}{2}m(m-1) họ, mỗi họ có n(n-1)^2 đa giác có tâm thẳng hàng.
Cách chứng minh và các khái niệm liên quan xin xem thêm tại sách "Lãng mạn toán học" tác giả Hoàng Quý nhà xuất bản giáo dục
(Ai có ebook của quyển này up lên thì tốt quá)

trung anh
26-08-2008, 10:18 PM
I.38)Định lí con bướm với đường tròn

Định lí:
Cho đường tròn (O) và dây cung AB. I là trung điểm của AB. Qua I vẽ hai dây cung tùy ý MN và PQ sao cho MP và NQ cắt AB tại E,F. Khi đó I là trung điểm của EF.

[Only registered and activated users can see links]

Chứng minh:
Gọi K,T là trung điểm MP và NQ. Nên OIEK, OIFT là tứ giác nội tiếp \Rightarrow (OE,OI)\equiv(KE,KI)(mod\pi); (OF,OI)\equiv(TF,TI)(mod\pi)
Ta lại có \Delta MIP \sim \Delta QIN \Rightarrow (TF,TI) \equiv (KE,KI)(mod\pi) \Rightarrow \Delta EOF cân tại O \Rightarrow I là trung điểm EF
Xem [Only registered and activated users can see links]

chu t tung
27-08-2008, 10:20 AM
I.39)Định lí con bướm với cặp đường thẳng
Định lí:Cho tam giác ABC. Lấy I là trung điểm của BC. Qua I kẻ các đường thẳng \Delta cắt AB,AC tại N,Q, đường thẳng \Delta' cắt AB,AC tại P,M. Gọi MN,PQ cắt BC tại F,E. Khi đó ta có I là trung điểm cưa EF
Chứng minh:
[Only registered and activated users can see links]
Áp dụng định lí menelaus trong tam giác ABC ta có các hệ thức sau:
\frac{\bar{EB}}{\bar{EC}}.\frac{\bar{QC}}{\bar{QA} }.\frac{\bar{PA}}{\bar{PB}}=1
\frac{\bar{FC}}{\bar{FB}}.\frac{\bar{NB}}{\bar{NA} }.\frac{\bar{MA}}{\bar{MC}}=1
\frac{\bar{IB}}{\bar{IC}}.\frac{\bar{QC}}{\bar{QA} }.\frac{\bar{NA}}{\bar{NB}}=1 \Rightarrow \frac{\bar{QC}}{\bar{QA}}.\frac{\bar{NA}}{\bar{NB} }=1 (1)
\frac{\bar{IC}}{\bar{IB}}.\frac{\bar{MA}}{\bar{MC} }.\frac{\bar{PB}}{\bar{PA}}=1 \Rightarrow \frac{\bar{MA}}{\bar{MC}}.\frac{\bar{PB}}{\bar{PA} }=1 (2)
từ (1) và (2) ta có:
\frac{\bar{QC}}{\bar{QA}}.\frac{\bar{PA}}{\bar{PB} }=\frac{\bar{NB}}{\bar{NA}}.\frac{\bar{MA}}{\bar{M C}} \Rightarrow \frac{\bar{EB}}{\bar{EC}}=\frac{\bar{FC}}{\bar{FB} }
Vậy I là trung điểm của EF. (ĐPCM)

Chí Thắng th
28-08-2008, 08:39 AM
I.6) Định lý Desargues


Định lý:
Cho tam giác ABC và tam giác A'B'C'. Khi đó AA', BB', CC' đồng quy khi và chỉ khi các giao điểm của BC và B'C', CA và C'A', AB và A'B' thẳng hàng.

[Only registered and activated users can see links] ([Only registered and activated users can see links])


Chứng minh:

Gọi X, Y, Z là lần lượt là các giao điểm của các cặp cạnh BC và B’C’, CA và C’A’, AB và A’B’ .

Phần thuận:
Giả sử các đường thẳng AA’, BB’, CC’ đồng quy tại S. Ta chứng minh X, Y, Z thẳng hàng.
Áp dụng định lí Menelaus cho tam giác SBC với cát tuyến XB'C' ta có:
\frac{\bar{XB}}{\bar{XC}}.\frac{\bar{C'C}}{\bar{C' S}}.\frac{\bar{B'S}}{\bar{B'B}}=1 hay \frac{\bar{XB}}{\bar{XC}} = \frac{\bar{SC'}}{\bar{SB'}}.\frac{\bar{BB'}}{\bar{ CC'}}

Tương tự, ta có:
\frac{\bar{YC}}{\bar{YA}} = \frac{\bar{SA'}}{\bar{SC'}}.\frac{\bar{CC'}}{\bar{ AA'}} và \frac{\bar{ZA}}{\bar{ZB}} = \frac{\bar{SB'}}{\bar{SA'}}.\frac{\bar{AA'}}{\bar{ BB'}}

Nhân từng vế các đẳng thức trên lại với nhau, và theo định lí Menelaus suy ra X, Y, Z thẳng hàng.

Phần đảo:
Giả sử các điểm X, Y, Z thẳng hàng. Ta chứng minh các đường thẳng AA’, BB’, CC’ đồng quy.
Gọi S là giao điểm của AA’ và BB’. SC cắt đường thẳng AC’ tại C”.
Xét 2 tam giác ABC và A’B’C” có các đường nối các đỉnh tương ứng đồng quy, do đó theo phần thuận giao điểm của các cạnh tương ứng cũng đồng quy.
Ta thấy AB cắt A’B’ tại Z, AC cắt A’C” tại Y (do A’, C’, C” thẳng hàng), suy ra giao điểm X’ của BC và B’C” phải thuộc YZ. Tức là X’ là giao của YZ và BC nên X’ trùng với X.
Suy ra C” trùng với C’, hay AA’, BB’, CC’ đồng quy.

Cái này nếu nhìn theo góc độ hình không gian thì đúng luôn. X,Y,Z cùng thuộc giao tuyến của mp(ABC) và mp(A'B'C'):hornytoro:

ma 29
28-08-2008, 08:48 AM
Cái này nếu nhìn theo góc độ hình không gian thì đúng luôn. X,Y,Z cùng thuộc giao tuyến của mp(ABC) và mp(A'B'C'):hornytoro:

Rất chính xác nếu (ABC) và (A'B'C') không đồng phẳng:hornytoro:

CHÚ Ý:
từ bây giờ đừng ai post những bài về cách giải khác ,hoặc là phát triển,ứng dụng ,.. của những định lí nhé.
Lí do: đó là những trao đổi hữu ích cần giữ lại,do đó khi hoàn thành các bài trong mục lục các bạn hẵng post,bởi post chen ngang thế này thì rồi lại bị xóa:))

***Chí Thắng vào làm cùng cho vui :)

thaithuan_GC
28-08-2008, 09:27 PM
I.40 Định Lí Blaikie
Định lí: Cho tam giác ABC và đường thẳng d sao cho d cắt BC,CA,AB lần lượt ở M,N,P. Gọi S là 1 điểm bất kì trên d. Gọi M',N',P' lần lượt là điểm đối xứng của M,N,P qua S. Khi đó AM',BN',CP' đồng quy tại một điểm P và ta gọi P là điểm Blaikie của d và S đối với tam giác ABC.


[Only registered and activated users can see links]
Chứng Minh :
Có thể cho S nằm giữa N,M.
Giả sử AM' cắt BN' tại I . Ta chứng mình I,C,P
thẳng hàng .
Xét tam giác BN'M với 3 điểm I,C,P . Ta cần cm :
\frac{IB}{IN'}.\frac{P'N'}{P'M}.\frac{CM}{CB}=1

Xét tam giác PBN' với 3 điểm thẳng hàng A,I,M' trên 3 cạnh :
\frac{AP}{AB}.\frac{IB}{IN'}.\frac{M'N'}{M'P}=1 (1)

Xét tam giác MBP với 3 điểm thẳng hàng C,A,N trên 3 cạnh :
\frac{CM}{CB}.\frac{AB}{AP}.\frac{PN}{MN}=1 (2)

Nhân 2 vế (1),(2) và rút gọn , chú ý MN=M'N' ta được :
\frac{IB}{IN'}.\frac{NP}{M'P}.\frac{CM}{CB}=1
Chú ý là NP=P'N' và P'M=M'P nên ta có đpcm. :))

ma 29
29-08-2008, 09:35 AM
I.41)Định lí chùm đường thẳng đồng quy.


Định lí 1: Ba đường thẳng đồng quy thì định ra trên hai đường thẳng song song những đoạn thẳng tỉ lệ.

Chứng minh:

trung anh
29-08-2008, 05:05 PM
Không biết có phải đưa về độ dài đại số không nhỉ ????
I.42)Đường tròn Apollonius



Định lí:Cho hai điểm A và B cố định. Khi đó quĩ tích điểm M sao cho \frac{MA}{MB}=k (k \neq 1) là một đường tròn cố định được gọi là đường tròn Apollonius.

[Only registered and activated users can see links]

Chứng minh:
Lấy D,E thuộc đường thẳng AB sao cho \frac{DA}{DB}=\frac{EA}{EB}=k
\Rightarrow (DEAB)=-1
a\Phần thuận
Ta có \frac{MA}{MB}=\frac{EA}{EB}=k \Rightarrow ME là phân giác của \hat{AMB} \Rightarrow \hat{EMD}=90^0 \Rightarrow M thuộc đường tròn đường kính DE.
b\Phần đảo
Lấy M' thuộc đường tròn đường kính DE \Rightarrow \hat{EMD}=90^0.
Lại có (DEAB)=-1 \Rightarrow ME là phân giác của \hat{AMB} \Rightarrow đpcm
***Chu t tung:Xem thêm [Only registered and activated users can see links]

trung anh
29-08-2008, 08:25 PM
I.43)Định lí Blanchet



Định lí:Cho tam giác ABC có AH là đường cao ứng với cạnh BC. Gọi I là một diểm tùy ý thuộc đoạn AH.các đoạn thẳng BI,CI cắt các cạnh tam giác tại E và F.Chứng minh rằng HA là phân giác của góc EHF

[Only registered and activated users can see links]
Chứng minh:
Qua I kẻ đường thẳng song song với BC cắt AB,HF,HE,AC lần lượt tại M,N,P,Q.
Ta chỉ cần chứng minh IN=IP là xong !!
Theo Thales:
\frac{IN}{IM} =\frac{CH}{CB}

\frac {IP} {IQ} = \frac{BH}{BC}
Do đó :
\frac{IN}{IP} . \frac{IQ}{IM} = \frac{CH}{BH}

Hiển nhiên \frac{IQ}{IM} = \frac{CH}{BH}
Nên IN =IP ---------------> dpcm
Các cách c/m khác [Only registered and activated users can see links]

chu t tung
29-08-2008, 09:18 PM
I.44)Mở rộng của định lí Blanchet
Định lí:Cho tam giác ABC, lấy T,E,F lần lượt thuộc các đoạn BC,CA,AB sao cho 3 đường thẳng AT,BE,CF đồng quy tại một điểm.Gọi L là giao điểm của AT và EF.Gọi H là hình chiếu của L xuống BC. Chứng minh rằng HL là phân giác của \hat{EHF}
Chứng minh:
[Only registered and activated users can see links]
Sử dụng Ceva và Menelaus ta chứng minh được (KTBC)=-1
Theo định lí về chùm điều hòa ta lại có (KLFE)=-1
\hat{LHK}=90^o------>đpcm (hệ quả của chùm điều hòa)

nbkschool
03-09-2008, 11:00 PM
I.45) Định lí Jacobi:

Định lí:
Cho tam giác ABC và các điểm A_1,B_1,C_1 trên mặt phẳng sao cho:\hat{BAC_1}=\hat{CAB_1}=\alpha,\hat{ABC_1}=\ha t{CBA_1}=\beta,\hat{BCA_1}=\hat{ACB_1}=\gamma.Khi đó AA_1,BB_1,CC_1 đồng quy tại điểm Jacobi N.

[Only registered and activated users can see links]

Chứng minh:

Do AA_1,BA_1,CA_1 đồng quy tại A_1 và tương tự cho A_2,A_3 nên áp dụng định lý Ceva dạng Sin ta có:
\frac{sin \hat{CBA_1}.sin \hat{BAA_1}.sin \hat{ACA_1}}{sin \hat{ABA_1}.sin \hat{CAA_1}.sin \hat{BCA_1}}=1
->\frac{sin \beta .sin \hat{BAA_1}.sin (\hat{ACB}+\gamma)}{sin (\hat{ABC}+ \beta).sin \hat{CAA_1}.sin \gamma}=1
Xây dựng hai đẳng thức tương tự cho A_2,A_3 rồi nhân 3 đẳng thức trên với nhau ta được:
\frac{\sin \hat{BAA_1}.\sin \hat{ACC_1}.\sin \hat{CBB_1}}{\sin \hat{CAA_1}.\sin \hat{BCC_1}.\sin \hat{ABB_1}}=1
Như vậy AA_1,BB_1,CC_1 đồng quy theo định lý Ceva dạng Sin.

ma 29
05-09-2008, 10:00 AM
I.46)Định lí Kiepert

Định lí:Dựng ra phía ngoài tam giác ABC các tam giác cân đồng dạng
BCM,CAN,ABP(Cân ở M,N,P).Khi ấy ta có AM,BN,CP đồng quy

Chứng minh
[Only registered and activated users can see links]
Do các tam giác BCM,CAN,ABP cân và đồng dạng nên dễ thấy:
\hat{MBC} =\hat{PBA} ; \hat{PAB}=\hat{NAC}; \hat{NCA}=\hat{MCB}
Theo định lí Jacobi ta có điều cần chứng minh.

ma 29
05-09-2008, 10:12 AM
I.47)Định lí Kariya

Định lí :
Cho tam giác ABC nhận (I) là đường tròn nội tiếp.Ở phía ngoài tam giác lấy các điểm M,N,P sao cho IM =IN=IP và IM,IN,IP tương ứng vuông góc BC,CA,AB.Khi đó ta có AM,BN,CP đồng quy.
Chứng minh:
[Only registered and activated users can see links]
Từ giả thiết bài toán dễ suy ra:
\Delta{BIM}=\Delta{BIP}(c.g.c)
Do đó:
\hat{IBM}=\hat{IBP}
=>\hat{MBC}=\hat{PBA}
Tương tự:
\hat{PAB}=\hat{NAC}; \hat{NCA}=\hat{MCB}
Theo định lí Jacobi ta có điều cần chứng minh.

trung anh
05-09-2008, 11:06 AM
I.48)Cực trực giao

Đây là một khái niệm mở rộng kết quả về trực tâm tam giác.

Định lí:
Cho tam giác ABC. d là một đường thẳng bất kì trong mặt phẳng. Gọi A_1,B_1,C_1 lần lượt là hình chiếu của A,B,C trên d. Gọi A_2,B_2,C_2 lần lượ là hình chiếu của A_1,B_1,C_1 trên BC,CA,AB. Khi đó A_1A_2,B_1B_2,C_1C_2 đồng quy tại một điểm gọi là cực trực giao của đường thẳng d đối với \Delta ABC.

[Only registered and activated users can see links]

Chứng minh:
Áp dụng định lí carnot ta có đpcm \Leftrightarrow
(A_2B^2-A_2C^2)+(B_2C^2-B_2A^2)+(C_2A^2-C_2B^2)=0
\Leftrightarrow (A_1B^2-A_1C^2)+(B_1C^2-B_1A^2)+(C_1A^2-C_1B^2)=0
\Leftrightarrow (A_1B^2-C_1B^2)+(B_1C^2-A_1C^2)+(C_1A^2-B_1A^2)=0
\Leftrightarrow (A_1B_1^2-C_1B_1^2)+(B_1C_1^2-A_1C_1^2)+(C_1A_1^2-B_1A_1^2)=0(hiển nhiên đúng)
Trực tâm là trường hợp khi d trùng với một trong ba cạnh của \Delta ABC

Các bạn có thể tìm thấy nhiều điều hữu ích liên quan ở đây:[Only registered and activated users can see links]

trung anh
05-09-2008, 01:47 PM
I.49)Khái niệm tam giác hình chiếu ,công thức Euler về diện tích tam giác hình chiếu

Định lí:
Cho (O,R) là đường tròn nội tiếp tam giác ABC.Cho điểm M nằm trong tam giác.Gọi A_1,B_1,C_1là hình chiếu của M lên ba cạnh BC,AC,AB. Khi đó ta gọi A_1,B_1,C_1 là tam giác hình chiếu của điểm M đối với tam giác ABC. Ta có công thức Euler về diện tích của tam giác hình chiếu:
\frac{S_{A_1B_1C_1}}{S_{ABC}}=\frac{|R^2-OM^2|}{4R^2}

[Only registered and activated users can see links]

Chứng minh:(Mashimaru)
Ta có tứ giác MA_1CB_1 nội tiếp đường tròn đường kính MC nên A_1B_1=MCsinC, tương tự thì B_1C_1=MAsinA.
Gọi D là giao điểm của MC với đường tròn ngoại tiếp tam giác ABC. Ta có: \widehat{MB_1A_1}=\widehat{MCA_1}=\widehat{BAD} và \widehat{C_1B_1M}=\widehat{C_1AM}.
Mặt khác:
\widehat{A_1B_1C_1}=\widehat{MB_1A_1}+\widehat{MB_ 1C_1}=\widehat{DAB}+\widehat{MAC_1}=\widehat{MAD}
Xét tam giác ADM, theo định lý sin ta có: \frac{AM}{sin\widehat{ADM}}=\frac{DM}{sin\widehat{ MAD}}=\frac{DM}{sin\widehat{A_1B_1C_1}}
Suy ra: AMsin\widehat{A_1B_1C_1}=DMsin\widehat{ADM}. Từ đó ta có:
S_{A_1B_1C_1}=\frac{1}{2}B_1A_1.C_1B_1.sin\widehat {A_1B_1C_1}=\frac{1}{2}.MCsinC.MAsinA.sin\widehat{ A_1B_1C_1}=\frac{1}{2}MC.MD.sinAsinBsinC
Mặt khác, ta lại có: S_{ABC}=2R^2sinAsinBsinC và MC.MD=P_{M/(O)}=|OM^2-R^2| nên:
\frac{S_{A_1B_1C_1}}{S_{ABC}}=|\frac{R^2-OM^2}{4R^2}|


Chú ý:Tam giác hình chiếu còn có tên gọi là tam giác bàn đạp hoặc tam giác thùy túc.

Và ở đây là tài liệu tiếng Anh bên MR:

trung anh
06-09-2008, 02:31 PM
I.50)Khái niệm hai điểm đẳng giác

Định lí:
Cho tam giác ABC. M là một điểm nằm trong tam giác.
1/Khi đó các đường thẳng đối xứng với AM,BM,CM qua tia phân giác đồng quy tại M'. M' được gọi là điểm đẳng giác của M.
2/Lần lượt đặt D,E,F và D',E',F' là chân các đường cao hạ từ M và M' xuông BC,AC,AB.
a/Khi đó D,E,F,D'E',F' cùng thuộc một đường tròn tâm O. Và O là trung điểm của M và M'.
b/Khi đó cũng có AM' \perp EF, BM' \perp FD, CM' \perp DE.
Và AM \perp E'F', BM \perp F'D', CM \perp D'E'



Chứng minh:
1/(hình 1)

[Only registered and activated users can see links]

\frac{S_{ABI}}{S_{ACJ}}=\frac{\bar{BI}}{\bar{CJ}}= \frac{\bar{AB}.\bar{AI}.sin(AB,AI)}{\bar{AC}.\bar{ AJ}.sin(AC,AJ)}
\frac{S_{ABJ}}{S_{ACI}}=\frac{\bar{BJ}}{\bar{CI}}= \frac{\bar{AB}.\bar{AJ}.sin(AB,AJ)}{\bar{AC}.\bar{ AI}.sin(AC,AI)}
\Rightarrow \frac{\bar{BI}}{\bar{CI}}.\frac{\bar{BJ}}{\bar{CJ} }=\frac{AB^2}{AC^2}
Tương tự áp dụng định lí Ceva thuận và đảo ta có đpcm1
2/
a/(hình 2)

[Only registered and activated users can see links]

(BA;BM) \equiv (BM';BC) (mod \pi)
\Leftrightarrow (DF;DM) \equiv (F'M';F'D') (mod \pi)
\Leftrightarrow (DF;DM) + 90^0 \equiv (F'M';F'D') + 90 ^0 (mod \pi)
\Leftrightarrow (DF;DM)+(DM;DD') \equiv (F'M';F'D')+(F'F;F'M') (mod \pi)
\Leftrightarrow (DF;DD') \equiv (F'F;F'D') (mod \pi)
\Rightarrow FF'D'D nội tiếp. Trung trực FF' và DD' gặp nhau tại trung điểm O của MM'(t/c đường trung bình hình thang) \Rightarrow F,F'D,D' thuộc đường tròn tâm O.
Tương tự ta có đpcm.
b/(hình 3)

[Only registered and activated users can see links]

\widehat{MAF}=\widehat{M'AE}
lại có \widehat{AMF}=\widehat{AEF} \Rightarrow \Delta AMF \sim \Delta AEG(g.g) \Rightarrow \widehat{AGE}=\widehat{AE'M'}=90^0 \Rightarrow
Tương tự ta có đpcm

Trường hợp M là trọng tâm thì M' là điểm đối trung (AM,BM,CM lần lượt được gọi là các đường đối trung) có tính chất rằng: (M'E'^2 + M'F'^2+ M'D'^2 ) min

trung anh
09-09-2008, 07:03 PM
Em đang đọc bên Cực đối cực thì thấy có link nhảy sang đây(liên hệ các định lí)
Em thấy nên đưa cả thêm phần định nghĩa về các phép biến hình. Ví dụ như phép nghịch đảo có ít bạn học sinh lớp 10 như em biết!!!!

ma 29
10-09-2008, 09:37 AM
Em đang đọc bên Cực đối cực thì thấy có link nhảy sang đây(liên hệ các định lí)
Em thấy nên đưa cả thêm phần định nghĩa về các phép biến hình. Ví dụ như phép nghịch đảo có ít bạn học sinh lớp 10 như em biết!!!!
Cái điều này cũng đang làm anh băn khoăn T.Anh ạ.Anh rất do dự ,nếu thêm phép nghich đảo thì chắc chắn sẽ cần thêm phép tuyến tính,phép quay vecto và nếu có thể dc thì sẽ là cả phép xạ ảnh,phép chiếu xuyên trục,...
Nói chung là khá nhiều nội dung (và dài)......
Mà hiện nay tài liệu về những thứ này đã có và hình như khá phổ biến ở VN nên anh cũng không định cho vào ..........
Những cái anh nêu ở trên đa phần là những thứ mà ít tài liệu đề cập rõ ràng lí thuyết thôi.

ma 29
10-09-2008, 05:51 PM
I.51)Khái niệm tứ giác toàn phần .


Khái niệm:
Một tứ giác toàn phần là một hình được tạo nên bởi bốn đường thẳng, từng đôi một cắt nhau nhưng không có ba đường nào đồng qui. Một hình tứ giác toàn phần có 4 cạnh là 4 đường thẳng ấy, có 6 đỉnh là 6 giao điểm của chúng và 3 đường chéo là 3 đoạn đi qua đỉnh đối diện (chú ý hai đỉnh này không cùng thuộc một cạnh).


Chúng ta có một kết quả cơ bản và thú vị về tứ giác này như sau:

Định lí :Trong hình tứ giác toàn phần cặp đỉnh đối diện nằm trên một đường chéo và cặp giao điểm của đường chéo đó với hai đường chéo còn lại lập thành một hàng điểm điều hòa.

Xem hình vẽ sau:
[Only registered and activated users can see links]

trung anh
18-09-2008, 10:39 PM
I.52)Đường thẳng Droz-Farny

Định lí:
Cho hai đường thẳng bất kì vuông góc với nhau tại trực tâm của tam giác ABC. Chúng tương ứng cắt các cạnh BC, AC, AB tại X, X'; Y, Y'; Z, Z'. Khi đó ta có M_a, M_b, M_c tương ứng là các trung điểm của XX',YY',ZZ' thẳng hàng

[Only registered and activated users can see links]

Chứng minh:(dịch từ bài viết của Jean-Louis Ayme)
Đặt C là đường tròn ngoại tiếp \Delta ABC. C_a là đường tròn ngoại tiếp HXX' và H_a là điểm đối xứng với H qua BC. Tương tự với các đường tròn khác.
\Rightarrow C_a,C_b,C_c có tâm lần lượt là M_a,M_b,M_c.XX' là đường kính của đường tròn C_a, H_a nằm trên đường tròn này \Rightarrow H_a là giao của C và C_a và HH_a \perp BC.
Áp dụng định lí Collings(trong bài viết điểm Anti-steiner) với đường thẳng XYZ đi qua H, ta có H_aX, H_bY, H_cZ đồng quy tại N trên C. Áp dụng định lí Miquel cho tam giác XNY với các điểm H_a, H_b, H \Rightarrow C, C_a,C_b cùng đi qua M. Tương tự C_c cũng đi qua M.
Như vậy C_a,C_b,C_c cùng đi qua H và M (đồng trục). Nên tâm của chúng thẳng hàng.
Các tài liệu có liên quan:

trung anh
19-09-2008, 01:58 PM
I.53)Đường tròn Droz-Farny

Định lí:
Cho điểm P bất kì và tam giác ABC. Điểm Q là điểm đẳng giác với P đối với tam giác ABC. Chân các đường vuông góc với các cạnh BC,AC,AB của P là P_a, P_b, P_c. Lấy P_a làm tâm vẽ đường tròn đi qua Q cắt BC tại A_1, A_2..B_1,B_2,C_1,C_2 định nghĩa tương tự. Khi đó A_1, A_2,B_1,B_2,C_1,C_2 cùng thuộc đường tròn tâm P.
[Only registered and activated users can see links]

Chứng minh:
O là trung điểm của PQ
Ta đã biết O cách đều P_a,P_b,P_c,Q_a,Q_b,Q_c
Mà :

PC_2^2=PP_c^2+P_cC_2^2=PP_c^2+QP_c^2=\frac{PQ^2}{2 }+2P_cO^2(công thức đường trung tuyến)
Hoàn toàn tương tự ta sẽ có các điểm A_1,A_2,B_1,B_2,C_1,C_2 cách đều P
(điều cần chứng minh).

nbkschool
11-10-2008, 11:36 PM
Em xin được bổ sung định lý sau: Định lý Maxwell
[Only registered and activated users can see links]

Ngoài ra khái niệm về hai tam giác orthologic với nhau và điểm orthotransversal chắc cũng nên đề cập tới (em thì chả biết gì về mấy cái này) :dumb:

ma 29
12-10-2008, 05:13 PM
Em xin được bổ sung định lý sau: Định lý Maxwell
[Only registered and activated users can see links]

Ngoài ra khái niệm về hai tam giác orthologic với nhau và điểm orthotransversal chắc cũng nên đề cập tới (em thì chả biết gì về mấy cái này) :dumb:

Đồng ý!:hornytoro:

ma 29
07-11-2008, 08:07 AM
I.54)Định lí Van Aubel về tứ giác và các hình vuông dựng trên cạnh.

Định lí:Về phía ngoài tứ giác ABCD ta dựng các hình vuông ABUI,BCQP,CDJW,DAFE với các tâm tương ứng là T,N,V,M.Khi đó ta có TV và MN vuông góc với nhau.

Chứng minh:
[Only registered and activated users can see links]
Gọi O là trung điểm đoạn AC.
Ta thấy:F_C^{45^0 ,\sqrt{2}} :N \to B

F_A^{45^0 ,\frac{1}{\sqrt{2}}} :B \to T

Do vậy:F=F_C^{45^0 ,\sqrt{2}} o F_A^{45^0 ,\frac{1}{\sqrt{2}}}:N \to T

Chú ý F là phép đồng dạng góc 90^0 với tỉ số 1 có O là điểm bất động tức là F là phép quay tâm O góc 90^0. (1)
Như vậy :F:N \to T(2)
Tương tự ta có:F:M \to V (3)

Từ (1) ,(2) và (3) ta có điều cần chứng minh.

p/s: ma 29 lại tiếp tục đánh phá topic này và rất mong các chiến hữu cũ và mới lại tham gia để hoàn thành nó trong năm nay................... :hornytoro:

ma 29
11-11-2008, 09:31 AM
I.2 Mở rộng định lí Menelaus theo diện tích.
Định lí:Cho tam giác ABC và 3 điểm M,N,P lần lượt nằm trên BC,CA,AB.Khi đó ta có:
\frac{{S\left[ {MNP} \right]}}{{S\left[ {ABC} \right]}} = \frac{{\overline {BM} .\overline {CN} .\overline {AP} - \overline {CM} .\overline {AN} .\overline {BP} }}{{\overline {AB} .\overline {BC} .\overline {CA} }}
Chứng minh

Gọi e_1 ,e_2 ,e_3 là vector chỉ phương của BC, CA, AB.
Ta có:
\begin{array}{l}S\left[ {ABC} \right] = S\left[ {MAB} \right] + S\left[ {MCA} \right] \\ \Rightarrow S\left[ {ABC} \right] = S\left[ {PMA} \right] + S\left[ {PBM} \right] + S\left[ {NMC} \right] + S\left[ {NAM} \right] \\\Rightarrow S\left[ {ABC} \right] = S\left[ {MNP} \right] + S\left[ {BMP} \right] + S\left[ {CNM} \right] + S\left[ {APN} \right] \\ \end{array}
mặt khác :
\frac{{S\left[ {BMP} \right]}}{{S\left[ {ABC} \right]}} = \frac{{\overline {BM} .\overline {BP} .\sin \left( {e_1 ;e_2 } \right)}}{{\overline {BC} .\overline {BA} .\sin \left( {e_1 ;e_2 } \right)}} = \frac{{\overline {BM} .\overline {BP} }}{{\overline {BC} .\overline {BA} }}
tương tự:

\frac{{S\left[ {CNM} \right]}}{{S\left[ {ABC} \right]}} = \frac{{\overline {CN} .\overline {CM} }}{{\overline {CA} .\overline {CB} }}

\frac{{S\left[ {APN} \right]}}{{S\left[ {ABC} \right]}} = \frac{{\overline {AP} .\overline {AN} }}{{\overline {AB} .\overline {AC} }}
Ta suy ra:

\begin{array}{l}\frac{{S\left[ {MNP} \right]}}{{S\left[ {ABC} \right]}} = 1 - \frac{{S\left[ {BMP} \right]}}{{S\left[ {ABC} \right]}} - \frac{{S\left[ {CNM} \right]}}{{S\left[ {ABC} \right]}} - \frac{{S\left[ {APN} \right]}}{{S\left[ {ABC} \right]}} \\\Rightarrow \frac{{S\left[ {MNP} \right]}}{{S\left[{ABC}\right]}} = 1 - \frac{{\overline {BM} .\overline {BP} }}{{\overline {BC} .\overline {BA} }} - \frac{{\overline {CN} .\overline {CM} }}{{\overline {CA} .\overline {CB} }} - \frac{{\overline {AP} .\overline {AN}}}{{\overline {AB} .\overline {AC} }} \\\Rightarrow \frac{{S\left[ {MNP} \right]}}{{S\left[ {ABC} \right]}} = \frac{{\overline {BM} .\overline {CN} .\overline {AP} - \overline {CM} .\overline {AN} .\overline {BP} }}{{\overline {AB} .\overline {BC} .\overline {CA} }} \\ \end{array}
:D

Một số bài cách đây khá xa nhưng em muốn giải quyết cho hoàn chỉnh chuyên đề vì thế em hi vọng các anh hãy chèn(nếu có thể) một vài bài viết của em vào phần còn thiếu:)) .Em cũng có ý kiến về một số định lí tới các anh có thể post trước phần nội dung định lí lên topic bởi em thấy một số định lí có thể có người biết nội dung nhưng tên gọi thì chưa thống nhất hay chưa rõ tên :)) và khi hoàn thành anh có thể xóa các bài này đi được rồi.

Em làm cái này tốt lắm tuy nhiên thật tiếc để đảm bảo thứ tự anh sẽ phải xóa bài này và gộp vào cái mục trên của anh.,rất mong em hiểu và thông cảm.
Bây giờ em hãy post theo đúng thứ tự nhé ,về tên gọi các định lí em hãy xem 2 web sau (là tài liệu tham khảo chính) :

1) [Only registered and activated users can see links]

2)[Only registered and activated users can see links]

ma 29
11-11-2008, 10:30 AM
I.55)Hệ thức Van Aubel

Định lí:Cho tam giác ABC và các điểm D,E,F lần lượt thuộc BC,CA,AB sao cho AD,BE,CF đồng quy ở S.Khi đó ta có:
\frac{\bar{AS}}{\bar{SD}} = \frac{\bar{AF}}{\bar{FB}} + \frac{\bar{AE}}{\bar{EC}}
Và 2 hệ thức tương tự.
Chứng minh:
[Only registered and activated users can see links]
Cách chứng minh dưới đây rất độc đáo mình đọc trong một file của anh Darij Grinberg(xem cuối bài)
Chúng ta sử dụng định lí Gergonne -Euler(mục I.28)cho điểm A với tam giác SBC.
Ta có:
\frac{\bar{AD}}{\bar{SD}}+ \frac{\bar{AF}}{\bar{BF}} +\frac{\bar{AE}}{\bar{CE}}=1


=>\frac{\bar{AD}}{\bar{SD}}-1= \frac{\bar{AF}}{\bar{FB}} +\frac{\bar{AE}}{\bar{EC}}

=>\frac{\bar{AS}}{\bar{SD}} = \frac{\bar{AF}}{\bar{FB}} + \frac{\bar{AE}}{\bar{EC}}

ma 29
12-11-2008, 02:23 PM
I.56)Định lí Pithot

Định lí: Tứ giác lồi ABCD là tứ giác ngoại tiếp khi và chỉ khi :AB+CD =BC+DA

Chứng minh:

+)Chiều thuận: ABCD là tứ giác ngoại tiếp thì: AB+CD=BC+DA

Phần này chứng minh hoàn toàn đơn giản xin dành cho bạn đọc.

+)Chiều đảo:Nếu tứ giác lồi ABCD thỏa mãn : AB+CD=BC+DA (*)thì nó là tứ giác ngoại tiếp.
[Only registered and activated users can see links]

Không giảm tổng quát giả sử :AB\leq AD ,khi ấy từ (*) cũng suy ra:BC \leq CD.
Khi ấy trên AD,DC tương ứng tồn tại hai điểm N,M sao cho:AN=AB,CM=CB.
Từ (*) sẽ có: DN=DM.

Do đó các đường phân giác của các góc tại đỉnh A,D,C sẽ là ba đường trung trực của tam giác BMN nên chúng đồng quy tại một điểm (mà ta kí hiệu là O).
Nhận thấy O cách đều 4 cạnh tứ giác nên ta có điều cần chứng minh.

ma 29
12-11-2008, 03:48 PM
I.57)Định lí Johnson

Định lí:Cho ba đường tròn có cùng bán kính R với tâm lần lượt là M,N,P và cùng đi qua một điểm A.Khi ấy ba giao điểm khác A của ba đường tròn ấy cùng nằm trên một đường tròn có bán kính là R.

Chứng minh:
[Only registered and activated users can see links]
Mình gặp định lí này và không hề biết chứng minh của nó ,ở đây mình trình bày một phép chứng minh mà mình nghĩ ra như sau mà theo mình nó là một chứng minh dài.
Chúng ta kí hiệu các giao điểm khác A là B,C,D như hình vẽ và gọi Q là tâm (BCD).
Ta thấy : PA=PB=MA=MB nên MAPB là hình thoi
=>M là điểm đối xứng của P qua AB.
Chú ý bán kính của (P) và (M) là bằng nhau nên suy ra (P) và (M) đối xứng với nhau qua AB.(1)
Tương tự (P) và (N) đối xứng với nhau qua AC (2).
(P) là đường tròn ngoại tiếp tam giác ABC (3)

Từ (1) ,(2) và (3) ta sẽ suy ra D là trực tâm tam giác ABC.
Đến đây công việc còn lại đã rất đơn giản........................

ma 29
12-11-2008, 04:29 PM
I.58)Định lí Eyeball

Định lí: Cho hai đường tròn(O) và (O') ngoài nhau.Hai tiếp tuyến kẻ từ O tới (O') cắt (O') tại C,D.Hai tiếp tuyến kẻ từ O' tới (O) cắt (O) tại A,B.Khi đó ta có : AB =CD.


Chứng minh:


Chúng ta kí hiệu các điểm A,B,C,D như hình vẽ.
OM,O'N tương ứng là các tiếp tuyến của (O') và (O).
AD cắt lại (O') ở P.

Ta thấy:

\hat{MPC} =\hat{OMC} (1)
\hat{MCP} = \hat{CMO'} (2)

Cộng theo vế (1) và (2) ta có:

\hat{MPC}+\hat{MCP}=\hat{OMO'}=90^0
Do đó CP là đường kính của (O').
Vì vậy sẽ có: \hat{CDA}=90^0
Hoàn toàn tương tự ta cũng có các đẳng thức góc khác nên sẽ có ABCD là hình chữ nhật => AB=CD.

ma 29
13-11-2008, 02:56 PM
I.59)Bổ đề Haruki

Bổ đề:Cho AB và CD là hai dây cung không cắt nhau của cùng một đường tròn và P là một điểm bất kì trên cung AB không chứa CD của đường tròn ấy.Gọi E và F lần lượt là giao điểm của PC,PD với AB.Thế thì giá trị biểu thức sau là không đổi:

\frac{{AE} . {BF}}{EF}

Chứng minh:
[Only registered and activated users can see links]

(AED) cắt lại AB ở G.
Ta thấy:\hat{AGD}=\hat{CPD} (Không đổi) => G cố định => BG không đổi (1)

Mặt khác :
AF.FB=FP.FD=FE.FG

=> (AE+EF).FB=FE.(FB+BG)

=> AE.FB=FE.BG => \frac{AE.BF}{FE}=BG(2)

Từ (1) và (2 ) suy ra dpcm.

Một file liên quan:

thamtuhoctro
15-11-2008, 09:50 AM
I.60)Bài toán Langley

Bài toán:Cho \bigtriangleup ABC cân tại A có \hat{BAC} = 20^ \circ. Trên cạnh AB, AC lấy điểm D,E sao cho \hat{BCD} = 50^ \circ ,\hat{CBE} = 60^ \circ .Tính \hat{BED}?

Lời giải:

[Only registered and activated users can see links]
Đặt \hat{BED}=x
Trên cạnh AClấy điểmF sao choBF=BC.Khi đó \bigtriangleup BCF cân tại B
\Rightarrow BF=BC
Mặt khác \bigtriangleup BCD cân tại B(do \hat{BCD}=\hat{BDC}=50^ \circ)
\Rightarrow BD=BC
Suy ra BF=BD
Do đó \bigtriangleup BDF đều(\hat{DBF}= 60^ \circ)
\Rightarrow DF=BF
Lại có \bigtriangleup BFE cân tại F
\Rightarrow EF=BF
Ta suy ra EF=DF
\Rightarrow \bigtriangleup DEF cân tại F
\Rightarrow (x+40^ \circ)=\frac{{180^ \circ - 40^ \circ }}{2}
\Rightarrow x=30^ \circ

thamtuhoctro
15-11-2008, 11:22 AM
I.61)Định lí Paul Yiu về đường tròn bàng tiếp
Định lí:Cho \bigtriangleup ABC các đường tròn bàng tiếp góc A,B,C tiếp xúc với 3 cạnh lần lượt tại M,N,P,Q,R,S.Các đường thẳng qua MN,PQ,RS giao nhau tại A_1 ,B_1 ,C_1.Thế thì AA_1,BB_1,CC_1 đồng quy tại trực tâm H của tam giác ABC.

Chứng minh.
[Only registered and activated users can see links]
Gọi I là tâm đường tròn nội tiếp tam giác ABC,D là tiếp điểm của (I) trên BC.
Chúng ta sẽ chứng minh AA_1 vuông góc với BC.
Ta biết rằng:CR=p,CD=p-c nên RD=c.
Từ đó RB=RD-BD=c-(p-b)=p-a.
Hoàn toàn tương tự thì:ND=b và NC=p-a.
Ta thấy:
RI^2-RC^2 +AC^2-AB^2+NB^2-NI^2=(RI^2-NI^2)+b^2-c^2=(DR^2-DN^2)+b^2-c^2=c^2-b^2+b^2-c^2=O
Do đó theo định lí Carnot thì đường thẳng đi qua A vuông góc với BC,đường thẳng đi qua R vuông góc với CI và đường thẳng đi qua N vuông góc với BI đồng quy.
Do đó AA_1 vuông góc với BC ,hay AA_1 đi qua H.
Hoàn toàn tương tự BB_1,CC_1 đi qua H.
Như vậy ta có đpcm:)

trung anh
25-11-2008, 09:59 PM
I.62)Định lí Maxwell

Định lí: Cho \Delta ABC và một điểm P, các cạnh của \Delta A'B'C' song song với các đường thẳng đi qua một đỉnh \Delta ABC và điểm P. Qua A',B',C' kẻ các đường thẳng song song với các cạnh của \Delta ABC. Khi đó ta có các đường thẳng này đồng quy tại một điểm P'.

[Only registered and activated users can see links]

Chứng minh:
Dễ dàng c/m được các góc \widehat{ABP}=\widehat{A'C'P'} và \widehat{CBP}=\widehat{P'A'C'}. Tương tự áp dụng định lí Ceva Sin ta có đpcm.

Trường hợp tương tự cũng xảy ra nếu đổi 'song song' thành 'vuông góc'.
Liên kết ngoài:
[Only registered and activated users can see links]
[Only registered and activated users can see links]
[Only registered and activated users can see links]
Không phiền thì mong anh ma 29 hãy sửa lại cho em bài "Đường tròn Droz_Farny"

ma 29
26-11-2008, 10:57 AM
I.62)Định lí Maxwell

Định lí: Cho \Delta ABC và một điểm P, các cạnh của \Delta A'B'C' song song với các đường thẳng đi qua một đỉnh \Delta ABC và điểm P. Qua A',B',C' kẻ các đường thẳng song song với các cạnh của \Delta ABC. Khi đó ta có các đường thẳng này đồng quy tại một điểm P'.

[Only registered and activated users can see links]

Chứng minh:
Dễ dàng c/m được các góc \widehat{ABP}=\widehat{A'C'P'} và \widehat{CBP}=\widehat{P'A'C'}. Tương tự áp dụng định lí Ceva Sin ta có đpcm.

Trường hợp tương tự cũng xảy ra nếu đổi 'song song' thành 'vuông góc'.
Liên kết ngoài:
[Only registered and activated users can see links]
[Only registered and activated users can see links]
[Only registered and activated users can see links]
Không phiền thì mong anh ma 29 hãy sửa lại cho em bài "Đường tròn Droz_Farny"

Há,hôm nay onl vui quá,thấy chú vào đây giúp anh :) :hornytoro:

Còn cái Droz hôm trước anh có đọc qua và thấy đoạn chứng minh 6 điểm đồng viên còn bị thiếu xót ,anh đã bỏ nó đi.Chúng ta sẽ sử dụng định nghĩa để cm là nhanh nhất:)
Chắc là vẫn còn thắc mắc? Nếu vậy thì cứ nói nhé:)

ma 29
20-12-2008, 07:14 AM
Ơ sao ko làm tiếp đi Khánh, từ ngày lên Smod thấy em lười đi hẳn :)

Dạo này đang thi học kì mà anh ,em ôn tí lí hóa đó mà:))
Em sẽ sớm làm tiếp thôi ạ:)

ma 29
25-12-2008, 03:21 PM
I.63)Định lí Brahmagupta về tứ giác nội tiếp có hai đường chéo vuông góc.


Định lí:Cho tứ giác nội tiếp ABCD có AC vuông góc với BD tại S. Khi đó đoạn nối trung điểm một cạnh với S sẽ vuông góc với cạnh đối diện.
Chứng minh:
[Only registered and activated users can see links]

Ta chứng minh đại diện,chẳng hạn gọi M là trung điểm BC ta cần chứng minh MS vuông góc với AD.
Thật vậy,MS cắt AD ở H.

Ta có:\hat{BSC} =90^0 ,M là trung điểm BC nên MS=MC.
Do đó:
(DA,DB) \equiv (CA,CB) \equiv (SC,SM) \equiv (SA,SH) (mod \pi)
Dễ =>dpcm:)

Quân -k47DHV
25-12-2008, 08:50 PM
Định lý Feuer bach-luchterhand:
tứ giác ABCD nội tiếp ,O là điểm bất kì trong mặt phẳng thì:
OA^2. BC. CD. BD - OB^2. CD. DA. AB+ OC^2. DA. AB. BD- CD^2. AB. BC. CD =0

ma 29
26-12-2008, 03:15 PM
I.64)Định lí Schooten

Định lí: Cho tam giác đều ABC nhận (O) là đường tròn ngoại tiếp.Khi đó với mọi điểm S nằm trên (O) thì một trong 3 đoạn SA,SB,SC có một đoạn có độ dài bằng tổng độ dài hai đoạn còn lại.


Chứng minh:

[Only registered and activated users can see links]

Không giảm tổng quát ,giả sử S thuộc cung BC nhỏ.
Ta sẽ chứng minh SA=SB+SC.Thật vậy:
Gọi I là một điểm trên đoạn SA sao cho SI=SC.
Đến đây công việc của chúng ta là chứng minh AI=SB (*) mà điều này thì khá đơn giản ,chỉ cần để ý một chút:
Dễ thấy tam giác SIC đều nên CI=CS (1)
Mà:CA=CB (2) và \hat{ACI}=\hat{ACS} -\hat{ICS}=\hat{ACS}-60^0=\hat{ACS}-\hat{ACB}=\hat{BCS} (3)
Từ (1),(2) và (3) ta dễ suy ra (*) :)

ma 29
26-12-2008, 03:31 PM
I.65)Định lí Bottema


Định lí:Về phía ngoài tam giác ABC ta dựng hai hình vuông ABDE, ACFG .Gọi M là trung điểm DF.Thế thì Vị trí điểm M không phụ thuộc vào vị trí điểm A và tam giác MBC vuông cân tại M.


Chứng minh:
[Only registered and activated users can see links]

Chúng ta sẽ chứng minh ý hai bởi từ điều đó cũng suy ra ngay ý một.
Vậy công việc của chúng ta là chứng minh tam giác MBC vuông cân tại M.
Bài này có nhiều cách giải,ở đây ma 29 xin trình bày bằng pp phép biến hình:)

Kí hiệu:R_S^a là chỉ phép quay tâm S góc quay a,giả sử chiều dương là ngược chiều kim đồng hồ.
Ta thấy:
R_C^{90^0} : F \to A; R_B^{90^0}: A \to D
=> F=R_C^{90^0} o R_B^{90^0} : F \to D
Mà F là phép quay với góc quay 90^0 +90^0 =180^0 nên tâm phép quay này chính là M. Dựa vào cách xác định tâm của tích hai phép quay ta dễ có dpcm :)

ma 29
27-12-2008, 09:11 AM
I.66)Định lí Pompeiu

Định lí;Cho tam giác ABC đều ,và một điểm D trên mặt phẳng tam giác.Khi đó luôn tồn tại một tam giác với độ dài các cạnh là DA,DB,DC.

Chứng minh:
[Only registered and activated users can see links]

Chúng ta sẽ dùng bất đẳng thức Ptolemy (Xem mục I.10)để giải quyết bài này một cách cực kì nhanh gọn!:)
Bây giờ,theo nguyên lí khởi đầu cực trị trong ba đoạn DA,DB,DC sẽ có một đoạn có độ dài lớn nhất.
Không giảm tổng quát ,giả sử đó là DA.
Đến đây ta chỉ cần chứng minh DB +DC\geq {DA} ($)là xong.
Kí hiệu a là độ dài cạnh tam giác đều ABC
Sử dụng bất đẳng thức Ptolemy ta có:

DB.AC + DC.AB \geq DA.BC => a.DB + a.DC \geq a.DA
Đến đây thì thấy tiền ($)rồi :))

Lời bình:Từ lời giải trên và điều kiện xảy ra dấu bằng của BDT Ptolemy ta thấy định lí Schooten (Xem mục I.64)chỉ là hệ quả đơn giản của bài này,và chiều ngược lại của nó vẫn đúng.!:)

thamtuhoctro
02-01-2009, 11:25 AM
I.67)Định lí Zaslavsky
Định lí:Cho tam giác ABC và điểm O.Tam giác A_1B_1C_1 là ảnh của tam giác ABC qua phép đối xứng tâm O.Từ A_1,B_1,C_1 kẻ các đường thẳng song song với nhau cắt BC,CA,AB tại N, P, M.Chứng minh rằng M, N, P thẳng hàng.

[Only registered and activated users can see links]

Chứng minh:

Từ B kẻ đường thẳng song song với C_1M cắt AC_1, CA_1, A_1C_1 tại H,K,P_1.
Vì (BK),(A_1C_1) là ảnh của (B_1P), (AC) qua phép đối xứng tâm O
nên P_1 là ảnh của P qua phép đối xứng tâm O
suy ra \vec{P_1C_1} = -\vec{PC} , \vec{P_1A_1} = -\vec{PA}
Ta có:
\frac{MB}{MA}.\frac{PA}{PC}.\frac{NC}{NB}
= \frac{C_1H}{C_1A}.\frac{PA}{PC}.\frac{A_1C}{A_1K}
=\frac{C_1H}{A_1K}.\frac{PA}{PC}
=\frac{P_1C_1}{P_1A_1}.\frac{PA}{PC}=1
Ta suy ra dpcm :D
==============

thamtuhoctro
02-01-2009, 04:14 PM
I.68)Định lí Archimedes
Định lí:Cho M là trung điểm \stackrel{\frown}{AB}, điểm C chuyển động tùy ý trên \stackrel{\frown}{AMB}.Từ M kẻ MD \perp AC.Chứng minh rằng AD=AC+CB

[Only registered and activated users can see links]

Chứng minh:
Trên tia AC dựng điểm E sao cho CB=CE.
Ta có: \hat{AMB}=2\hat{AEB}
đồng thời suy ra 2\hat{MAB}=\hat{BCE}
Vậy MC đồng thời là đường cao, đường trung tuyến tam giác BME
nên tam giác MBE cân tại M suy ra ME=MB=MA
do đó D là trung điểm cạnh AE
hay nói cách khác AD=CD+CB(dpcm) :D
==============

thamtuhoctro
04-01-2009, 10:03 AM
I.69) Định lí Urquhart
Định lí:Cho hai bộ ba điểm thẳng hàng ABB_1 và AC_1C, D là giao điểm của BC và B_1C_1.Chứng minh rằng AB + BD = AC_1 + C_1D khi và chỉ khi AB_1 + B_1D = AC + CD.

[Only registered and activated users can see links]

Chứng minh:
Đầu tiên ta cần chứng minh bổ đề sau:Trong tam giác ABC ta có \frac{1}{p} = \frac{1}{a}.(1-tan\frac{B}{2}.tan\frac{C}{2}) với p là nửa chu vi và a=BC.
Ta có: a + b + c = a + \frac{h_a}{sinC} + \frac{h_a}{sinB}
mặt khác a=h_a.(\frac{1}{tanC} + \frac{1}{tanB})
=h_a.(\frac{sin(B+C)}{sinB.sinC}.
nên suy ra a + b + c = a(1 + \frac{sinB + sinC}{sin(B+C)})
= 2a(\frac{cos(\frac{B}{2}).cos(\frac{C}{2})}{\cos (\frac{B}{2}).cos(\frac{C}{2}) - sin(\frac{B}{2}).sin(\frac{C}{2})})
do cos(\frac{B+C}{2}) \neq 0 nên nghịch đảo hai vế ta được dpcm

Trở lại bài toán gọi các góc như trên hình vẽ ta có:
AB + BD = AC_1 + C_1D
\Leftrightarrow {p}_{ABD}={p}_{AC_1D}
\Leftrightarrow \tan \frac{\alpha _1}{2}. \tan \frac{\beta _1}{2} = \tan \frac{\alpha_2 }{2}.\tan \frac{\beta_2 }{2}
\Leftrightarrow \tan \frac{\beta _1}{2}.\tan \frac{ADB_1}{2}=\tan \frac{\beta _2}{2}. \tan \frac{ADC}{2}
\Leftrightarrow {p}_{ADB_1}={p}_{ADC}
\Leftrightarrow AB_1 + B_1D = AC + CD(dpcm) :D

thamtuhoctro
07-01-2009, 11:01 AM
I.70)Định lí Mairon Walters
Định lí:Cho tam giác ABC và các đường thẳng chia 3 cạnh đối diện như hình vẽ.Chứng minh rằng {S}_{MNPQRS}=\frac{1}{10}{S}_{ABC}

[Only registered and activated users can see links]

Chứng minh::
Trước tiên ta cần chứng minh bổ đề sau:Cho tam giác ABC điểm N, M di động trên đường thẳng AB, AC sao cho \vec{MA}=m\vec{MC} , \vec{NA}=n\vec{NC}.Giả sử CN, BM giao nhau tại P thì P chia đoạn CN theo tỉ số \frac{m}{1-n}.
Giả sử \vec{PN}=k\vec{PC} thì ta có:
\vec{BP}=\frac{\vec{BN} - k\vec{BC}}{1-k}
= \frac{\vec{BA}}{(1-n)(1-k)} - \frac{k\vec{BC}}{1-k}
mà \vec{BM}=\frac{\vec{BA}}{1-m} - \frac{mBC}{1-m}
lại do \vec{BM} cùng phương với \vec{BP} nên tồn tại một số t sao cho \vec{BM}=t\vec{BP}
Mặt khác cách biểu diễn này là duy nhất nên ta có đồng nhất thức:
\frac{t}{(1-n)(1-k)}=\frac{1}{1-m}
\frac{tk}{1-k}=\frac{m}{1-m}
suy ra:
k=\frac{m}{1-n}.
Quay lại bài toán ban đầu ta áp dụng bổ đề nhiều lần liên tiếp ta được(đây chỉ là kĩ năng tính toán nên mình chỉ ghi kết quả các bác thông cảm)
{S}_{BDQFC}=\frac{7}{15}{S}_{ABC}
{S}_{APF}=\frac{1}{6}{S}_{ABC}
{S}_{AEN}=\frac{2}{15}{S}_{ABC}
{S}_{DHME}=\frac{5}{42}{S}_{ABC}
{S}_{SHR}=\frac{1}{70}{S}_{ABC}
suy ra:
{S}_{MNPQRS}=\frac{1}{10}{S}_{ABC}(dpcm) :D

thamtuhoctro
08-01-2009, 08:56 AM
I.71)Định lí Poncelet về bán kính đường tròn nội tiếp,bàng tiếp trong tam giác vuông.
Định lí:Cho tam giác ABC có r, r_a, r_b, r_c lần lượt là bán kính các đường tròn nội tiếp, bàng tiếp góc A, B, C.Chứng minh rằng: tam giác ABC vuông tại A khi và chỉ khi r_a = r + r_b + r_c .

[Only registered and activated users can see links]

Chứng minh::
Ta có:
{r}_{a} = r + {r}_{b} + {r}_{c}
\Leftrightarrow \frac{S}{p-a} = \frac{S}{p} + \frac{S}{p-b} + \frac{S}{p-c}
\Leftrightarrow \frac{1}{p-a} - \frac{1}{p} = \frac{1}{p-b} + \frac{1}{p-c}
\Leftrightarrow \frac{c}{p(p-a)} = \frac{c}{(p-b)(p-c)}
\Leftrightarrow {a}^{2} = {b}^{2} + {c}^{2}
\Leftrightarrow tam giác ABC vuông tại A(dpcm) :D

hophinhan_LHP
08-01-2009, 10:44 AM
(Định lí Mairon Walters)

Tồn tại phép biến hình tuyến tính biến tam giác ABC thành tam giác đều ...các tỉ số trên cạnh và diện tính không đồi => dpcm

@: tính chất khác là : MQ , NR và SP đồng quy :D

thamtuhoctro
08-01-2009, 07:30 PM
I.72)Ðịnh lí Hansen
Ðịnh lí:Cho tam giác ABC.Chứng minh rằng các điều kiện sau tương đương:
1)Tam giác ABC vuông
2) r + r_a + r_b + r_c = a + b + c.
3) {r}^{2} + {{r}_{a}}^{2} + {{r}_{b}}^{2} + {{r}_{c}}^{2} = {a}^{2} + {b}^{2} + {c}^{2}

[Only registered and activated users can see links]

Chứng minh:
Ðầu tiên ta chứng minh một vài hệ thức phụ sau:
1) r = 4R\sin(\frac{A}{2})\sin(\frac{B}{2})\sin(\frac{C}{ 2})
2){r}_{a} = 4R\sin(\frac{A}{2})\cos(\frac{B}{2})\cos(\frac{C}{ 2})
3)\sum {(\sin A)}^{2} = 2 + 2\prod \cos A
Ta có:
a = r(\cot \frac{B}{2} + \cot \frac{C}{2})
\Leftrightarrow 2R\sin A = r(\cot \frac{B}{2} + \cot \frac{C}{2})
\Leftrightarrow 4R.\sin \frac{A}{2}.\cos\frac{A}{2} = r(\frac{\cos \frac{A}{2}}{\sin \frac{B}{2}.\sin \frac{C}{2}})
\Leftrightarrow 4R\prod \sin \frac{A}{2} = r

\frac{r}{{r}_{a}} = \frac{p-a}{p} = \frac{\sin B + \sin C - \sin A}{\sin B + \sin C + \sin A} = \frac{\cos \frac{A}{2}.\sin \frac{B}{2}. \sin \frac{C}{2}}{\cos \frac{A}{2}.\cos \frac{B}{2}.\cos \frac{C}{2}}
\Rightarrow {r}_{a} = 4R\sin \frac{A}{2}.\cos \frac{B}{2}.\cos \frac{C}{2}

{(\vec{AB} + \vec{BC} + \vec{CA})}^{2} = 0
\Leftrightarrow \sum {a}^{2} - 2(\sum ab\cos C) = 0
\Leftrightarrow {(2R)}^{2}(\sum {(\sin A)}^{2}) - 2{(2R)}^{2}(\sum \sin A.\sin B.\cos C)
\Leftrightarrow \sum {(\sin A)}^{2} = 2({(\sin A)}^{2} + \cos A.\sin B.\sin C)
\Leftrightarrow \sum {(\sin A)}^{2} = 2({(\sin A)}^{2} + \frac{1}{2}(\cos (B - c) - \cos (B + C)))
\Leftrightarrow \sum {(\sin A)}^{2} = 2(1 + \frac{1}{2}.\cos (B - C).\cos A - \frac{1}{2}{(\cos A)}^{2}
\Leftrightarrow \sum {(\sin A)}^{2} = 2(1 - \frac{1}{4}(\cos 2B + \cos 2C) - \frac{1}{2}{(\cos A)}^{2}
\Leftrightarrow \sum {(\sin A)}^{2} = 2(1 + \frac{1}{2}{(\cos A)}^{2}(\cos (B - C) + \cos (B + C))
\Leftrightarrow \sum {(\sin A)}^{2} = 2(1 + \prod \cos A)

Trở lại bài toán ban đầu ta có:
r + r_a + r_b + r_c = a + b + c
\Leftrightarrow 4R(\prod \sin \frac{A}{2} + \sum \sin \frac{A}{2}.\cos \frac{B}{2}.\cos \frac{C}{2}) = 2R(\sum \sin A)
\Leftrightarrow \prod \sin \frac{A}{2} + \sum \sin \frac{A}{2}.\cos \frac{B}{2}.\cos \frac{C}{2} = 2\prod \cos \frac{A}{2}
Do \prod \cos \frac{A}{2} \neq 0 nên chia cả hai vế cho \prod \cos \frac{A}{2} ta được"
\Leftrightarrow \prod \tan \frac{A}{2} + \sum \tan \frac{A}{2} = 2 (1)
Ðặt \tan \frac{A}{2} = x, \tan \frac{B}{2} = y. \tan \frac{C}{2} = z ta suy ra xy + yz + zx = 1 do dó
(1) \Leftrightarrow xyz + x + y + z = 1 + xy + yz + zx
\Leftrightarrow (1-x)(1-y)(1-z)=0
\Leftrightarrow tam giác ABC vuông.

{r}^{2} + {{r}_{a}}^{2} + {{r}_{b}}^{2} + {{r}_{c}}^{2} = {a}^{2} + {b}^{2} + {c}^{2}
\Leftrightarrow {(4R)}^{2}(\prod {(\sin \frac{A}{2})}^{2} + \sum {(\ \sin \frac{A}{2}.\cos \frac{B}{2}.\cos \frac{C}{2})}^{2}) = {(2R)}^{2}(\sum {(\sin A)}^{2})
\Leftrightarrow 4(\prod {(\sin \frac{A}{2})}^{2} + \sum {(\sin \frac{A}{2}.\cos \frac{B}{2}.\cos \frac{C}{2})}^{2}) = 2(1 + \prod \cos A)
\Leftrightarrow 4{\prod (\cos \frac{A}{2})}^{2}({\prod (\tan \frac{A}{2})}^{2} + \sum {(\tan \frac{A}{2})}^{2}) = 2(1 + \prod \cos A) (2)
tương tự thay x, y, z như trên với chú ý \cos \frac{A}{2} = \frac{1}{1 + {x}^{2}} và \cos A = \frac{1 - {x}^{2}}{1 + {x}^{2}} ta được:
(2) \Leftrightarrow \frac{2({(xyz)}^{2} + \sum {x}^{2})}{\prod (1 + {x}^{2})} = 1 + \prod \frac{(1 - {x}^{2})}{(1 + {x}^{2})}
\Leftrightarrow 2({(xyz)}^{2} + \sum {x}^{2}) = \prod (1 + {x}^{2}) + \prod (1 - {x}^{2})
\Leftrightarrow {(xyz)}^{2} + \sum {x}^{2} = 1 + \sum {(xy)}^{2}
\Leftrightarrow (1 - {x}^{2})(1 - {y}^{2})(1 - {z}^{2}) = 0 (3)
do \tan \frac{A}{2}, \tan \frac{B}{2}, \tan \frac{C}{2} lớn hơn 0 nên
(3) \Leftrightarrow tam giác ABC vuông. :))
*chú thích:
\prod \sin \frac{A}{2} = \sin \frac{A}{2}.\sin \frac{B}{2}.\sin \frac{C}{2}
\sum \sin \frac{A}{2} = \sin \frac{A}{2} + \sin \frac{B}{2} + \sin \frac{C}{2}

ma 29
15-01-2009, 08:41 PM
anh ma29 có thể tổng hợp thành 1 file pdf đựoc ko ạ? cảm ơn anh

Cái này chắc là sẽ làm thôi,mỗi tội phải xong hết đã:))
Cám ơn em đã quan tâm:)

thamtuhoctro
19-01-2009, 03:17 PM
I.73)Định lí Steinbart mở rộng
Định lí:.Cho tam giác ABC nội tiếp (O).Các tiếp tuyến của đường tròn tại A, B, C giao nhau tại A_1, B_1,C_1.Trên (O) lấy các điểm A_2, B_2, C_2 .Chứng minh rằng A_1A_2, B_1B_2, C_1C_2 đồng quy khi và chỉ khi AA_2, BB_2, CC_2 đồng quy hoặc các giao điểm của AA_2, BB_2, CC_2 với 3 cạnh tam giác thẳng hàng.

[Only registered and activated users can see links]

[Only registered and activated users can see links]

Chứng minh:
Gọi M = A{A}_{2} \bigcap BC
N = B{B}_{2} \bigcap AC
P = C{C}_{2} \bigcap AB
Ta có:
\frac{{S}_{[{A}_{1}B{A}_{2}]}}{{S}_{[{A}_{1}C{A}_{2}]}} = \frac{B{A}_{2} . \sin (\vec{B{A}_{2}} ; \vec{B{A}_{1}}))}{C{A}_{2} . \sin (\vec{C{A}_{2}} ; \vec{C{A}_{1}})} = \frac{\sin (\vec{{A}_{1}B} ; \vec{{A}_{1}{A}_{2}})}{\sin (\vec{{A}_{1}C} ; \vec{{A}_{1}{A}_{2}})}
mà - \frac{B{A}_{2}}{C{A}_{2}} = \frac{\sin (\vec{B{A}_{2}} ; \vec{B{A}_{1}})}{\sin (\vec{C{A}_{2}} ; \vec{C{A}_{1}})} = \frac{\sin (\vec{A{A}_{2}} ; \vec{AB})}{\sin (\vec{A{A}_{2}} ; \vec{AC})}
nên - {(\frac{\sin (\vec{A{A}_{2}} ; \vec{AB})}{\sin (\vec{A{A}_{2}} ; \vec{AC})})}^{2} = \frac{\sin (\vec{{A}_{1}B} ; \vec{{A}_{1}{A}_{2}})}{\sin (\vec{{A}_{1}C} ; \vec{{A}_{1}{A}_{2}})}
Tương tự ta suy ra:
{(\frac{\sin (\vec{A{A}_{2}} ; \vec{AB})}{\sin (\vec{A{A}_{2}} ; \vec{AC})} . \frac{\sin (\vec{B{B}_{2}} ; \vec{BC})}{\sin (\vec{B{B}_{2}} ; \vec{BA})} . \frac{\sin (\vec{C{C}_{2}} ; \vec{CA})}{\sin (\vec{C{C}_{2}} ; \vec{CB})})}^{2} = - \frac{\sin (\vec{{A}_{1}B} ; \vec{{A}_{1}{A}_{2}})}{\sin (\vec{{A}_{1}C} ; \vec{{A}_{1}{A}_{2}})} . \frac{\sin (\vec{{C}_{1}A} ; \vec{{C}_{1}{C}_{2}})}{\sin (\vec{{C}_{1}B} ; \vec{{C}_{1}{C}_{2}})} . \frac{\sin (\vec{{B}_{1}C} ; \vec{{B}_{1}{B}_{2}})}{\sin (\vec{{B}_{1}A} ; \vec{{B}_{1}{B}_{2}})}
do đó nếu vế phải bằng -1 thì biểu thức trong ngoặc ở vế trái bằng 1 hoặc -1 và ngược lại hay nói cách khác {A}_{1}{A}_{2}, {B}_{1}{B}_{2}, {C}_{1}{C}_{2} đồng quy khi và chỉ khi A{A}_{2}, B{B}_{2}, C{C}_{2} đồng quy hoặc M, N, P thẳng hàng :D

thamtuhoctro
19-01-2009, 04:24 PM
I.74)Định lí Monge & d'Alembert I
Định lí:Cho 3 đường tròn (A,{R}_{1}), (B,{R}_{2}), (C,{R}_{3}) có bán kính khác nhau và không chứa nhau.Tiếp tuyến chung ngoài của mỗi đường tròn giao nhau lần lượt tại M, N, P.Chứng minh rằng:M, N, P thẳng hàng.

[Only registered and activated users can see links]

Chứng minh:
Vì các đường tròn có vai trò như nhau nên không mất tính tổng quát ta giả sử:
{R}_{1} > {R}_{2} > {R}_{3}.Khi đó ta có thể chứng minh được:
V(P, \frac{{R}_{1}}{{R}_{2}}) : (B) \rightarrow (A)
V(N, \frac{{R}_{3}}{{R}_{1}}) : (A) \rightarrow (C)
V(N, \frac{{R}_{2}}{{R}_{3}}) : (C) \rightarrow (B)
Suy ra:
\frac{\bar{PA}}{\bar{PB}}.\frac{\bar{MB}}{\bar{MC} }.\frac{\bar{NC}}{\bar{NA}} = \frac{{R}_{1}}{{R}_{2}}. \frac{{R}_{2}}{{R}_{3}}. \frac{{R}_{3}}{{R}_{1}} = 1
Theo định lí Menelaus ta suy ra dpcm :D
*Chú thích:V(P, \frac{{R}_{1}}{{R}_{2}}) : (B) \rightarrow (A)
là phép vị tự tâm P tỉ số \frac{{R}_{1}}{{R}_{2}} biến (B) thành (A)

thamtuhoctro
19-01-2009, 07:19 PM
I.75)Định lí Monge & d'Alembert II
Định lí:Cho 3 đường tròn (A,{R}_{1}), (B,{R}_{2}), (C,{R}_{3}) có bán kính khác nhau và không chứa nhau.Tiếp tuyến chung trong của (A) và (C), (B) và (C) giao nhau lần lượt tại N, M, tiếp tuyến chung ngoài của (A) và (B) giao nhau tại M.Chứng minh rằng:M, N, P thẳng hàng.

[Only registered and activated users can see links]

Chứng minh:
Vì các đường tròn có vai trò như nhau nên không mất tính tổng quát ta giả sử:
{R}_{1} > {R}_{2} > {R}_{3}.Khi đó ta có thể chứng minh được:
V(P, \frac{{R}_{1}}{{R}_{2}}) : (B) \rightarrow (A)
V(N, - \frac{{R}_{3}}{{R}_{1}}) : (A) \rightarrow (C)
V(N, - \frac{{R}_{2}}{{R}_{3}}) : (C) \rightarrow (B)
Suy ra:
\frac{\bar{PA}}{\bar{PB}}.\frac{\bar{MB}}{\bar{MC} }.\frac{\bar{NC}}{\bar{NA}} = \frac{{R}_{1}}{{R}_{2}}. (- \frac{{R}_{2}}{{R}_{3}}). (- \frac{{R}_{3}}{{R}_{1}}) = 1
Theo định lí Menelaus ta suy ra dpcm :D
*Chú thích:V(P, \frac{{R}_{1}}{{R}_{2}}) : (B) \rightarrow (A)
là phép vị tự tâm P tỉ số \frac{{R}_{1}}{{R}_{2}} biến (B) thành (A)

thamtuhoctro
19-01-2009, 08:31 PM
I.76)Định lí Steiner về bán kính các đường tròn
Định lí:Chứng minh rằng trong tam giác ta có: {r}_{a} + {r}_{b} + {r}_{c} = 4R + r

[Only registered and activated users can see links]

Chứng minh:
Ta có:
{r}_{a} + {r}_{b} + {r}_{c} = 4R + r
\Leftrightarrow \frac{S}{(p - a)} + \frac{S}{(p - b)} + \frac{S}{(p - c)} = \frac{abc}{S} + \frac{S}{p}
\Leftrightarrow S(\frac{1}{(p - a)} + \frac{1}{(p - b)} + \frac{1}{(p - c)} - \frac{1}{p} = \frac{abc}{S}
\Leftrightarrow p(\sum (p - b)(p - c)) - (p - a)(p - b)(p - c) = abc
\Leftrightarrow p(3{p}^{2} - 2p(a + b + c) +\sum ab) - {p}^{3} + {p}^{2}\sum a - p\sum ab + abc = abc
\Leftrightarrow -2{p}^{3} + {p}^{2}\sum a + p\sum ab - p\sum ab = 0(hiển nhiên) suy ra dpcm :))

thamtuhoctro
20-01-2009, 09:52 AM
==============
I.77)Định lí Bellavitis
Định lí::Cho tứ giác ABCD là tứ giác điều hoà kí hiệu \hat{DAC} = \alpha, \hat{ABD} = \beta, \hat{ACB} = \gamma, \hat{CDB} = \delta.Chứng minh rằng: \alpha + \beta + \gamma + \delta = {180}^{o}

[Only registered and activated users can see links]

Chứng minh:
Gọi đường tròn đường kính MN là đường tròn Apollonius của tam giác ABC ứng với đỉnh B.
Vì ABCD là tứ giác điều hoà nên AB.CD = AD.BC do đó D thuộc đường tròn Apollonius của tam giác ABC.
Dựng BP đối xứng với BD qua phân giác BM.
Ta suy ra:
\hat{ADP} = \delta
\hat{DPC} = \alpha + \delta = \hat{BPC}( = \frac{1}{2} sd \stackrel{\frown}{BD})
\hat{PBC} = \beta
Vậy \alpha + \beta + \gamma + \delta = {180}^{o}(dpcm) :D

ma 29
20-01-2009, 10:12 AM
I.78)Định lí Feuer bach-Luchterhand:

thamtuhoctro
20-01-2009, 10:29 AM
II/Một số điểm và đường đặc biệt được xác định duy nhất với tam giác và tứ giác
II.1) Đường thẳng Euler của tam giác.
Định lí:Cho tam giác ABC gọi H, G, O là trực tâm, trọng tâm, tâm đường tròn ngoại tiếp.Chứng minh rằng: H, G, O thẳng hàng.

[Only registered and activated users can see links]

Chứng minh:
Gọi {A}_{1}, {B}_{1}, {C}_{1} là trung điểm BC, AC, AB.
Ta có
{V}_{(G, - \frac{1}{2})} : ABC \rightarrow {A}_{1}{B}_{1}{C}_{1}
H \rightarrow O
do đó \vec {GO} = - \frac{1}{2} \vec {GH}
suy ra H, G, O thẳng hàng :))

ma 29
20-01-2009, 10:31 AM
II)Đường tròn và tâm Euler

Kết quả : Trong một tam giác ,trung điểm các cạnh của tam giác ,chân các đường cao và trung điểm các đoạn thẳng nối trực tâm với các đỉnh cùng nằm trên một đường tròn gọi là đường tròn Euler của tam giác ấy.

Chỉ dẫn chứng minh:
[Only registered and activated users can see links]
Thực ra đây chỉ là một trường hợp đặc biệt của hai điểm đẳng giác (Xem mục I.50)

thamtuhoctro
20-01-2009, 11:08 AM
II.3)Đường đối trung, điểm Lemoine
Kết quảCho tam giác ABC thì 3 đường đối trung của tam giác đồng quy tại điểm Lemoine của tam giác.

[Only registered and activated users can see links]

Chỉ dẫn chứng minh::
Đường đối trung của tam giác ứng với 1 đỉnh là đường thẳng đối xứng với trung tuyến qua phân giác tương ứng của đỉnh đó.
Từ định nghĩa trên áp dụng định lí Xeva dạng sin ta có:
\frac{\sin (\vec{AB} ; \vec{A{A}_{2}})}{\sin (\vec{AC} ; \vec{A{A}_{2}})} . \frac{\sin (\vec{BA} ; \vec{B{B}_{2}})}{\sin (\vec{BC} ; \vec{B{B}_{2}})} . \frac{\sin (\vec{CA} ; \vec{C{C}_{2}})}{\sin (\vec{CB} ; \vec{C{C}_{2}})} = \frac{\sin (\vec{A{A}_{1}} ; \vec{AC})}{\sin (\vec{A{A}_{1}} ; \vec{AB})} . \frac{\sin (\vec{B{B}_{1}} ; \vec{BC})}{\sin (\vec{B{B}_{1}} ; \vec{BA})} . \frac{\sin (\vec{C{C}_{1}} ; \vec{CB})}{\sin (\vec{C{C}_{1}} ; \vec{CA})} = -1
Suy ra A{A}_{2}, B{B}_{2}, C{C}_{2} đồng quy tại điểm Lemoine của tam giác.
*Chú thích: kí hiệu L[A({a}^{2}) ; B({b}^{2}) ; C({c}^{2})] tương đương với
{a}^{2} \vec{LA} + {b}^{2} \vec{LB} + {c}^{2} \vec{LC} = \vec{o} :))

ma 29
20-01-2009, 11:40 AM
II.4)Điểm Gergonne,điểm Nobb, đường thẳng Gergone

1)Kết quả về điểm Gergonne:Tam giác ABC với đường tròn nội tiếp (I).Tiếp điểm của (I) trên BC,CA,AB lần lượt là D,E,F.Khi đó AD,BE,CF đồng quy tại một điểm gọi là điểm Gergonne của tam giác ABC.
[Only registered and activated users can see links]

Chỉ dẫn chứng minh:
Chỉ cần dùng định lí Ceva và các kết quả đơn giản : DB=DC,EA=EC,FA=FB là ra.:D

2)Kết quả về điểm Nobb và đường thẳng Gergonne(Vẫn với các kí hiệu trên)Một tam giác không cân có 3 điểm Nobb tương ứng là giao điểm của các cặp đường thẳng EF và CB ,DE và AB ,DF và AC. Và 3 điểm Nobb cùng nằm trên một đường thẳng gọi là đường thẳng Gergonne của tam giác ABC.

Chỉ dẫn chứng minh:

Xét cực và đối cực đối với (I).
Đường đối cực của A là EF đi qua M,nên đường đối cực của M đi qua A.
Mặt khác dễ thấy đường đối cực của M đi qua D nên suy ra đường đối cực của M là AD.
Hoàn toàn tương tự ta có:
Đường đối cực của N là BE và đường đối cực của P là CF

Theo trên ,do AD,BE,CF đồng quy nên sẽ có điều phải chứng minh.
Bình luận: Kết quả trên có thể mở rộng như sau:

Cho tam giác ABC và 3 điểm D,E,F theo thứ tự thuộc BC,CA,AB sao cho
AD,BE,CF đồng quy và D,E,F khác trung điểm đoạn thẳng.Gọi M,N,P lần lượt là điểm chung của các cặp đường thẳng (EF,BC) ,(DF,CA) ,(DE,AB).Khi đó M,N,P thẳng hàng

Bạn có thể chứng minh kết quả trên bằng định lí Menelaus nhưng thậm chí bài toán mở rộng này
cũng chỉ là trường hợp đặc biệt của định lí Desargues mà thôi!!!!

**Xem thêm:

thamtuhoctro
20-01-2009, 02:38 PM
II.5)Điểm Nagel
Kết quả:.Cho tam giác ABC. Các đường tròn bàng tiếp xúc với 3 cạnh tương ứng đỉnh lần lượt tại D, E, F thì ta có 3 đường thẳng AD, BE, CF đồng quy tại điểm Nagel của tam giác.

[Only registered and activated users can see links]

Chỉ dẫn chứng minh:
Ta có:
\frac{\bar{DB}}{\bar{DC}} . \frac{\bar{EC}}{\bar{EA}} . \frac{\bar{FA}}{\bar{FB}} = - \frac{(p - c)}{(p - b)} . \frac{(p - a)}{(p - c)} . \frac{(p - b)}{(p - a)} = -1
Suy ra dpcm :D
*Chú thích:
N [A(p - a) ; B(p - b) ; C(p - c)]

ma 29
20-01-2009, 03:17 PM
II.6) Điểm Brocard

Định nghĩa:Trong một tam giác ABC cho trước có hai điểm Brocard M,N được xác định sao cho:

\hat{MAB} =\hat{MBC} =\hat{MCA} và \hat {NAC}=\hat{NCB} =\hat{NBA}.

[Only registered and activated users can see links]

thamtuhoctro
21-01-2009, 05:10 PM
II.7)Điểm Schiffler
Định nghĩa:Cho tam giác ABC có I là tâm đường tròn nội tiếp tam giác.Khi đó 4 đường thẳng Euler của tam giác IBC, IAC, IAB và ABC đồng quy tại điểm Schiffler của tam giác.

[Only registered and activated users can see links]

Chỉ dẫn chứng minh:
Gọi O là tâm đường tròn ngoại tiếp tam giác, G là trọng tâm tam giác, M là trung điểm BC, {G}_{1} là trọng tâm tam giác IBC, AI cắt BC tại D cắt (O) tại J, (I) tiếp xúc với cạnh BC tại K, J{G}_{1} cắt OG tại S, cắt AM tại E.
Rõ ràng J là tâm đường tròn ngoại tiếp tam giác IBC.Do đó J{G}_{1} là đường thằng Euler của tam giác IBC.
Áp dụng định lí Menelaus cho tam giác GOM với cát tuyến SEJ ta có:
\frac{SG}{SO} . \frac{JO}{JM} . \frac{EM}{EG} = 1.
Suy ra:
\frac{SG}{SO} = \frac{JM}{JO} . \frac{EG}{EM} = \frac{JM}{R} . \frac{EG}{EM}
Ap dụng định lí Menelaus cho tam giác IAM với cát tuyến J{G}_{1}E ta có:
\frac{JI}{JA} . \frac{EA}{EM} . \frac{{G}_{1}M}{{G}_{1}I} = 1
Do {G}_{1} là trọng tâm tam giác IBC nên
\frac{EA}{EM} = 2 . \frac{JA}{JI} = 2 . \frac{JA}{JB} = 2 . \frac{JB}{JD} = 2 . \frac{JI}{JD} (do {JI}^{2} = {JB}^{2} = JA . JD)
Do đó:
\frac{EG}{EM} = \frac{GM}{EM} - 1 = \frac{1}{3} . \frac{AM}{EM} - 1 = \frac{1}{3}(\frac{EA}{EM} + 1) - 1 = \frac{1}{3} . \frac{EA}{EM} - \frac{2}{3} = \frac{2}{3}(\frac{JI}{JD} - 1) = \frac{2}{3} . \frac{ID}{JD} = \frac{2}{3} . \frac{IK}{JM} = \frac{2}{3} . \frac{r}{JM}

\frac{SG}{SO} = \frac{JM}{R} . \frac{EG}{EM} = \frac{JM}{R} . \frac{2}{3} . \frac{r}{JM} = \frac{2}{3} . \frac{r}{R} (1)
Tương tự ta thấy các đường thẳng Euler của các tam giác IAC, IAB cũng cắt OG tại S (được xác định bởi hệ thức (1))
Vậy các đường thẳng Euler của 4 tam giác IAB, IBC, ICA và ABC đồng quy tại S

ma 29
24-01-2009, 09:51 AM
II.8)Điểm Feuerbach

Kết quả:Trong một tam giác ,đường tròn Euler tiếp xúc với đường tròn nội tiếp của nó,và tiếp điểm đó được gọi là điểm Feuerbach của tam giác trên.

[Only registered and activated users can see links]

Chỉ dẫn chứng minh:(leductam post)
Gọi E, O, I là tâm đường tròn Euler, ngoại tiếp, nội tiếp của tam giác ABC. H là trực tâm, QOS là đường kính vuông góc với BC. P là hình chiếu của A lên OQ. R, r là bán kính đường tròn ngoại tiếp, nội tiếp.D là chân đường phân giác góc A

1.Đường tròn Euler tiếp xúc trong với đường tròn nội tiếp.
Ta dễ dàng chứng minh được PO = 2EF
Vì \Delta AQP \sim \Delta IDN
suy ra QP . IN = MK . DN (1)
Chứng minh tiếp: MK . DN = MN . NK
Vì \Delta SDC \sim SCA
nên {SC}^{2} = SD . SA
mà \Delta SCI cân tại S
do đó SC = SI
Vậy {SI}^{2} = SD . SA
Chiếu hệ thức trên lên BC theo phương vuông góc với BC ta được:
{MN}^{2} = MD . MK
\Leftrightarrow MN . MK - {MN}^{2} = MN . MK - MD . MK
\Leftrightarrow MN . NK = MK . DN (2)
Từ (1) và (2) ta suy ra:
IN . QP = MN . MK (3)
Ta có:
{EI}^{2} = {(IN - EF)}^{2} + {(MN - MF)}^{2}
\Leftrightarrow {EI}^{2} = {(IN - \frac{OP}{2})}^{2} + {(MN - \frac{MK}{2})}^{2}
\Leftrightarrow {EI}^{2} = {IN}^{2} - IN . PO + \frac{1}{4}{PO}^{2} + {MK}^{2}) - MN . MK + {MN}^{2}
\Leftrightarrow {EI}^{2} = \frac{{AO}^{2}}{4} + {IN}^{2} - IN . PO - MN . NK (4)
Từ (3) và (4) ta có:
{EI}^{2} = \frac{{R}^{2}}{4} + {r}^{2} - IN.(QP + PO) = \frac{{R}^{2}}{4} + {r}^{2} - Rr = {(\frac{R}{2} - r)}^{2}
Vậy EI = \frac{R}{2} - r suy ra dpcm.
Hoàn toàn tương tự ta cũng có:
2.Đường tròn Euler tiếp xúc ngoài với các đường tròn bàng tiếp
Từ {I}_{a} kẻ {I}_{a}{X}_{a} vuông góc với BC do {I}_{a}S = SI nên {X}_{a}M = MN
Từ (2) ta có:
\frac{MN}{NK} = \frac{DN}{NK} = \frac{2MN - DN}{(MK - NK) + MK} = \frac{{X}_{a}N - DN}{{X}_{a}M + MK} = \frac{{X}_{a}D}{{X}_{a}K}
Vậy MN . {X}_{a}K = MK . {X}_{a}D (5)
Mặt khác: \Delta AQP \sim \Delta {I}_{a}D{X}_{a} nên:
PQ . {X}_{a}{I}_{a} = MK . {X}_{a}D (6)
Từ (5) và (6) ta có: PQ . {X}_{a}{I}_{a} = MK . {X}_{a}D = MN . {X}_{a}K (7)
Từ E kẻ EL\perp {X}_{a}{I}_{a}. Trong tam giác vuông {I}_{a}LE ta có:
{{I}_{a}E}^{2} = {(EF + {X}_{a}{I}_{a})}^{2} + {({X}_{a}M + MF)}^{2}
= \frac{{OP}^{2}}{4} + OP . {X}_{a}{I}_{a} + {{X}_{a}{I}_{a}}^{2} + {{X}_{a}M}^{2} + {X}_{a}M . MK + \frac{{MK}^{2}}{4}
= \frac{1}{4}(\frac{{OP}^{2}}{4} + \frac{{MK}^{2}}{4}) + OP . {X}_{a}{I}_{a} + {{X}_{a}{I}_{a}}^{2} + {X}_{a}M({X}_{a}M + MK)
= \frac{{OA}^{2}}{4} + {{X}_{a}{I}_{a}}^{2} + OP . {X}_{a}{I}_{a} + MN . {X}_{a}K (8)
Từ (7) và (8) ta có:
{{I}_{a}E}^{2} = \frac{{OA}^{2}}{4} + {{X}_{a}{I}_{a}}^{2} + OP . {X}_{a}{I}_{a} + PQ . {X}_{a}{I}_{a}
= \frac{{R}^{2}}{4} + {{r}_{a}}^{2} + R{r}_{a} = {(\frac{R}{2} + {r}_{a})^{2}
Vậy E{I}_{a} = \frac{R}{2} + {r}_{a} suy ra dpcm :)) :D

nbkschool
25-01-2009, 12:14 AM
II.4)Điểm Gergonne,điểm Nobb, đường thẳng Gergone

1)Kết quả về điểm Gergonne:Tam giác ABC với đường tròn nội tiếp (I).Tiếp điểm của (I) trên BC,CA,AB lần lượt là D,E,F.Khi đó AD,BE,CF đồng quy tại một điểm gọi là điểm Gergonne của tam giác ABC.
[Only registered and activated users can see links]

Chỉ dẫn chứng minh:
Chỉ cần dùng định lí Ceva và các kết quả đơn giản : DB=DC,EA=EC,FA=FB là ra.:D

2)Kết quả về điểm Nobb và đường thẳng Gergonne(Vẫn với các kí hiệu trên)Một tam giác không cân có 3 điểm Nobb tương ứng là giao điểm của các cặp đường thẳng EF và CB ,DE và AB ,DF và AC. Và 3 điểm Nobb cùng nằm trên một đường thẳng gọi là đường thẳng Gergonne của tam giác ABC.

Chỉ dẫn chứng minh:

Xét cực và đối cực đối với (I).
Đường đối cực của A là EF đi qua M,nên đường đối cực của M đi qua A.
Mặt khác dễ thấy đường đối cực của M đi qua D nên suy ra đường đối cực của M là AD.
Hoàn toàn tương tự ta có:
Đường đối cực của N là BE và đường đối cực của P là CF

Theo trên ,do AD,BE,CF đồng quy nên sẽ có điều phải chứng minh.
Bình luận: Kết quả trên có thể mở rộng như sau:

Cho tam giác ABC và 3 điểm D,E,F theo thứ tự thuộc BC,CA,AB sao cho
AD,BE,CF đồng quy và D,E,F khác trung điểm đoạn thẳng.Gọi M,N,P lần lượt là điểm chung của các cặp đường thẳng (EF,BC) ,(DF,CA) ,(DE,AB).Khi đó M,N,P thẳng hàng

Bạn có thể chứng minh kết quả trên bằng định lí Menelaus nhưng thậm chí bài toán mở rộng này
cũng chỉ là trường hợp đặc biệt của định lí Desargues mà thôi!!!!

**Xem thêm:
Bổ sung tí,đường thẳng Gergonne vuông góc với đường thẳng đi qua điểm Gergonne và tâm nội tiếp (theo chứng minh trên)-đường thẳng này gọi là đường thẳng Soddy.Ngoài ra đường thẳng này còn đi qua điểm de Longchamps (điểm đối xứng với trực tâm qua tâm ngoại tiếp) và nhiều điểm linh tinh khác.(nhưng chắc cũng không cần thiết phải biết):dumb:

leductam
16-02-2009, 09:33 AM
II.9)Điểm Kosnita
Định nghĩa.Cho tam giác ABC, O là tâm đường tròn ngoại tiếp tam giác.Gọi O_1, O_2, O_3 là tâm các đường tròn ngoại tiếp tam giác BOC, AOC, AOB.Khi đó ba đường thẳng AO_1, BO_2 và CO_3 đồng quy tại điểm Konista của tam giác.

[Only registered and activated users can see links]

Chỉ dẫn chứng minh:
Gọi M, N, P tương ứng là giao điểm của AO_1, BO_2, CO_3 với BC, AC, AB.
Ta có:
\frac{\bar{MB}}{\bar{MC}} = \frac{{S}_{[A{O}_{1}B]}}{{S}_{[A{O}_{1}C]}} = - \frac{BA.\sin (A - C + 90)}{CA.\sin (A - B + 90)} = - \frac{BA.\cos (A - C)}{CA.\cos (A - B)}.
Tương tự với điểm N và P rồi sau đó nhân các tỉ lệ thức với nhau ta được dpcm :))

Mashimaru
20-02-2009, 09:44 PM
II.8)Điểm Feuerbach

Kết quả:Trong một tam giác ,đường tròn Euler tiếp xúc với đường tròn nội tiếp của nó,và tiếp điểm đó được gọi là điểm Feuerbach của tam giác trên.

[Only registered and activated users can see links]

Chỉ dẫn chứng minh:(leductam post)
Gọi E, O, I là tâm đường tròn Euler, ngoại tiếp, nội tiếp của tam giác ABC. H là trực tâm, QOS là đường kính vuông góc với BC. P là hình chiếu của A lên OQ. R, r là bán kính đường tròn ngoại tiếp, nội tiếp.D là chân đường phân giác góc A

1.Đường tròn Euler tiếp xúc trong với đường tròn nội tiếp.
Ta dễ dàng chứng minh được PO = 2EF
Vì \Delta AQP \sim \Delta IDN
suy ra QP . IN = MK . DN (1)
Chứng minh tiếp: MK . DN = MN . NK
Vì \Delta SDC \sim SCA
nên {SC}^{2} = SD . SA
mà \Delta SCI cân tại S
do đó SC = SI
Vậy {SI}^{2} = SD . SA
Chiếu hệ thức trên lên BC theo phương vuông góc với BC ta được:
{MN}^{2} = MD . MK
\Leftrightarrow MN . MK - {MN}^{2} = MN . MK - MD . MK
\Leftrightarrow MN . NK = MK . DN (2)
Từ (1) và (2) ta suy ra:
IN . QP = MN . MK (3)
Ta có:
{EI}^{2} = {(IN - EF)}^{2} + {(MN - MF)}^{2}
\Leftrightarrow {EI}^{2} = {(IN - \frac{OP}{2})}^{2} + {(MN - \frac{MK}{2})}^{2}
\Leftrightarrow {EI}^{2} = {IN}^{2} - IN . PO + \frac{1}{4}{PO}^{2} + {MK}^{2}) - MN . MK + {MN}^{2}
\Leftrightarrow {EI}^{2} = \frac{{AO}^{2}}{4} + {IN}^{2} - IN . PO - MN . NK (4)
Từ (3) và (4) ta có:
{EI}^{2} = \frac{{R}^{2}}{4} + {r}^{2} - IN.(QP + PO) = \frac{{R}^{2}}{4} + {r}^{2} - Rr = {(\frac{R}{2} - r)}^{2}
Vậy EI = \frac{R}{2} - r suy ra dpcm.
Hoàn toàn tương tự ta cũng có:
2.Đường tròn Euler tiếp xúc ngoài với các đường tròn bàng tiếp
Từ {I}_{a} kẻ {I}_{a}{X}_{a} vuông góc với BC do {I}_{a}S = SI nên {X}_{a}M = MN
Từ (2) ta có:
\frac{MN}{NK} = \frac{DN}{NK} = \frac{2MN - DN}{(MK - NK) + MK} = \frac{{X}_{a}N - DN}{{X}_{a}M + MK} = \frac{{X}_{a}D}{{X}_{a}K}
Vậy MN . {X}_{a}K = MK . {X}_{a}D (5)
Mặt khác: \Delta AQP \sim \Delta {I}_{a}D{X}_{a} nên:
PQ . {X}_{a}{I}_{a} = MK . {X}_{a}D (6)
Từ (5) và (6) ta có: PQ . {X}_{a}{I}_{a} = MK . {X}_{a}D = MN . {X}_{a}K (7)
Từ E kẻ EL\perp {X}_{a}{I}_{a}. Trong tam giác vuông {I}_{a}LE ta có:
{{I}_{a}E}^{2} = {(EF + {X}_{a}{I}_{a})}^{2} + {({X}_{a}M + MF)}^{2}
= \frac{{OP}^{2}}{4} + OP . {X}_{a}{I}_{a} + {{X}_{a}{I}_{a}}^{2} + {{X}_{a}M}^{2} + {X}_{a}M . MK + \frac{{MK}^{2}}{4}
= \frac{1}{4}(\frac{{OP}^{2}}{4} + \frac{{MK}^{2}}{4}) + OP . {X}_{a}{I}_{a} + {{X}_{a}{I}_{a}}^{2} + {X}_{a}M({X}_{a}M + MK)
= \frac{{OA}^{2}}{4} + {{X}_{a}{I}_{a}}^{2} + OP . {X}_{a}{I}_{a} + MN . {X}_{a}K (8)
Từ (7) và (8) ta có:
{{I}_{a}E}^{2} = \frac{{OA}^{2}}{4} + {{X}_{a}{I}_{a}}^{2} + OP . {X}_{a}{I}_{a} + PQ . {X}_{a}{I}_{a}
= \frac{{R}^{2}}{4} + {{r}_{a}}^{2} + R{r}_{a} = {(\frac{R}{2} + {r}_{a})^{2}
Vậy E{I}_{a} = \frac{R}{2} + {r}_{a} suy ra dpcm :)) :D

Em xin bổ sung một chứng minh khác bằng phép nghịch đảo ạ:

Xét \triangle ABC có:
+ Đường tròn nội tiếp (I) tiếp xúc với BC,CA,AB theo thứ tự tại P,Q,R.
+ Đường tròn (I_c) bàng tiếp trong góc C tiếp xúc với BC,CA,AB tại X,Y,Z.
+ Đường tròn Euler (S) qua trung điểm 3 cạnh BC,CA,AB là L,M,N.
Kẻ tiếp tuyến chung KG của (I) và (I_c) tiếp xúc với chúng lần lượt tại H,J. (chú ý là K\in CA và G \in CB). Gọi E là tâm vị tự trong của (I) và (I_c).

Ta có:
AR=BX=\fra{b+c-a}{2}\Rightarrow NX=NR=\fra{\|a-b\|}{2}

Nếu a=b thì hiển nhiên (S) đã tiếp xúc với (I) và (I_c) nên ta chỉ quan tâm đến trường hợp a\neq b.

Khi đó, (F,E,R,X)=(C,E,I,I_c)=-1 nên theo hệ thức Newton, NR^2=\overline{NE}.\overline{NF}.

Đặt \{M_1\} = MN \cap KG\Rightarrow NM_1=\fra{(a-b)^2}{2a}, lại có NM=\fra{a}{2} nên suy ra \overline{NM}.\overline{NM_1}=\fra{(a-b)^2}{4}.

Cuối cùng, xét phép nghịch đảo cực N, phương tích k=\fra{(a-b)^2}{4}: (I)\mapsto (I), (I_c)\mapsto (I_c), F\mapsto E,M\mapsto M_1 \Rightarrow (S)\mapsto KG. Nhưng KG tiếp xúc với (I) và (I_c) còn bản thân 2 đường tròn này bất biến qua phép nghịch đảo đang xét nên ta cũng có (S) cũng tiếp xúc với (I) và (I_c).

Lập luận tương tự cho thấy (S) tiếp xúc với (I), (I_a), (I_b), (I_c).

Kết thúc chứng minh!

ma 29
21-02-2009, 09:19 AM
Cái này có nhiều cách cm lắm ,mình có gửi thêm 2 file của anh Darij Grinberg +bên FG ở trên nữa :)

À,Mashi vẽ hình cho chứng minh của em để anh gộp luôn vào kia nhé:)

tqdung
19-03-2009, 06:09 PM
các bạn vào trang web này để chuyển web thành pdf ( dung lượn down về khá lớn ) và các bạn chỉ cẩn paste tên web vào để nó converter
[Only registered and activated users can see links]
(bạn nào converter xong có thể upload dc ko) :beatbrick:

Quân -k47DHV
28-03-2009, 04:04 PM
Bổ Đề:
1, tam giác ABC, BE,CF là phân giác. Điểm M nằm trong mặt phẳng tam giác thì M \in [EF] \Leftrightarrow d(M/BC)=d(M/AB)+d(M/AC)
2, tứ giác ABCD nội tiếp được, khi đó AD+BC=CD \Leftrightarrow phân giác góc \angle BAD,\angle ABC, và CD đồng quy.

magison
30-03-2009, 01:06 PM
Mình đã tổng hợp thành một file hoàn chỉnh, Nhưng tạm thời mới up lên một nửa để mọi người đọc xem có cần phải chỉnh thêm để cho đẹp hơn. Đây là link download
[Only registered and activated users can see links]

leductam
07-04-2009, 11:25 AM
II.10)Điểm Musselman,định lí Paul Yiu về điểm Musselman
Kết quả: Cho tam giác ABC. Các điểm {A}_{1}, {B}_{1}, {C}_{1} lần lượt là điểm đối xứng với A, B, C qua các cạnh đối diện và O là tâm đường tròn ngoại tiếp. Khi đó (AO{A}_{1}), (BO{B}_{1}) và (CO{C}_{1}) cùng đi qua 1 điểm là ảnh của điểm Kosnita qua phép nghịch đảo đường tròn ngoại tiếp tam giác ABC.
Chỉ dẫn chứng minh:

[Only registered and activated users can see links]

Gọi M và E lần lượt là ảnh của điểm {K}_{o} và {A}_{1} qua phép nghịch đảo đường tròn ngoại tiếp tam giác ABC.
Khi đó ta có:
O{K}_{o}.OQ={OA}^{2}=OE.O{A}_{1}
Suy ra \hat{OMA} = \hat{OA{K}_{o}}
\hat{O{A}_{1}A} = \hat{OAE}
Bây giờ ta cần chứng minh 3 điểm A, {K}_{o} và E thẳng hàng.
Ta có bộ tâm tỉ cự của điểm Kosnita (\frac{a}{\cos (B-C)} : \frac{b}{\cos (A-C)} : \frac{c}{\cos (B-A)})
và tâm đường tròn chín điểm N(a.\cos (B - C) : b.\cos (A - C) : c.\cos (B - A)) do đó điểm Kosnita và tâm đường tròn chín điểm là 2 điểm đẳng giác của tam giác ABC
Mặt khác O - tâm đường tròn ngoại tiếp và H - trực tâm cũng là 2 điểm đẳng giác của tam giác ABC nên suy ra \hat{OA{K}_{o}} = \hat{NA{A}_{1}}
Ta cũng dễ dàng chứng minh được \hat{NA{A}_{1}} = \hat{O{A}_{1}A}
Vậy \hat{OMA} = \hat{O{A}_{1}A} hay nói cách khác 4 điểm O, A, {A}_{1} và M cùng nằm trên 1 đường tròn
Tương tự với các đường tròn còn lại ta suy ra dpcm :D
Điểm M được gọi là điểm Musselman của tam giác ABC.

Định lí Paul Yiu về điểm Musselman: Với giả thiết như trên thì 3 đường tròn (A{B}_{1}{C}_{1}), (B{A}_{1}{C}_{1}) và (C{B}_{1}{A}_{1}) cũng đi qua điểm M.
Chỉ dẫn chứng minh:

[Only registered and activated users can see links]

Về định lí này em xin trích dẫn trực tiếp lời giải của Darij Grinberg:
Theo chứng minh trên thì điểm M nằm trên (BO{B}_{1}) và (CO{C}_{1}) nên ta được:
\hat{{B}_{1}MO} = \hat{{B}_{1}BO}
\hat{{C}_{1}MO} = \hat{{C}_{1}CO}
Ta có:
\hat{{B}_{1}M{C}_{1}} = \hat{{B}_{1}MO} + \hat{{C}_{1}MO}
= \hat{{B}_{1}BO} + \hat{{C}_{1}CO}
= (\hat{CB{B}_{1}} - \hat{CBO}) + (\hat{BC{C}_{1}} - \hat{BCO})
= \hat{CB{B}_{1}} + \hat{BC{C}_{1}} - (\hat{CBO} + \hat{BCO}
= \hat{CB{B}_{1}} + \hat{BC{C}_{1}} - \pi + \hat{BOC}
= \frac{\pi }{2} - \hat{C} + \frac{\pi }{2} - \hat{B} - \pi + 2\hat{A}
= 3\hat{A} - \pi
Mặt khác: \hat{{B}_{1}A{C}_{1}} = 2\pi - (\hat{BA{C}_{1}} + \hat{BAC} + \hat{CA{B}_{1}})
= 2\pi - 3\hat{A}
Vậy \hat{{B}_{1}M{C}_{1}} + \hat{{B}_{1}A{C}_{1}} = \pi
hay A, {B}_{1}, {C}_{1} và M cùng nằm trên đường tròn.
Tương tự với các đường tròn khác ta suy ra dpcm :))

thamtuhoctro
07-04-2009, 11:57 AM
II.11)Khái niệm vòng cực của tam giác.
Khái niệm::Cho tam giác tù ABC. Chân các đường cao đối diên các đỉnh là {H}_{a}, {H}_{b} và {H}_{c}.
Khi đó vòng cực của tam giác là đường tròn có tâm là H và bán kính r xác định với {r}^{2} = \bar{HA}.\bar{H{H}_{a}} = \bar{HB}.\bar{H{H}_{b}} = \bar{HC}.\bar{H{H}_{c}} = -4{R}^{2}\cos A.\cos B.\cos C = 4{R}^{2} - \frac{1}{2}(\sum {a}^{2})
Trong đó: R - bán kính đường tròn ngoại tiếp
a, b, c - độ dài 3 cạnh tam giác
A, B, C - 3 góc tam giác ABC

[Only registered and activated users can see links]

Ngoài ra ta có 1 tính chất của vòng cực là: cho 3 điểm bất kì chuyển động trên các đường cạnh của tam giác ABC dựng các đường tròn có đường kính là đoạn thẳng nối 1 đỉnh với điểm chuyển động trên cạnh đối diện thì khi đó vòng cực của tam giác trực giao với tất cả các đường tròn đó.

leductam
08-04-2009, 11:47 AM
II.12)Điểm Gibert
Kết quả:: Cho tam giác ABC và điểm Musselman - ảnh của điểm Kosnita qua phép nghịch đảo đường tròn ngoại tiếp tam giác ABC. Gọi {M}_{1}, {M}_{2}, {M}_{3} là điềm đối xứng với điểm Musselman qua các đường cạnh của tam giác. Khi đó các đường thẳng A{M}_{1}, B{M}_{2} và C{M}_{3} đồng quy tại điểm nằm trên đường tròn ngoại tiếp tam giác ABC.
Chỉ dẫn chứng minh:

[Only registered and activated users can see links]

Gọi giao điểm của A{M}_{1} và B{M}_{2} là {G}_{i} thì ta có:
\hat{{M}_{1}{G}_{i}B} = \hat{{M}_{1}AB} + \hat{{M}_{2}BA} = \hat{M{A}_{1}B} + \hat{M{B}_{1}A} = \pi - \hat{M{C}_{1}B} + \pi - \hat{M{C}_{1}A} = \hat{A{C}_{1}B} = \hat{C}
Vậy {G}_{i} nằm trên đường tròn ngoại tiếp tam giác ABC.
Chứng minh tương tự với A{M}_{1} và C{M}_{3} ta suy ra dpcm :))
Giao điểm của 3 đường thẳng trên được gọi là điểm Gibert của tam giác ABC

nbkschool
08-04-2009, 06:55 PM
II13/Trục Lemoine

Định lý:
Cho tam giác ABC nội tiếp đường tròn (O).Tiếp tuyến tại A của đường tròn cắt đường thẳng BC tại X.Định nghĩa tương tự cho Y,Z.Chứng minh rằng X,Y,Z thẳng hàng và đường thẳng chứa X,Y,Z được gọi là trục Lemoine của tam giác ABC

[Only registered and activated users can see links]

Chứng minh:
Không mất tính tổng quát giả sử hai vecto \vec{CB} và \vec{CX} cùng hướng.Do hai tam giác ABX và CXA đồng dạng với nhau suy ra:
\frac{XB}{XA}=\frac{AB}{AC}
\frac{XA}{XC}=\frac{AB}{AC}
Nhân hai đẳng thức trên với nhau suy ra:
\frac{XB}{XC}=\frac{AB}{AC}^2
Do X nằm ngoài tam giác nên suy ra:
\frac{\overline{XB}}{\overline{XC}}=\frac{AB}{AC}^ 2
Tương tự ta có hai đẳng thức sau:
\frac{\overline{YC}}{\overline{YA}}=\frac{BC}{BA}^ 2
\frac{\overline{ZA}}{\overline{ZB}}=\frac{CA}{CB}^ 2
Nhân ba đẳng thức với nhau ta có:
\frac{\overline{XB}.\overline{YC}.\overline{ZA}}{ \overline{XC}.\overline{YA}.\overline{ZB}}=1
Theo định lý Menelaus thì ba điểm X,Y,Z đồng quy.

Chứng minh (2):
Ta sẽ chứng minh đường đối cực của X,Y,Z đối với (O) lần lượt là ba đường đối trung của tam giác ABC.
Edit later...

leductam
11-04-2009, 03:01 PM
II.14)Tâm Morley
Kết quả::Tâm Morley thứ nhất được định nghĩa là tâm đường tròn ngoại tiếp của tam giác Morley thứ nhất. Tâm Morley thứ hai được định nghĩa là tâm phối cảnh của tam giác ABC với tam giác Morley thứ nhất.
Chỉ dẫn chứng minh:

[Only registered and activated users can see links]

Áp dụng định lí tri Xeva cho 3 đường thẳng đồng quy là A{A}_{1}, B{A}_{1} và C{A}_{1} ta có:
\frac{\sin BA{A}_{1}}{\sin CA{A}_{1}} . \frac{\sin CB{A}_{1}}{\sin AB{A}_{1}} . \frac{\sin AC{A}_{1}}{\sin BC{A}_{1}} = -1
mà \hat{{A}_{1}BC} = \frac{1}{2}\hat{AB{A}_{1}} = \frac{1}{3} \hat{ABC}
\hat{BC{A}_{1}} = \frac{1}{2}\hat{{A}_{1}CA} = \frac{1}{3} \hat{BCA}
Suy ra \frac{\sin BA{A}_{1}}{\sin CA{A}_{1}} = - \frac{\sin \frac{2}{3}ABC}{\sin \frac{1}{3}CBA} . \frac{\sin \frac{1}{3}BCA}{\sin \frac{2}{3}ACB}.
Tương tự với các đường còn lại ta được dpcm :D

leductam
11-04-2009, 04:54 PM
II.15) Tâm Spieker và đường thẳng Nagel
Kết quả:Cho tam giác ABC. Gọi {A}_{1}, {B}_{1}, {C}_{1} lần lượt là trung điểm của BC, CA và AB. Tâm đường tròn nội tiếp tam giác {A}_{1}{B}_{1}{C}_{1} là tâm Spieker của tam giác ABC.Khi đó 4 điểm tâm đường tròn nội tiếp, trọng tâm, điểm Nagel và tâm Spieker cùng nằm trên đường thẳng Nagel của tam giác ABC
Chỉ dẫn chứng minh:

[Only registered and activated users can see links]

Ta có phép vị tự tâm G tỉ số - \frac{1}{2} biến tam giác ABC thành tam giác {A}_{1}{B}_{1}{C}_{1} nên suy ra \vec{G{S}_{p}} = - \frac{1}{2}\vec{GI} (1)
Xét điểm Nagel có bộ số tâm tỉ cự:
\sum (p - a)\vec{NA} = \vec{o}
\Leftrightarrow p\sum \vec{NA} - \sum a\vec{NA} = \vec{o}
\Leftrightarrow 3p\vec{NG} = 2p\vec{NI}
\Leftrightarrow 3\vec{NG} = 2\vec{NI} (2)
Từ (1) và (2) ta suy ra 4 điểm I, N, G và {S}_{p} cùng nằm trên 1 đường thẳng.
Ngoài ra ta còn 1 vài tính chất của điểm Spieker như nó là tâm đẳng phương của 3 đường tròn bàng tiếp tam giác ABC, tâm Spieker, điểm Brocard thứ 3 và điểm đẳng cự với tâm đường tròn nội tiếp thẳng hàng...:))

leductam
11-04-2009, 06:12 PM
II.16)Hai điểm Fermat
Kết quả:Cho tam giác ABC. Dựng ra phía ngoài(vào trong) các tam giác đều BCM, ACN và ABP. Khi đó tâm phối cảnh của tam giác ABC và tam giác MNP được gọi là điểm Fermat thứ nhất(thứ hai) hay người ta còn gọi là điểm Fermat dương(âm).
Chỉ dẫn chứng minh:

[Only registered and activated users can see links]

[Only registered and activated users can see links]

Gọi giao điểm của AM, BN và CP lần lượt với BC, CA và AB là X, Y, Z ta có:
\frac{\bar{XB}}{\bar{XC}} = \frac{S[ABM]}{S[ACM]} = \frac{BM.BA.\sin (\vec{BM};\vec{BA})}{CM.CA.\sin (\vec{CM};\vec{CA})}
Tương tự với Y và Z ta được:
\frac{\bar{ZA}}{\bar{ZB}} . \frac{\bar{XB}}{\bar{XC}} . \frac{\bar{YC}}{\bar{YA}} = \frac{\sin (\vec{AP};\vec{AC})}{\sin (\vec{BP};\vec{BC})} . \frac{\sin (\vec{BM};\vec{BA})}{\sin (\vec{CM};\vec{CA})} . \frac{\sin (\vec{CN};\vec{CB})}{\sin (\vec{AN};\vec{AB})}
Mặt khác:(\vec{AP};\vec{AC}) \equiv (\vec{AP};\vec{AB}) + (\vec{AB};\vec{AC}) \equiv (\vec{AC};\vec{AN}) + (\vec{AB};\vec{AC}) \equiv (\vec{AB};\vec{AN})(mod 2\pi)
Tương tự ta suy ra dpcm :))
Ngoài ra:
{F}_{+} = (\frac{1}{\sqrt{3}{S}_{A} + S} : \frac{1}{\sqrt{3}{S}_{B} + S} : \frac{1}{\sqrt{3}{S}_{C} + S})
{F}_{-} = (\frac{1}{\sqrt{3}{S}_{A} - S} : \frac{1}{\sqrt{3}{S}_{B} - S} : \frac{1}{\sqrt{3}{S}_{C} - S})
Trong đó:
S là 2 lần diện tích tam giác ABC
{S}_{A} = S.\cot A

Xem thêm:
[Only registered and activated users can see links]

leductam
12-04-2009, 04:34 PM
II.17)Điểm Parry reflection.
Kết quả:: Cho tam giác ABC. Kẻ qua A, B, C các đường thẳng \alpha, \beta, \gamma song song với nhau và song song với đường thẳng Euler của tam giác. Gọi {\alpha}^{'}, {\beta}^{'}, {\gamma}^{'} lần lượt là các đường đối xứng với \alpha, \beta, \gamma qua BC, CA và AB. Khi đó các đường này đồng quy tại điểm Parry reflection của tam giác ABC.
Chỉ dẫn chứng minh:

[Only registered and activated users can see links]

Gọi H là trực tâm tam giác ABC, {A}_{2}, {B}_{2}, {C}_{2} là ảnh của A, B, C qua phép đối xứng trục BC, CA và AB. Phép vị tự tâm O tỉ số 2 biến tam giác ABC thành tam giác {A}_{1}{B}_{1}{C}_{1} khi đó 2 đường thẳng Euler của 2 tam giác này trùng nhau.
Gọi M, N là giao điểm của AH với BC và {B}_{1}{C}_{1}.
Ta có: \bar{H{A}_{2}} = \bar{A{A}_{2}} - \bar{AH} = 2(\bar{AX} - \bar{OP}) = 2(\bar{AH} + \bar{HM} - \bar{OP}) = 2(\bar{HM} + \bar{MN}) = 2\bar{HN}
Vậy {A}_{2} cũng là ảnh của H qua phép đối xứng trục {B}_{1}{C}_{1}.
Dựa vào tính chất đồng dạng ta suy ra được đường thẳng {\alpha}^{'} đối xứng với đường thẳng Euler qua trục {B}_{1}{C}_{1}
Tương tự với các đường thẳng còn lại ta suy ra 3 đường thẳng {\alpha}^{'}, {\beta}^{'}, {\gamma}^{'} đồng quy theo định lí Collings :D

leductam
13-04-2009, 09:32 AM
II.18)Đường tròn Taylor ,tâm Taylor
Kết quả:.Cho tam giác ABC các đường cao A{A}_{2}, B{B}_{2}, C{C}_{2}. Từ {A}_{2}, {B}_{2}, {C}_{2} kẻ các đường vuông góc với các cạnh khi đó chân các đường vuông góc này nằm trên cùng 1 đường tròn gọi là đường tròn Taylor của tam giác ABC.
Chỉ dẫn chứng minh:

[Only registered and activated users can see links]

Ta có: \hat{MSP} = \hat{A{A}_{2}P} = \hat{AH{B}_{2}} = \hat{A{C}_{2}{B}_{2}} = \hat{ANM}
Vậy 4 điểm M, N, P, S cùng nằm trên 1 đường tròn.
Tương tự với 2 bộ 4 điểm R, S, M, Q và P, Q, R, N.
Ta được: \hat{MRQ} = \hat{MSQ} = \hat{MSP} + \hat{PSQ} = \hat{MNA} + \hat{PNQ}
Vậy 4 điểm M ,N, Q, R cũng cùng nằm trên 1 đường tròn hay ta suy ra được dpcm :))
Tâm của đường tròn Taylor được gọi là tâm Taylor của tam giác

thamtuhoctro
13-04-2009, 10:38 AM
II.19)Điểm Bevan
Kết quả:. Cho tam giác ABC, {I}_{a}, {I}_{b}, {I}_{c} là tâm các đường tròn bàng tiếp. Khi đó tâm đường tròn ngoại tiếp tam giác {I}_{a}{I}_{b}{I}_{c} được gọi là điểm Bevan của tam giác ABC.

[Only registered and activated users can see links]

Sau đây ta sẽ đến với 1 vài tính chất cua điểm Bevan:
*Ta thấy rằng I là tâm đường tròn nội tiếp tam giác ABC chính là trực tâm của tam giác {I}_{a}{I}_{b}{I}_{c} khi đó đường tròn ngoại tiếp tam giác ABC là đường tròn chín điểm của tam giác {I}_{a}{I}_{b}{I}_{c} suy ra O là trung điểm của IB
*Tâm Spieker là trung điểm của đoạn nối trực tâm với điểm Bevan.
Đầu tiên ta chứng minh 3 điểm này thẳng hàng.
Xét tam giác IGO ta có:
\frac{\bar{BI}}{\bar{BO}} . \frac{\bar{HO}}{\bar{HG}} . \frac{\bar{SpG}}{\bar{SpI}} = 2 . \frac{3}{2} . \frac{1}{3} = 1
Vậy 3 điểm này thẳng hàng.
Xét tam giác HBO ta có 3 điểm I, G, Sp thẳng hàng suy ra:
\frac{\bar{SpH}}{\bar{SpB}} . \frac{\bar{IB}}{\bar{IO}} . \frac{\bar{GO}}{\bar{GH}} = 1
\Leftrightarrow \frac{\bar{SpH}}{\bar{SpB}} = \frac{1}{2} . (-2) = -1
Vậy ta có dpcm
*Điểm Bevan là trung điểm của đoạn nối điểm Nagel và điểm de Longchamp.
Điểm de Longchamp là điểm đối xứng của trực tâm qua tâm đường tròn ngoại tiếp.
Khi đó tương tự như trên ta cũng có được dpcm

[Only registered and activated users can see links]

leductam
13-04-2009, 11:11 AM
II.20)Điểm Vecten
Kết quả:. Cho tam giác ABC. Dựng ra phía ngoài(hay vào trong) các hình vuông CBMN, BARS và ACPQ. Khi đó đường nối 1 đỉnh của tam giác với tâm hình vuông dựng trên cạnh đối diện đồng quy tại điểm Vecten của tam giác ABC.
Chỉ dẫn chứng minh:

[Only registered and activated users can see links]

Theo định lí Kiepert thì điều này hiển nhiên và cũng dễ dàng suy ra được có 2 điểm Vecten trong và ngoài(hay âm và dương).
Ngoài ra ta còn có 1 tính chất khá thú vị về điểm Vecten: Tâm đường tròn chín điểm và 2 điểm Vecten thẳng hàng.

leductam
13-04-2009, 11:50 AM
II.21)Điểm Mittenpunkt
Kết quả: Cho tam giác ABC, {I}_{a}, {I}_{b}, {I}_{c} là tâm các đường tròn bàng tiếp, {A}_{1}, {B}_{1}, {C}_{1} lần lượt là trung điểm các cạnh BC, CA, AB. Khi đó các đường thẳng {I}_{a}{A}_{1}, {I}_{b}{B}_{1}, {I}_{c}{C}_{1} đồng quy tại điểm Mittenpunkt của tam giác ABC.
Chỉ dẫn chứng minh:

[Only registered and activated users can see links]

Ta có:2\vec{{I}_{a}{A}_{1}} = \vec{{I}_{a}B} + \vec{{I}_{a}C} = \frac{\sin {I}_{c} . \cos {I}_{a}}{\sin {I}_{b}}\vec{{I}_{a}{I}_{c}} + \frac{\sin {I}_{b} . \cos {I}_{a}}{\sin {I}_{c}}\vec{{I}_{a}{I}_{b}}
Chiếu hệ thức theo phương {I}_{a}{A}_{1} lên trục {I}_{b}{I}_{c} ta được:
\frac{\sin {I}_{c} . \cos {I}_{a}}{\sin {I}_{b}}\bar{MC} + \frac{\sin {I}_{b} . \cos {I}_{a}}{\sin {I}_{c}}\bar{MB} = 0
\Leftrightarrow \frac{\bar{MB}}{\bar{MC}} = \frac{{(\sin {I}_{c})}^{2}}{{(\sin {I}_{b})}^{2}}
Tương tự với các đường còn lại ta suy ra dpcm
Ta có: M = ({(\sin {I}_{a})}^{2} : {(\sin {I}_{b})}^{2} : {(\sin {I}_{c})}^{2}) = ({(\cos \frac{A}{2})}^{2} : {(\cos \frac{B}{2})}^{2} : {(\cos \frac{C}{2})}^{2})
Ngoài ra ta còn có 1 vài tính chất bên lề khá thú vị: Với giả thiết như trên gọi D, E, F là điểm tiếp xúc của đường tròn nội tiếp tam giác ABC với 3 cạnh thì {I}_{a}D, {I}_{b}E, {I}_{c}F đồng quy. Khi đó điểm này và điểm Mittenpunkt là 2 điểm đẳng giác.
*Điểm Mittenpunkt, tâm Spieker và trực tâm thẳng hàng
*Điểm Mittenpunkt, tâm đường tròn nội tiêp và điểm Lemoine thẳng hàng.
*Điểm Mittenpunkt, trọng tâm va điểm Gergonne thẳng hàng với GeG : GM = 2 : 1 :D

thamtuhoctro
13-04-2009, 12:03 PM
II.22)Điểm Napoleon
Kết quả:.Cho tam giác ABC, dựng ra phía ngoài( hay vào trong) các tam giác đều BCM, CAP và ABQ. Khi đó đường nối 1 đỉnh của tam giác với trọng tâm tam giác đều dựng trên cạnh đối diện đồng quy tại điểm Napoleon của tam giác ABC.
Chỉ dẫn chứng minh:

[Only registered and activated users can see links]

Áp dụng định lí Kiepert ta suy ra dpcm cũng dễ dàng suy ra có 2 điểm Napoleon là trong và ngoài
{N}_{-} = (\frac{1}{{S}_{A} - \sqrt{3}S} : \frac{1}{{S}_{B} - \sqrt{3}S} : \frac{1}{{S}_{C} - \sqrt{3}S})
{N}_{+} = (\frac{1}{{S}_{A} + \sqrt{3}S} : \frac{1}{{S}_{B} + \sqrt{3}S} : \frac{1}{{S}_{C} + \sqrt{3}S}) :D

ma 29
11-05-2009, 04:16 PM
II.23)Đường tròn Adam

Kết quả:Cho tam giác ABC với điểm Gergonne G.Đường thẳng qua G song song với EF cắt AB,AC ở S,P.Đường thẳng qua G song song với DE cắt AC,BC ở Q,M.Đường thẳng qua G song song với DF cắt BA,BC ở R,N.Khi đó các điểm M,N,P,Q,R,S cùng thuộc một đường tròn gọi là đường tròn Adam của tam giác ABC.



Chỉ dẫn chứng minh:
[Only registered and activated users can see links]

Gọi (I) là đường tròn nội tiếp tam giác ABC,tiếp điểm của (I) trên BC,CA,AB lần lượt là D,E,F.Đường thẳng qua A và G song song với BC tương ứng cắt DE,DF ở (H,K),(V,T).
Ta thấy:
\frac{AK}{AH}=\frac{AK}{BD}. \frac{BD}{DC} .\frac{DC}{AH}=\frac{AF}{FB}.\frac{BF}{EC}.\frac{E C}{EA} =1.
Do đó AH=AK nên GT=GV.
Bây giờ để ý rằng GTDN và GVDM là hai hình bình hành nên ta cũng có DM=DN.
Kết hợp với ID vuông góc với BC ta thu được IM=IN.
Tương tự IP=IQ,IR=IS
Mặt khác dễ thấy :IM=IQ=IR.
Từ các khẳng định trên ta dễ nhận được điều cần chứng minh.

thamtuhoctro
06-06-2009, 10:06 AM
II.24)Tam giác Fuhrmann ,đường tròn Fuhrmann
Kết quả:Cho tam giác ABC nội tiếp đường tròn tâm O. Gọi {M}_{a}, {M}_{b}, {M}_{c} là trung điểm các cung BC, CA, AB. Lấy các điểm trên đối xứng qua các cạnh tương ứng ta được 3 điểm nữa là {F}_{a}, {F}_{b} và {F}_{c}.Khi ấy tam giác {F}_{a}{F}_{b}{F}_{c} được gọi là tam giác Fuhrmann của tam giác ABC.

[Only registered and activated users can see links]

Đường tròn ngoại tiếp tam giác Fuhrmann được gọi là đường tròn Fuhrmann.

Tính chất:
1) S = - \frac{{(\sum {{a^3}} - \sum {{a^2}b} + 3{\rm{a}}bc){S_{ABC}}}}{{(b + c - a)(a + c - b)(a + b - c)}} = - \frac{{(a + b + c)O{I^2}}}{{4{\rm{R}}}}

2){a^'} = \sqrt {\frac{{(b + c - a)(a + b + c)}}{{bc}}} OI

{b^'} = \sqrt {\frac{{(a + c - b)(a + b + c)}}{{ac}}} OI

{c^'} = \sqrt {\frac{{(a + b - c)(a + b + c)}}{{ab}}} OI

3)Trực tâm của tam giác Fuhrmann trùng với tâm đường tròn nội tiếp của tam giác ABC.(để chứng minh ta có thế sử dụng tích vô hướng)

4)Tâm đường tròn chín điểm của tam giác Fuhrmann và tam giác ABC trùng nhau. Bán kính đường tròn ngoại tiếp tam giác Fuhrmann có độ dài bằng OI.

Ngoài ra còn rất nhiều tính chất thú vị khác có thế tham khảo thêm trong link
[Only registered and activated users can see links]

leductam
06-06-2009, 10:08 AM
II.25)Hình luc giác và đường tròn Lemoine thứ nhất
Kết quả:Cho tam giác ABC và điểm Lemoine(L). Qua L kẻ các đường thẳng song song với các cạnh cắt các cạnh còn lại tại M, N, P, Q, R, S. Khi ấy lục giác MNPQRS được gọi là lục giác Lemoine thứ nhất của tam giác ABC.

[Only registered and activated users can see links]

Tính chất:
1)Lục giác Lemoine thứ nhất là lục giác ngoại tiếp. Đường tròn ngoại tiếp lục giác này được gọi là đường tròn Lemoine thứ nhất:
Do AL là đường thẳng đối trung đi qua trung điểm của QR nên QR là đường đối song tương ứng cạnh BC. Suy ra tứ giác PQRS là tứ giác nội tiếp.

2)Các cạnh bị kẹp giữa các đường song song Lemoine tỉ lệ với lũy thừa bậc ba cạnh tương ứng:
Gọi x, y, z là khoảng cách từ L tới 3 cạnh tam giác. Khi đó:
x = \frac{{a}^{2}{h}_{a}}{{a}^{2} + {b}^{2} + {c}^{2}}

y = \frac{{b}^{2}{h}_{b}}{{a}^{2} + {b}^{2} + {c}^{2}}

z = \frac{{c}^{2}{h}_{c}}{{a}^{2} + {b}^{2} + {c}^{2}}

\Rightarrow MN : RS : PQ = {a}^{3} : {b}^{3} : {c}^{3}

3)3 đoạn trên cùng 1 cạnh tỉ lệ với bình phương các cạnh của tam giác
BM : MN : NC = {c}^{2} : {a}^{2} : {b}^{2}

4)Tâm của đường tròn Lemoine thứ nhất là trung điểm đoạn nối điểm Lemoine với tâm đường tròn ngoại tiếp tam giác ABC:
Để ý AO vuông góc với QR khi đó ta dễ dàng suy ra được dpcm.

5)Bán kính {R_1} = \frac{{abc\sqrt {{a^2}{b^2} + {c^2}{b^2} + {a^2}{c^2}} }}{{({a}^{2} + {b}^{2} + {c}^{2})4{{\rm{S}}_{ABC}}}}

Ngoài ra ta còn có cách viết khác:
{(2{R}_{1})}^{2} = {R}^{2} + {{R}_{2}}^{2}
Trong đó {R}_{2} là bán kính đường tròn Lemoine thứ hai(sẽ được nói đến trong phần sau):))

thamtuhoctro
06-06-2009, 11:19 AM
II.26)Hình lục giác và đường tròn Lemoine thứ hai

Khái niệm về đường đối song:Cho tam giác ABC. Với 2 điểm D,E bất kì thuộc các cạnh AB và AC ta có 2 kiểu chọn DE sao cho tam giác ADE đồng dạng với tam giác ABC. Thứ nhất là DE song song với BC. Thứ hai là tứ giác BCED nội tiếp như hình vẽ. Khi ấy DE và BC được gọi là các đường đối song tương ứng với góc A.

Tính chất:Đường đối trung luôn đi qua trung điểm của các đường đối song tương ứng với cùng 1 đỉnh.

[Only registered and activated users can see links]

Kết quả:Cho tam giác ABC và điểm Lemoine(L). Qua L kẻ các đường đối song tương ứng với các cạnh của tam giác ABC cắt các cạnh còn lại tại M, N, P, Q, R, S. Khi đó lục giác MNPQRS được gọi là lục giác Lemoine thứ hai.

[Only registered and activated users can see links]

Tính chất:
1)Lục giác Lemoine thứ hai nội tiếp 1 đường tròn và đường tròn này được gọi là đường tròn Lemoine thứ hai của tam giác ABC.

2)Bán kính {R}_{2} = \frac{abc}{{a}^{2} + {b}^{2} + {c}^{2}}

Thực chất lục giác Lemoine và đường tròn Lemoine chỉ là trường hợp đặc biệt của lục giác Tucker và đường tròn Tucker. Xem thêm trong:
[Only registered and activated users can see links]

leductam
06-06-2009, 04:08 PM
II.27)Điểm Euler của Tứ giác nội tiếp
Kết quả:Cho tứ giác nội tiếp ABCD. Gọi {H}_{a}, {H}_{b}, {H}_{c}, {H}_{d} lần lượt là trực tâm các tam giác BCD, CDA, DAB và ABC. Khi ấy thì các đường thẳng A{H}_{a}, B{H}_{b}, C{H}_{c} và D{H}_{d} đồng quy. Điểm đồng quy được gọi là điểm Euler của tứ giác nội tiếp.

[Only registered and activated users can see links]

Chỉ dẫn chứng minh:
Ta có A{H}_{c}, C{H}_{a} song song và cùng bằng 2 lần khoảng cách từ O tới BD nên tứ giác AC{H}_{a}{H}_{c} là hình bình hành suy ra A{H}_{a}, C{H}_{c} giao nhau tại trung điểm mỗi đường.
Tương tự ta suy ra bốn đường thẳng đồng quy tại trung điểm mỗi đường.

Từ đây ta suy ra được nhiều tính chất thú vị của điểm Euler:

1)Điểm Euler nằm trên đường vuông góc hạ từ trung điểm một cạnh tới cạnh đối diện(hoặc trung điểm đường chéo tới đường chéo còn lại).

2)Đường thẳng Simson của đỉnh A với tam giác BCD thì đi qua điểm Euler. Tương tự với các đỉnh còn lại.

3)Đường tròn chín điểm của các tam giác ABC, BCD, CDA và DAB đồng quy tại điểm Euler.
Xem thêm:

[Only registered and activated users can see links]

thamtuhoctro
06-06-2009, 04:11 PM
II.28)Đường thẳng Steiner của tứ giác toàn phần
Kết quả:Cho tứ giác toàn phần ABCDEF. Khi đó trực tâm của các tam giác AEF, DCE, ABC và BDF cùng nằm trên 1 đường thẳng được gọi là đường thẳng Steiner của tứ giác toàn phần.

[Only registered and activated users can see links]

Chỉ dẫn chứng minh:
Gọi {H}_{1}, {H}_{2}, {H}_{3}, {H}_{4} lần lượt là trực tâm các tam giác AEF, DCE, ABC và BDF.
Gọi X, Z là trung điểm các đường chéo BE, AD.
Khi đó: {P}_{{H}_{2}/(X,XB)} = \bar{{H}_{2}P}.\bar{{H}_{2}E} = \bar{{H}_{2}Q}.\bar{{H}_{2}D} = {P}_{{H}_{2}/(Z,ZD)}
Vậy {H}_{2} nằm trên trục đẳng phương của 2 đường tròn (X,XB) và (Z,ZD).
Tương tự ta cũng có 3 trực tâm còn lại cùng nằm trên trục đẳng phương của hai đường tròn này suy ra dpcm.:D

leductam
06-06-2009, 04:12 PM
II.29)Đường thẳng Gauss của tứ giác toàn phần.
Kết quả:Cho tư giác toàn phần ABCDEF. Khi đó trung điểm các đường chéo cùng nằm trên một đường thẳng được gọi là đường thẳng Gauss của tứ giác toàn phần.

[Only registered and activated users can see links]

Chỉ dẫn chứng minh:
Gọi M, N, P lần lượt là trung điểm các đường chéo BE, CF và AD.
IJK là tam giác trung bình của tam giác ABC.
Khi đó các điểm M, N, P nằm trên các cạnh của tam giác IJK.
Ta có:
\frac{\bar{MK}}{\bar{MI}} = \frac{\bar{EA}}{\bar{EC}}

\frac{\bar{NI}}{\bar{NJ}} = \frac{\bar{FB}}{\bar{FA}}

\frac{\bar{PJ}}{\bar{PK}} = \frac{\bar{DC}}{\bar{DB}}

Nhân các vế các đẳng thức trên ta được:
\frac{\bar{MK}}{\bar{MI}} . \frac{\bar{NI}}{\bar{NJ}} . \frac{\bar{PJ}}{\bar{PK}} = \frac{\bar{EA}}{\bar{EC}} . \frac{\bar{FB}}{\bar{FA}} . \frac{\bar{DC}}{\bar{DB}} = 1
Suy ra dpcm.
Từ 2 bài viết trên ta thấy rằng trong 1 tứ giác toàn phần thì đường thẳng Steiner vuông góc với đường thẳng Gauss.:))

thamtuhoctro
06-06-2009, 04:14 PM
II.30) Điểm Miquel của tứ giác toàn phần
Kết quả:Cho tứ giác toàn phần ABCDEF. Khi ấy đường tròn ngoại tiếp của các tam giác AEF, CDE, ABC và BDE đồng quy. Điểm đồng quy đó được gọi là điểm Miquel của tứ giác toàn phần.

[Only registered and activated users can see links]

Chỉ dẫn chứng minh:
Giả sử đường tròn ngoại tiếp các tam giác AEF và CDE giao nhau tại điểm M khác E.
Khi đó ta có:
(\vec{MA} ; \vec{MC}) \equiv (\vec{MA} ; \vec{ME}) + (\vec{ME} ; \vec{MC}) \equiv ( - \vec{FA} ; \vec{FE}) + (\vec{DE} ; \vec{DC}) \equiv (\vec{AF} ; \vec{AE}) + (\vec{AE} ; \vec{FE}) + (\vec{DE} ; \vec{DC})
\equiv (\vec{AF} ; \vec{AE}) + (\vec{EC} ; \vec{ED}) + (\vec{DE} ; \vec{DC}) \equiv (\vec{AB} ; \vec{AC}) + (\vec{CA} ; \vec{CB}) \equiv \pi - (\vec{BC} ; \vec{BA}) \equiv (\vec{AB} ; \vec{BC})(mod 2\pi)
Vậy M nằm trên đường tròn ngoại tiếp tam giác ABC. Tương tự với đường tròn còn lại ta suy ra dpcm.:))

leductam
06-06-2009, 04:18 PM
II.31)Đường tròn Miquel của tứ giác toàn phần
Kết quả:Cho tứ giác toàn phần ABCDEF. Khi đó điểm Miquel và tâm của các đường tròn ngoại tiếp tam giác AEF, CDE, ABC và BDE cùng nằm trên 1 đường tròn - đường tròn Miquel của tứ giác toàn phần.

[Only registered and activated users can see links]

Chỉ dẫn chứng minh:
Gọi {O}_{1}, {O}_{2}, {O}_{3} và {O}_{4} lần lượt là tâm đường tròn ngoại tiếp các tam giác AEF, CDE, ABC và BDE.
Gọi {P}_{1}, {P}_{2}, {P}_{3} lần lượt là chân các đường vuông góc kẻ từ M tới {O}_{2}{O}_{4}, {O}_{2}{O}_{3} và {O}_{3}{O}_{4}.
Do {P}_{1}, {P}_{2}, {P}_{3} là trung điểm của MD, MC, MB nên chúng thẳng hàng.
Theo định lí đảo về đường thẳng Simson ta có M, {O}_{2}, {O}_{3}, {O}_{4} cùng nằm trên 1 đường tròn.
Tương tự ta suy ra dpcm.:)

ma 29
07-06-2009, 08:07 AM
II.32)Hình bình hành Varignon của tứ giác .


Kết quả:Cho tứ giác ABCD có M,N,P,Q lần lượt là trung điểm của AB,BC,CD,DA.Khi đó M,N,P,Q là bốn đỉnh của một hình bình hành gọi là hình bình hành Varignon của tứ giác ABCD.

Chỉ dẫn chứng minh:

[Only registered and activated users can see links]

Chứng minh kết quả này khá đơn giản,dễ thấy MN,PQ tương ứng là đường trung bình của các tam giác ABC và ACD thế nên MN//PQ ,MN=PQ=\frac{AC}{2}.
Do vậy MNPQ là một hình bình hành.

phuonglvt
04-11-2009, 01:34 PM
bài viết khá hay và dầy đủ, nhưng sao mọi người không viết tiếp phần một số mảng kiến thức quan trọng đi vì mấy phần đó hầu như mình đều chưa biết tới, ai có thế up nốt phần còn lại cho mình down với

caubedien
07-11-2009, 02:03 PM
ai có thể up tất cả cái này lên PDF đc ko

falling down
26-12-2009, 10:06 PM
Cho mình hỏi cách CM đlý Papus bằng Menelaus với :D cách đó phù hợp với kiến thức THCS hơn nhiều :))

353535
23-08-2010, 10:04 PM
Ai có thể tổng hợp tất cả bài viết trên cho mọi người đc ko :sad:

novae
23-08-2010, 10:14 PM
Cái đó anh đang làm, còn thiếu mấy phần nữa là xong

khicon
10-09-2010, 01:44 PM
Cái đó anh đang làm, còn thiếu mấy phần nữa là xong

sắp có chưa ạ?? rất mong tin của anh B-)

novae
12-09-2010, 03:19 PM
Chắc khoảng cuối tuần sau sẽ có (nếu không có gì trục trặcB-))

Thien tai
12-09-2010, 03:51 PM
:)) hình như đã có người làm rùi đây nez

novae
12-09-2010, 04:03 PM
Bản đó mới được một nửa:))
sắp tới sẽ là bản full, có bổ sung thêm một số định lý và tính chất:))

hungchng
24-09-2010, 11:41 PM
Bản đó mới được một nửa:))
sắp tới sẽ là bản full, có bổ sung thêm một số định lý và tính chất:))

Khi nào làm xong, gởi source cho tôi, sẽ tinh chỉnh và làm đẹp lại cho.
Không lẻ phí công làm lại từ đầu à.

Dr@gon
26-09-2010, 03:20 PM
I.10) Bất đẳng thức Ptolemy


Định lý:
Cho tứ giác ABCD. Khi đó có AC.BD \leq AB.CD + AD.BC
[Only registered and activated users can see links]
Chứng minh:
Lấy E nằm trong tứ giác ABCD sao cho
\hat{EDC}=\hat{ADB} và \hat{ECD}=\hat{ABD}
Khi đó \Delta ABD ~ \Delta ECD \Rightarrow \frac{AB}{BD}=\frac{EC}{DC} hay AB.DC=EC.BD.
Hơn nữa \Delta ADE ~ \Delta BDC (c.g.c) \Rightarrow \frac{AD}{AE}=\frac{BD}{CB} hay AD.CB=BD.AE.
Vậy ta có AB.CD + BC.AD = BD(EA+EC) \geq BD.AC(đpcm).
Xem thêm:[Only registered and activated users can see links]
lộn rồi Bất đẳng thức PTÔ-LÊ-MÊ mới đúng

novae
26-09-2010, 04:39 PM
Biết gì mà ăn nói hàm hồ, Ptô-lê-mê là phiên âm tiếng Việt của Ptolemy

Dr@gon
27-09-2010, 11:13 AM
Biết gì mà ăn nói hàm hồ, Ptô-lê-mê là phiên âm tiếng Việt của Ptolemy
Dạ tại em gà mà sao tới giờ vẫn chưa có bản full dzậy ?

khicon
27-09-2010, 07:13 PM
Khi nào làm xong, gởi source cho tôi, sẽ tinh chỉnh và làm đẹp lại cho.
Không lẻ phí công làm lại từ đầu à.

vậy đã có bản full chưa ạ??
sắp thi rồi, mong tập tài liệu này quá :-x:-x

novae
27-09-2010, 07:28 PM
Sau khi trao đổi với anh ma thì phải sửa đổi bổ sung một số chỗ, do đó sẽ mất thêm một thời gian nữa, nhưng mình sẽ cố gắng làm nhanh nhất có thể :D

tungcybtk22
20-10-2010, 09:21 PM
Thiếu mất 3/4 của định lí morley rồi:)

novae
20-10-2010, 09:23 PM
Thiếu mất 3/4 của định lí morley rồi:)

Thiếu ở đâu thì bạn nói cụ thể ra xem nào

craft_man
20-10-2010, 09:59 PM
Anh ơi có bản full anh gửi tin nhắn thông báo cho em cái nhá
đọc file " một nửa " thấy hay quá ạ
sợ không chờ nổi !!!!!

novae
20-10-2010, 10:01 PM
Có lâu rồi :))
[Only registered and activated users can see links]

avip
15-12-2010, 10:55 PM
Em có một ý kiến nho nhỏ: Trong một số TH, việc tra cứu ngay trên diễn đàn sẽ tiện lợi hơn so với phải mở ebook và đọc. Nhưng có một bất tiện của topic là dài đến 11 trang. Do đó, anh mod ma_29 có thể edit post #1 của topic và đánh số trang sau những định lí để dễ bề tra cứu được không ạ? Anh chỉ cần đánh số trang vào một định lí ở đầu trang đó là được rồi. Cảm ơn anh!

n.v.thanh
17-12-2010, 06:57 PM
In thành sách mà đọc.Các anh ý làm ebook để cho mọi người tiện trao đổi và đọc....Các anh ý đều rất bận,mới lại làm việc đó chẳng hay ho gì.Học vở học sách hay hơn học WEB:-??

BMW
26-12-2010, 12:36 AM
I.23) Định lí Thébault
Định lí: Cho tam giác ABC nội tiếp đường tròn (O). D là một điểm nằm trên cạnh BC. Đường tròn tâm P tiếp xúc với 2 đoạn AD,DC và tiếp xúc trong với (O). Đường tròn tâm Q tiếp xúc với 2 đoạn AD,DB và tiếp xúc trong với (O). Gọi I là tâm nội tiếp tam giác ABC. Ta có: P,I,Q thẳng hàng.
Chứng minh
[Only registered and activated users can see links]
Gọi G,H lần lượt là tiếp điểm của (Q) với DB,AD. Gọi I là giao điểm của EF và GH. Theo định lí lyness mở rộng(đã có trong bài của trung anh), I là tâm nội tiếp tam giác ABC. Vậy ta chỉ cần chứng minh P,I,Q thẳng hàng. Thật vậy, gọi X,Y lần lượt là giao điểm của GH và DQ; EF và DP. Áp dụng định lí Thales ta có: \frac{IX}{PD}=\frac{YD}{PD}=\frac{QX}{QD}. Vậy , P,I,Q thẳng hàng(dpcm)


------------------------------


Chứng minh: \frac{IQ}{IP}=tan^{2}\frac{\widehat{ADC}}{2}

avip
26-12-2010, 11:14 AM
------------------------------


Chứng minh: \frac{IQ}{IP}=tan^{2}\frac{\widehat{ADC}}{2}

Phải là \frac{IQ}{IP}=cot^{2}\frac{\widehat{ADC}}{2} mới đúng.
Ta có: \frac{IQ}{IP} = \frac{YD}{YP} = \frac{YD/YE}{YP/YE} = \frac{\cot{\frac{ADC}{2}}}{\tan{\frac{ADC}{2}}} = \cot^2{\frac{ADC}{2}}.

luatdhv
28-03-2011, 12:00 AM
I.11) Định lý Pascal


Định lý:
Cho 6 điểm A,B,C,D,E,F cùng thuộc một đường tròn. Khi đó các giao điểm của các cặp cạnh AB và DE, BC và EF, CD và FA thẳng hàng.

[Only registered and activated users can see links]
Chứng minh:
Gọi P,M,N lần lượt là giao điểm của AF và CD, AB và DE, BC và EF. Gọi P', M', N' lần lượt là giao điểm của BC và DE, BC và AF, DE và AF.
Áp dụng định lí Menelaus cho \Delta P'M'N' với cát tuyến PCD:
\frac{\bar{CP'}}{\bar{CM'}}.\frac{\bar{DN'}}{\bar{ DP'}}.\frac{\bar{PM'}}{\bar{PN'}}=1
\Leftrightarrow \frac{\bar{PM'}}{\bar{PN'}}=\frac{\bar{CM'}}{\bar{ CP'}}.\frac{\bar{DP'}}{\bar{DN'}}
Tương tự ta có:
\frac{\bar{NP'}}{\bar{NM'}}=\frac{\bar{FN'}}{\bar{ FM'}}.\frac{\bar{EP'}}{\bar{EN'}} và \frac{\bar{MN'}}{\bar{MP'}}=\frac{\bar{AN'}}{\bar{ AM'}}.\frac{\bar{BM'}}{\bar{BP'}}
Nhân các biểu thức trên lại kết hợp với các biểu thức phương tích sau:
\bar{BM'}.\bar{CM'}=\bar{AM'}.\bar{FM'}
\bar{EN'}.\bar{DN'}=\bar{FN'}.\bar{AN'}
\bar{CP'}.\bar{BP'}=\bar{DP'}.\bar{EP'}
Ta có :
\frac{\bar{NP'}}{\bar{NM'}}.\frac{\bar{MN'}}{\bar{ MP'}}.\frac{\bar{PM'}}{\bar{PN'}}=1.
Áp dụng định lí Menelaus đảo ta có đpcm.

Các bạn có thể vào đây xem thêm:[Only registered and activated users can see links]

Cho mình hỏi Đinh lý Pascal có được phát biểu cho siêu mặt bậc 2 cấp n không nhỉ?

abctom123
17-04-2011, 09:25 PM
Mình sẽ làm nốt phần về Menelaus :))

I.2)Mở rộng định lí Menelaus theo diện tích


Định lí:Cho tam giác ABC và 3 điểm M,N,P lần lượt nằm trên BC,CA,AB.Khi đó ta có:

\frac{S_{MNP}}{S_{ABC}}= \frac{ \bar{BM}.\bar{CN}.\bar{AP}-\bar{CM}.\bar{AN}.\bar{BP} }{\bar{AB} .\bar{BC}.\bar{CA}}

Chứng minh :(thamtuhoctro post)
[Only registered and activated users can see links]

Gọi e_1 ,e_2 ,e_3 là vector chỉ phương của BC, CA, AB.
Ta có:
\begin{array}{l}S\left[ {ABC} \right] = S\left[ {MAB} \right] + S\left[ {MCA} \right] \\ \Rightarrow S\left[ {ABC} \right] = S\left[ {PMA} \right] + S\left[ {PBM} \right] + S\left[ {NMC} \right] + S\left[ {NAM} \right] \\\Rightarrow S\left[ {ABC} \right] = S\left[ {MNP} \right] + S\left[ {BMP} \right] + S\left[ {CNM} \right] + S\left[ {APN} \right] \\ \end{array}
mặt khác :
\frac{{S\left[ {BMP} \right]}}{{S\left[ {ABC} \right]}} = \frac{{\overline {BM} .\overline {BP} .\sin \left( {e_1 ;e_2 } \right)}}{{\overline {BC} .\overline {BA} .\sin \left( {e_1 ;e_2 } \right)}} = \frac{{\overline {BM} .\overline {BP} }}{{\overline {BC} .\overline {BA} }}
tương tự:

\frac{{S\left[ {CNM} \right]}}{{S\left[ {ABC} \right]}} = \frac{{\overline {CN} .\overline {CM} }}{{\overline {CA} .\overline {CB} }}

\frac{{S\left[ {APN} \right]}}{{S\left[ {ABC} \right]}} = \frac{{\overline {AP} .\overline {AN} }}{{\overline {AB} .\overline {AC} }}
Ta suy ra:

\begin{array}{l}\frac{{S\left[ {MNP} \right]}}{{S\left[ {ABC} \right]}} = 1 - \frac{{S\left[ {BMP} \right]}}{{S\left[ {ABC} \right]}} - \frac{{S\left[ {CNM} \right]}}{{S\left[ {ABC} \right]}} - \frac{{S\left[ {APN} \right]}}{{S\left[ {ABC} \right]}} \\\Rightarrow \frac{{S\left[ {MNP} \right]}}{{S\left[{ABC}\right]}} = 1 - \frac{{\overline {BM} .\overline {BP} }}{{\overline {BC} .\overline {BA} }} - \frac{{\overline {CN} .\overline {CM} }}{{\overline {CA} .\overline {CB} }} - \frac{{\overline {AP} .\overline {AN}}}{{\overline {AB} .\overline {AC} }} \\\Rightarrow \frac{{S\left[ {MNP} \right]}}{{S\left[ {ABC} \right]}} = \frac{{\overline {BM} .\overline {CN} .\overline {AP} - \overline {CM} .\overline {AN} .\overline {BP} }}{{\overline {AB} .\overline {BC} .\overline {CA} }} \\ \end{array}
:D

Cái này kiểm tra lại đi chứ không đúng
Dấu trừ trong biểu thức thì phải thay bằng dấu cộng

kien10a1
15-05-2011, 11:47 AM
I.59)Bổ đề Haruki

Bổ đề:Cho AB và CD là hai dây cung không cắt nhau của cùng một đường tròn và P là một điểm bất kì trên cung AB không chứa CD của đường tròn ấy.Gọi E và F lần lượt là giao điểm của PC,PD với AB.Thế thì giá trị biểu thức sau là không đổi:

\frac{{AE} . {BF}}{EF}

Chứng minh:
[Only registered and activated users can see links]

Em tính thẳng cái tỉ số này luôn. Qua B kẻ đường thẳng song song PC cắt DP tại K. Ta có\frac{BF}{EF}=\frac{BK}{PE}\Rightarrow \frac{AE.FB}{EF}= \frac{AE.BK}{PE}=\frac{AC.BK}{BP}

Chú ý rằng BCD và KPB đồng dạng góc- góc nên \frac{BK}{BP}=\frac{BD}{CD}
Vậy \frac{AE.BF}{EF}= \frac{AC.BD}{CD} không đổi

RAIZA
01-09-2011, 09:50 AM
I.30)Định lí Miobiut
[Only registered and activated users can see links]

Keke đánh y nguyên báo cho đỡ mệt:))

Định lí:Cho ngũ giác lồi

Chứng minh:

I.30) Định lý Miobiut.
Cho ngũ giác lồi A_{1}A_{2}A_{3}A_{4}A_{5}. Đặt S_{A_{i}A_{i+1}A_{i+2}}=a_{i}; A_{6} \equiv A_{1}; A_{7} \equiv A_{2}. Thế thì: S^{2}-S.(\sum_{i=1}^{5} a_i)+(\sum_{cyc}a_{i}a_{i+1})=0
Bổ đề: Cho \vec{a};\vec{b};\vec{c};\vec{d}. Khi đó:(\vec{a} \wedge \overrightarrow{b})(\overrightarrow{c} \wedge \overrightarrow{d})+(\overrightarrow{a} \wedge \overrightarrow{c})(\overrightarrow{d} \wedge \overrightarrow{b})+(\overrightarrow{a} \wedge \overrightarrow{d})(\overrightarrow{b} \wedge \overrightarrow{c})=0
Chứng minh:
(\overrightarrow{a} \wedge \overrightarrow{b})(\overrightarrow{c} \wedge \overrightarrow{d})+(\overrightarrow{a} \wedge \overrightarrow{c})(\overrightarrow{d}\wedge \overrightarrow{b})+(\overrightarrow{a}\wedge \overrightarrow{d})(\overrightarrow{b} \wedge \overrightarrow{c})

=\vec{a} \wedge [(\vec{c} \wedge \vec{d})\vec{b}]+\vec{a} \wedge [(\vec{d} \wedge \vec{b})\vec{c}]+\vec{a} \wedge [(\vec{b} \wedge \vec{c})\vec{d}]

=\vec{a} \wedge [(\vec{c} \wedge \vec{d})\vec{b}+(\vec{d} \wedge \vec{b})\vec{c}+(\vec{b} \wedge \vec{c})\vec{d}]

=\vec{a} \wedge \vec{0}

=0

Trở lại định lý, áp dụng bổ đề ta có:
(\overrightarrow{A_{1}A_{2}} \wedge \overrightarrow{A_{1}A_{3}}).(\overrightarrow {A_{1}A_{4}} \wedge \overrightarrow{A_{1}A_{5}})+(\overrightarrow{A_{1 }A_{2}} \wedge \overrightarrow{A_{1}A_{4}}).(\overrightarrow{A_{1 }A_{5}} \wedge \overrightarrow{A_{1}A_{3}})+(\overrightarrow {A_{1}A_{2}} \wedge \overrightarrow{A_{1}A_{5}}).(\overrightarrow {A_{1}A_{3}} \wedge \overrightarrow{A_{1}A_{4}}) = 0
\Rightarrow S{[A_{1}A_{2}A_{3}]}.S{[A_{1}A_{4}A_{5}]}+S{[A_{1}A_{2}A_{4}]}.S{[A_{1}A_{5}A_{3}]}+S{[A_{1}A_{2}A_{5}]}.S{[A_{1}A_{3}A_{4}]} = 0 (*)
Mà \bigtriangleup{A_{1}A_{2}A_{3}} cùng hướng \bigtriangleup{A_{1}A_{4}A_{5}}; \bigtriangleup{A_{1}A_{2}A_{4}} ngược hướng \bigtriangleup{A_{1}A_{5}A_{3}}; \bigtriangleup{A_{1}A_{2}A_{5}} cùng hướng\bigtriangleup{A_{1}A_{3}A_{4}}
Nên từ (*) ta có: a_{1}a_{4}-(S-a_{2}-a_{4})(S-a_{1}-a_{3})+a_{5}(S-a_{1}-a_{4})=0
Thu gọn đẳng thức này ta có đpcm.

vnclubchemgio
05-09-2015, 09:48 AM
I.40 Định Lí Blaikie
Định lí: Cho tam giác ABC và đường thẳng d sao cho d cắt BC,CA,AB lần lượt ở M,N,P. Gọi S là 1 điểm bất kì trên d. Gọi M',N',P' lần lượt là điểm đối xứng của M,N,P qua S. Khi đó AM',BN',CP' đồng quy tại một điểm P và ta gọi P là điểm Blaikie của d và S đối với tam giác ABC.


[Only registered and activated users can see links]
Chứng Minh :
Có thể cho S nằm giữa N,M.
Giả sử AM' cắt BN' tại I . Ta chứng mình I,C,P
thẳng hàng .
Xét tam giác BN'M với 3 điểm I,C,P . Ta cần cm :
\frac{IB}{IN'}.\frac{P'N'}{P'M}.\frac{CM}{CB}=1

Xét tam giác PBN' với 3 điểm thẳng hàng A,I,M' trên 3 cạnh :
\frac{AP}{AB}.\frac{IB}{IN'}.\frac{M'N'}{M'P}=1 (1)

Xét tam giác MBP với 3 điểm thẳng hàng C,A,N trên 3 cạnh :
\frac{CM}{CB}.\frac{AB}{AP}.\frac{PN}{MN}=1 (2)

Nhân 2 vế (1),(2) và rút gọn , chú ý MN=M'N' ta được :
\frac{IB}{IN'}.\frac{NP}{M'P}.\frac{CM}{CB}=1
Chú ý là NP=P'N' và P'M=M'P nên ta có đpcm. :))


Mở rộng định Lí Blaikie: [Only registered and activated users can see links]